You are on page 1of 119

PROBLEMS AND SOLUTIONS IN

COMMUTATIVE ALGEBRA
Mahir Bilen Can
mcan@tulane.edu
Disclaimer: This file contains some problems and solutions in commutative algebra as
well as in field theory. About first hundred problems are those that we encountered at some
point probably between years 2003 and 2005. We do not claim correctness of those solutions
(neither of the other solutions). Read them at your own risk. However, we do appreciate if
you send us corrections and suggest new problems and solutions.1
Notation: Unless otherwise stated all rings are assumed to be commutative with unity.

1. Find two ideals I and J in a ring R such that I J 6= I J.


Solution.
Let I = J be the ideal generated by x in the polynomial ring R = k[x]. Then
I J = I = (x). Since the product of two ideals consists of finite sum of products of
elements of I and J, the ideal product I J is equal to (x2 ) which is different from (x).

2. Definition: ideals I and J from R are called co-prime, if their sum I + J is equal to R.
Show that if I and J are two co-prime ideals in a ring R, then I J = I J.
Solution.
For all ideals I and J the inclusion I J I J is clear. To prove the other inclusion
we observe the simple fact that, for any ideal K of R, the following equality KR = K
is true. Hence, assuming I and J are co-prime, on the one hand we have I J =
(I J)(I + J) = (I J) I + (I J) J. On the other hand, (I J) I J I and
(I J) J I J. Equality is now obvious.

3. Definition: A multiplicative subset S of a ring R is a multiplicative submonoid of R.


Let S be a multiplicative subset in a ring R and I be an ideal.
1

We thank Professor Lex Renner for his comments and critical eye on some of the problems with faulty
solutions. We thank afak zden, also.

(a) Show that S 1 I := {a/s : a I, s S} is an ideal in the localized ring


S 1 R = {r/s : r R, s S}.
(b) Show that the localization commutes with quotients: S 1 R/S 1 I
=
1
S (R/I). Here we are abusing the notation on the right: of course
R/I is localized at the image of S in R/I.
Solution.
(a) Let a1 /s1 and a2 /s2 be two elements from S 1 I. And let r/s S 1 R . Then
r/s a1 /s1 + a2 /s2 = (s2 ra1 s1 sa2 )/s1 s2 S 1 I. Therefore S 1 I is an ideal.
(b) Elements of S 1 (R/I) are of the form r/
s where bar denotes the images of elements
of S in R/I. If we start with an element r/s of S 1 R, then r/
s makes sense. So we
1
1
can define the homomorphism : S R S (R/I) by (r/s) = r/
s. By its
construction is surjective. How about its kernel? Suppose r/
s = 0 in S 1 (R/I).
Then there exists s0 in the image of S in R/I such that r s0 = 0 that is rs0 I. But
it is always the case that rs0 /ss0 = r/s in S 1 R. Therefore r/s S 1 I. This shows
that the kernel of is S 1 I, hence we get the desired isomorphism.
Remark 0.1. Let S be the complement of a prime ideal P in a ring R and I be an
ideal contained in P so that I S = . By the above proven fact, RP /IP
= (R/I)P .
Some authors write IRP for what we are calling IP ; the ideal generated by the image
of I in RP .

4. Let : A B be a ring homomorphism by which B has a finitely generated A-module structure. It is easy to verify that for any multiplicative
submonoid S A the image (S) B is a multiplicative submonoid also.
Show that the induced homomorphism S : AS B(S) gives B(S) a finitely
generated AS -module structure. (Here, without loss of generality we assume that 0
/ (S). Otherwise, B(S) = 0, which is a finitely generated
AS -module.)
Solution.
Let {b1 , . . . , br } be a generating set for B as an A-module. Let a/b be an element of
the localized ring AS . For c/(d) B(S) with c B, d S the action of AS on B(S)
is defined by
c
(a) c
a

=
.
b (d)
(bd)

Since B is generated by {b1 , . . . , br } as an A-module, c is of the form a1 b1 + ar br =


(a1 )b1 + + (ar )br for some a1 , . . . , ar A. Therefore,
r

(a1 )b1 + + (ar )br X ai bi


c
=
=

(d)
(d)
d 1
i=1
proving that {b1 /1, b2 /1, . . . , br /1} is a generating set for B(S) as an AS -module.

5. Let P1 , . . . , Pm be a finite set points from Cn such that P1 6= Pj for all j


{2, . . . , m}. Find an explicit polynomial F (x1 , . . . , xn ) C[x1 , . . . , xn ] which
takes constant value 1 on P1 and 0 on Pj for all j {2, . . . , m}.
Solution.
Let (ak1 , . . . , akn ) Cn denote the coordinates of Pk for k = 1, . . . , n. Since P1 6= Pj ,
there exists smallest index rj {1, . . . , n} such that ajrj 6= a1rj . Define gj (x1 , . . . , xn )
by
xr ajrj
gj (x1 , . . . , xn ) := j
a1rj ajrj
It is clear that gj (Pj ) = 0 and gj (P1 ) = 1. The product of gj s for j = 2, . . . , m is the
desired polynomial F .

6. Definitions: Given two ideals I and J, the ideal quotient I : J is defined to be the ideal
{h R : hJ I}. The radical of an ideal I, denoted by rad(I), is the ideal consisting
of elements r R such that some power rn , n N of r lies in I. Logically, we call an
ideal I radical if rad(I) = I.
Notation: Given an ideal I of a polynomial ring k[x1 , . . . , xn ], V (I) k n denotes the
set of points a k n such that all polynomials from I vanishes on a.
Let I and J be two radical ideals. Show that the ideal of the Zariski closure
V (I) \ V (J) coincides with the quotient ideal I : J.
Remark 0.2. (a) Hilberts Nullstellensatz in commutative algebra says that for an
algebraically closed field k, and for any finitely generated polynomial ideal J
the ideal of the vanishing locus of J is equal to radical of J. In other words,
I(V (J)) = rad(J).
(b) The complement V (I) \ V (J) of V (J) in V (I) need not to be an algebraic set (it
is an open subset of V (I)). It doesnt make sense to talk about the ideal of an
open subset. We must take its closure so that we can talk about the ideal of the
closed set.
3

(c) The quotient I : J need not be a radical ideal in general. But if I and J are
radicals then so is I : J; suppose f n I : J for some n. Then f n hn I for any
h J. But I being radical, f h I hence f I : J.
(d) f R is not a zero divisor in R/I if and only if I = I : f . In this case, the variety
of the ideal generated by f and I has dimension one less than V (I).
For obvious reasons we assume that V (I) is not equal to V (J) (otherwise there is
nothing to prove).
Let V (I) \ V (J) be a point, hence there exists a polynomial f in J such that
f () 6= 0.
We claim that each element h of I : J vanishes on , that is to say h() = 0. Indeed,
hf I. But f () 6= 0, so every element h of I : J vanishes on V (I) \ V (J).
It follows that I : J lies in the ideal of the closure of the complement. Conversely,
if we take a polynomial f vanishing on the closure of V (I) \ V (J), then obviously it
vanishes on V (I) \ V (J). Then for any h J, f h vanishes on all of V (I); f vanishes
on the complement and h vanishes on V (J). Thus f h I since I is radical. But then
f belongs to the quotient ideal I : J.

7. Let M be a finitely generated R-module and a R an ideal. Suppose


: M M is an R-module map such that (M ) aM . Find a monic
polynomial p(t) R[t] with coefficients from a such that p() = 0. Here, aM
is the module consisting of all finite sums of elements of the form bm, where
b a and m M .
Solution.
The solution technique is important here. Let {x1 , . . . , xm } be a generating set for M
as an R-module. By hypothesis, for each i = 1, . . . , m we have
X
(xi ) =
ai,j xj ,
(1)
for some ai,j a. We define Ai,j to be the operator i,j ai,j e, where e is the identity
endomorphism of M and i,j is the Kroneckers delta function.
P
It is clear from (1) that j=1 Ai,j xj = 0 for all i = 1, . . . , m. In other words, the matrix
m
of operators A := (Ai,j )m
i,j=1 annihilates the column vector v = (xj )j=1 . Notice that
we can consider M as an R[]-module, and that Ai,j R[]. Thus A is a matrix over
R[]. Therefore, its adjugate makes sense and multiplying Av = 0 on the left by the
adjugate gives us (det A)xj = 0 for all j = 1, . . . , m. Consequently, det A annihilates
all of M . Expanding the determinant we obtain a monic polynomial p in with entries
from a. Furthermore, p() = 0 on M .

8. If M is a finitely generated R-module such that aM = M for some ideal a,


then there exists x R such that 1 x a and xM = 0.
Solution.
By the previous problem we observe that the identity operator id on M satisfies a
monoic polynomial: p(id) = idr + a1 idr1 + + ar id = 0 for some aj a. Therefore,
if we define x = 1 + a1 + + ar , then x 1 a and furthermore xm = 0 for all
m M.

9. If a R is an ideal such that every element of 1 + a is invertible, M is a


finitely generated R-module, and aM = M , then M = {0}.
Solution.
Let x R be as in the previous problem; 1 x a and xM = {0}. In particular
x 1 a, hence x = 1 + x 1 is invertible. It follows from xM = {0} that M = {0}.

10. Let Jac(R) denote the intersection of all maximal ideals in R. (Jac(R) is
called the Jacobson radical of R.) Show that x Jac(R) then 1 xy is
invertible for any y R. Conversely, if 1 xy is invertible for all y R, then
x belongs to all maximal ideals.
Solution.
Suppose x is from Jac(R). If 1 xy is not invertible, then it is contained in a maximal
ideal m of R. In particular, since x is from m, we see that 1 m which is a contradiction.
Conversely, suppose that 1xy is invertible for all y R. If x does not lie in a maximal
ideal m, then the ideal generated by x and m is equal to R. Hence, xy + m = 1 for
some y R and m m. In this case, m = 1 xy m, so it is not a unit, contradicting
with our initial assumption. Therefore, x m.

Definition: Have a taste of Zorns lemma: A Noetherian ring is a ring in which every
non-empty set of ideals has a maximal element. For other definitions and properties of
Noetherian rings see 0.60.
Fact 0.3. Artin-Rees Lemma: Let a be an ideal in a Noetherian ring R and
let M be a finitely generated R-module. If N M is a submodule, then there
exists a positive integer k such that for all n k, an M N = ank ((ak M ) N ).
11. Let R be a Noetherian ring and a be an ideal such that every element of
1 + a is invertible in R. Show that n>0 an = (0).
5

Solution.
Let M denote n>0 an . Obviously, M is a R-submodule of a. By Artin-Rees lemma,
there exists n such that an+i M = ai (M an ) for all i 0. Since set theoretically
we have M an+i = M for any n + i, we get M = ai M for any i 0. By Problem 9
above, we get M = 0.

12. If a is an ideal such that every element of 1 + a is invertible, M a fininitely


generated R-module and M 0 M a submodule, then M 0 + aM = M implies
that M 0 = M .
Solution.
We consider the R-module M/M 0 . Our assumption M 0 + aM = M implies that
aM/M 0 = M/M 0 . Hence, by Problem 9 M/M 0 = {0}, or equivalently, M = M 0 .

13. If a is an ideal such that every element of 1 + a is invertible, M a fininitely


generated R-module. Show that the elements u1 , . . . , un generate M if and
only if the images u1 , . . . , un generate M/aM as an R-module
Solution.
The implication () is obvious. We prove the converse. Suppose u1 , . . . , un generate
M/aM as an R-module. Let {u1 , . . . , un } denote a set of preimages of ui s, and let
M 0 denote the submodule generated by ui s. It is clear that M 0 + aM = M , hence by
Problem 12 it follows that M 0 = M .

Definition: A local ring is a ring with unique maximal ideal.


14. Let (R, m) be a Noetherian local ring and suppose that the images of the
elements a1 , . . . , an m generate m/m2 as a vector space. Show that a1 , . . . , an
generate m as an ideal.
Solution.
We denote by M the maximal ideal m viewed as an R-module, and denote by a the
maximal ideal m viewed as an ideal. The solution is now an application of Problem 13.

15. In the notation of the previous problem, a1 , . . . , an generates m/m2 as a vector


space, then a1 , . . . , an generates m minimally, that is to say none of ai s is
redundant.
6

Solution.
Towards a contradiction, without loss of generality, assume that a1 redundant; a1 =
r2 a2 + + rn an for some ri R. Then, modulo m2 , a1 , . . . , an are linearly dependent
which is a contradiction.

Definition. An ideal I 6= (1) is primary if f g I implies either f I, or g m I for


some m N.
16. Prove that if Q is primary, then rad(Q) is a prime ideal. Furthermore, in
this case, rad(Q) is the smallest prime ideal containing Q.
Notation: If P denotes the prime ideal rad(Q), then Q is called P -primary.
Solution.
Let f g rad(Q), hence (f g)n Q for some n N. Since Q is primary, either f n Q,
or g nm Q for some m N. In other words, either f rad(Q), or g rad(Q)
implying that rad(Q) is a prime ideal.
If M is a prime ideal such that Q M , then rad(Q) M because of the following
two things: First, for any two ideals I and J, I J implies rad(I) rad(J). Indeed,
f rad(I), then f n I for some n N, hence f n J. In particular, f rad(J).
Secondly, if an ideal J is prime, then J is equal to its own radical. To see this it is
enough to show that rad(J) J whenever J is prime. Let f rad(J), hence f n J
for some n N. It follows primeness that f J.
We apply this observation to our original problem. If M is a prime ideal containing
Q, then rad(Q) M . Therefore, we conclude that rad(Q) is the smallest prime ideal
containing Q whenever Q is primary.

17. Let M be a finitely generated R-module and S R be a multiplicative subset


of R. Show that MS = 0 if and only if an element s of S annihilates M , that
is to say, sM = 0.
Solution.
If s annihilates M , then for any m/r MS , we have m/r = sm/sr = 0/sr = 0, thus
MS = 0. Conversely, assume that MS = 0, that is m/r = 0 for every m M and
r S. By definition this holds if there is an s R such that s(1 m 0 r) = 0.
Thus s m = 0. Now, this s is specific to m. Since M is finitely generated, there
exists a finite generating set {m1 , . . . , mn } of M , and there exists a corresponding set
{s1 , . . . , sn } of annihilators. Since R is commutative, the product s1 sn annihilates
all mi s, hence it annihilates whole of M .

Definition: Support of an R-module M is the set of all prime ideal p such that Mp
is non-trivial. Here, Mp is the localization of the module at the multiplicative subset
R p.
18. Let M be a finitely generated R-module. Show that a prime ideal p is in the
support of M if and only if the annihilator ideal ann(M ) of M is contained
in p.
Solution.
By Problem 17 we see that Mp = 0 if and only if there exists s R p such that
sM = 0. But then ann(M ) cannot be contained in p. This proves one implication.
For the converse, suppose that p is in the support of M . Then by the previous problem
again, there cannot be any s R \ p such that sM = 0. Hence ann(M ) must be
contained in p.

19. Show that a short exact sequence of R-modules:

1
2
0 A1
A2
A3 0

(2)

gives rise to a left exact sequence

1
2
HomR (A1 , N )
HomR (A2 , N )
0 HomR (A3 , N )

(3)

Solution.
Given a homomorphism f : A3 N , we pull it back to a homomorphism from

2
A2 to N by f 2 : A2 N . Hence we get a homomorphism HomR (A3 , N )
HomR (A2 , N ).

Next, let us see that 2 is injective: suppose 2 f = 2 g for some f and g from
HomR (A3 , N ). Then, 2 f = 2 g. But 2 is surjective by exactness (1). Thus, f and
g agree on every point of A3 showing that they are the same functions. Therefore 2
is injective.
Next, let us see that im(2 ) ker(1 ), namely 1 2 = 0. Let f HomR (A3 , N ). Then
1 2 f = f 2 1 = f 0 = 0 again by exactness of (1). Therefore im(2 ) ker(1 ).
Finally, let us see that im(2 ) ker(1 ): let f : A2 N be in the kernel of 1 ,
namely f 1 = 0. Define g : A3 N as follows, let g(a3 ) be the value f (a2 ) for
any a2 A2 such that a2 and g is well-defined. Notice that g2 = f . Therefore
im(2 ) = ker(1 ) and (2) is left exact.

20. Give an example of a module N and a short exact sequence such that
HomR (, N ) does not give a short exact sequence.
Solution.
Let p Z be a prime and consider the following exact sequence of Z-modules:
p

0 Z Z Z/p 0
Apply HomZ (, Z) and check that the result is not an exact sequence.

21. Let G be an abelian group and write G ' Zn Gtorsion . Show that HomZ (G, Z)
=
Zn .
Solution.
Let f HomZ (G, Z), then f is determined by the images to the generators of the
copies of Z in G. The reason for not affected by the torsion part of G is the following:
if a Gtorsion , then n a = 0 for some n Z. Then 0 = f (0) = f (n a) = nf (a) since
f is a Z-module homomorphism. Therefore effect of the torsion part of G is 0 showing
that an f is determined by the images of the generators of the copies of Z in G. So,
the result follows.

22. In the category of R = k[x1 , . . . , xn ]-modules show that HomR (R(a), R) '
R(a).
Solution.
Note that 1 in R(a) is an element of degree a. Therefore, a homomorphism f
HomR (R(a), R) ' R(a) of graded R-modules maps 1 to a degree a element in R that
is f (1) is of degree a. Now, since an R-module homomorphism R is determined by its
value on 1; we have an isomorphism of R-modules f HomR (R(a), R) R via
f 7 f (1). Thus if we declare 1 in R to be of degree a, namely, if we regard the image
as the graded module R(a), then we get an isomorphism of graded R-modules.

23. What is Z/2 Z Z/3?


Solution.
Let a b Z/2 Z Z/3. If b is 0 or 2, then a b = 0 0. If b = 1, then we can replace
it by 4 (since 4 1 mod 3) and get a b = 0 0 again. Therefore Z/2 Z Z/3 = 0.

24. More generally, show that Z/a Z Z/b ' Z/ gcd(a, b).
Solution.
First of all remember that gcd(a, b) of two integers is the largest (hence unique) integer
that divides both a and b. Furthermore, there are integers x and y such that gcd(a, b) =
ax + by.
Now, that said, define : Z/a Z Z/b Z/ gcd(a, b) by r s 7 rs mod gcd(a, b).
This map is a well-defined homomorphism of Z-modules. It is injective because: if
rs = 0 mod gcd(a, b), then rs = n(ax + by) for some n Z. Thus r s = 1 rs =
1 n(ax + by) = 1 nax = nax 1 = 0 1 = 0.
Surjectivity is clear by 1 r 7 r mod gcd(a, b). Therefore is an isomorphism of
Z-modules.

25. Show that R M is a right exact functor on the category of R-modules,


but it need not be an exact functor.
Solution.
Let

1
2
0 A1
A2
A3 0

be a short exact sequence of R-modules and let M be some other R-module. We want
to show that the following is a right exact sequence:
A1 M A2 M A3 M 0
Since the maps are defined by ai m 7 i (ai ) m, right exactness is straightforward
to check.
A counter example to exactness of tensoring is the following:
p

0 Z Z Z/p 0
This is an exact sequence of Z-modules. Assume that Z Z/p gives us an exact
sequence:
p
0 Z Z Z/p Z Z Z/p Z/p Z Z/p 0
p
But Z Z Z/p
= Z/p Z Z Z/p
= Z/p is the 0 map which, on the contrary, was
supposed to be an injection. Contradiction.

10

26. For R-modules M , N , and P , prove that


HomR (M R N, P ) ' HomR (M, HomR (N, P )).

Solution.

Consider the map f HomR (M R N, P ) HomR (M, HomR (N, P )) 3 (f ),


defined by (f )(m) = f (m) HomR (N, P ). Here, it should be clear that f (m)
is a homomorphism from N to P for every m M .
Let us prove that is an isomorphism of R-modules: If (f ) = 0, then f (m n) = 0
for every m n M R N , hence f is the zero homomorphism in HomR (M R N, P ).
So, is injective.
If g HomR (M, HomR (N, P )), then for every m M , g(m) is an R-homomorphism
from N to P . So, define f : M N P by f (m n) = g(m)(n). It is clear that
we get a homomorphism. Also, by definition, (f ) = g. So, is surjective, hence an
isomorphism of R-modules

27. It is easy to see that every direct sum of modules gives an exact sequence.
Prove that the converse is not true; there is an exact sequence
0 A B C 0
such that B  C

Solution.
Consider
0 Z/2 Z/4 Z/2 0
with obvious maps. This is an exact sequence. However, Z/4  Z/2

Z/2.

28. For a short exact sequence of R-modules

1
2
0 A1
A2
A3 0

show that A2
= A1 A3 if and only if there is a homomorphism 2 (or a
homomorphism 1 ) with 2 2 = idA3 (with 1 1 = idA1 ).
Solution.

Let A2 A1 A3 be an isomorphism. We have the following commutative diagram:

11

A1

A2

A1

A3

id
0

id

A1 A3

A3

An element of A1 A3 is of the form (a1 , a3 ) for unique a1 A1 and a3 A3 . Define


2 : A3 A2 as follows: given a3 A3 there exists unique a2 A2 such that (a2 )
= (0, a3 ) since is an isomorphism. Set 2 (a3 ) = a2 . Then by the commutativity of
the diagram it is clear that 2 2 (a3 ) = a3 .
Conversely, suppose that we have a homomorphism 2 : A3 A2 such that 2 2 =
idA3 . Then define : A1 A3 A2 by ((a1 , a3 )) = 1 (a1 ) + 2 (a3 ). This is a well
defined R-module homomorphism. Let us see why it is an isomorphism.
Suppose ((a1 , a3 )) = 1 (a1 ) + 2 (a3 ) = 0. Then 1 (a1 ) = 2 (a3 ). Compose this
equality with 2 to get 0 = a3 . Then 1 (a1 ) = 0. But 1 is injective, so a1 = 0.
Therefore is injective.
Now, let a2 A2 . Then 2 (a2 2 2 (a2 )) = 2 (a2 ) 2 (a2 ) = 0, therefore a2
2 2 (a2 ) im(1 ), say, 1 (a1 ) = a2 2 2 (a2 ). Then ((a1 , 2 (a2 )) = 1 (a1 ) +

2 (2 (a2 )) = a2 , therefore A1 A3
A2 is surjective and hence an isomorphism.
The case of 1 is proven similarly.

Definition 0.4. We recall some very basic definitions from homological algebra. A
chain complex is a sequence of abelian groups with homomorphisms between them:

4
3
2
1
0

C3
C2
C1
C0
0

satisfying
n+1 n = 0 for all n 0.
Last condition ensures that the image of n+1 lies in the kernel of n , hence next
definition makes sense: ith Homology group of the chain complex (C, ) is defined by
Hi (C) = ker i /im i+1 .
These groups measure how far the chain complex from being exact. Note that the
chain complex (C, ) does not need to be of infinite length, however, a finite length
chain complex can be extended by adding trivial groups and trivial connecting homomorphisms in between them.

12

29. Compute the homology of the complex 0 V1


V0 0 where V1 = V0 = k 3
and is:

1
0 1
1 1
0
0 1 1
Solution.


ac=0
1
a

b V1 : a + b = 0 which is spanned by 1. So, H1 is isoKernel of is:

c
b + c = 0
1
morphic
to
k.
For
H
,
note
that
the
image
of

is
spanned
by
the
image of the basis
0
1
0
0

0 , 1 , 0 under . It is straightforward to see that the image of this basis

0
0
1
is nothing but the same basis. Thus, H0 is trivial.

30. Prove that for a complex V : Vn . . . V0 0 of finite dimensional vector spaces, the following equality is always true:
X
X
(1)i dim Vi =
(1)i dim Hi (V )
i=0

i=0

This alternating sum is called the Euler characteristic of the complex. It is


clear that the complex is exact then its Euler characteristic is 0.
Solution.
This follows from the basic linear algebra fact that if Vi+1 and Vi are two finite dimensional vector spaces and i+1 is a linear map between them, then
dim(ker i+1 ) + dim(imi+1 ) = dim(Vi+1 ).
Alternating sum of these equations (for i 0) gives us the desired equality.

31. Let k be a field. Suppose k[x0 , x1 , . . . , xn ] is graded by


 degree. Show that ith
graded component has vector space dimension n+i
.
i
Solution.
P
The vector space basis for the ith graded piece is {xp00 xp11 . . . xpnn : t pj = i}. So, the
problem is equivalent to finding the number of ways of distributing
i candies to n + 1

n+1
children named x0 , x1 , . . . , xn . Of course, this is done in i different ways.

13

Definition 0.5. Let M = nZ Mn be a Z-graded R-module. The numerical function


HM : Z Z defined as
HM (n) = dimk (Mn )
is called the Hilbert function of M .
The power series
PM (z) =

HM (t)z t

is called the Hilbert series of M.


32. What is the Hilbert series of k[x1 , . . . , xn ]?
Solution.
Using Problem 31, we see that it is nothing but 1/(1 t)n .

33. Find the Hilbert series of k[x1 , x22 , . . . , xnn ].


Solution.
The set of monomials contained in the ith graded piece is
nin
2
{xi11 x2i
2 . . . xn : i1 + 2i2 + + nin = i}
Q
1
Therefore, the Hilbert series is given by r1 1t
r.

34. Let P be a function P : N Z such that the associated difference function


P (i) := P (i) P (i 1) is a polynomial with rational coefficients (for sufficiently large i). Show that P itself is a polynomial with rational coefficients
and has degree one greater than P .
Hint: Use induction on the degree s of the difference polynomial. The
 base case is
i
trivial. If the leading coefficient of P is as , then define h = as s! s+1
, and compute
h. By construction, P h will have degree s 1.
Solution.


i
i1
As given in the hint, obviously, h is of degree s 1, and h(i) = as s! s+1
as s! s+1
.

i1
Computing this we find h(i) = as s! s . Clearly h is of degree s. At the same
time, the leading coefficient of h is as . Thus, the degree of h P = (P h) is
s 1. By the induction hypothesis, this says that P h is a polynomial with rational
coefficients and of degree s. But h is a polynomial with rational coefficients of degree
s + 1, hence P is has the same type. This finishes the induction.

14

35. Let M be a finitely generated, Z-graded module. Show that there exists a
polynomial f (x) Q[x] such that for i  0, HM (i) = f (i).
Definition 0.6. The polynomial f (i) is called the Hilbert polynomial of M , written
HP (M, i).
Solution.
Applying induction on the number of variables in the ring over which M is defined, the
base case is trivial. So, suppose it is true for n 1 variables {x1 , . . . , xn1 }. Using the
xn
M , we can build an exact sequence:
homomorphism, multiplication by xn , M (1)
x

n
M coker(xn ) 0
0 ker(xn ) M (1)

Since multiplication by xn kills both ker and coker, they can be regarded as modules
over the polynomial ring of n 1 variables. (Of course, they are finitely generated).
By exactness, HM (i) HM (i 1) Q[i]. We are done by the previous exercise.
Definition 0.7. Let I be a homogenous ideal in R = k[x0 , . . . , xn ], hence it corresponds
to a projective variety Y = V (I) in Pn with ring of global sections R/I. Let us write
the Hilbert polynomial of Y (or that of I) as the Hilbert polynomial of the coordinate
ring R/I:
am m
am1 m1
HP (R/I, i) =
i +
i
+ ...
m!
(m 1)!
Then we define the dimension of the projective variety Y Pn as m, and its degree as
am .

36. Compute the Hilbert polynomial of Pn and find its degree.


Solution.
The coordinate ring of Pn is k[x0 , .. . , xn ]. Thus, for large is we have HP (Pn , i) =
HR (i) = dimk (k[x0 , . . . , xn ]i ) = n+i
= n+i
. Obviously this is a polynomial in i with
i
n
1 n
leading term n! i . Recall that for a Hilbert Polynomial as!s is + . . . the degree is the
dimension of the variety and as is the degree of the variety. Thus, the degree of Pn is
1.

Definition 0.8. Given n, d > 0, let M0 , . . . , MN be all the monomials of degree d in


1. We define a mapping d : Pn PN
the n+1 variables x0 , . . . , xn where N = n+d
n
by sending the point P = (a0 , . . . , ad ) to the point d (P ) = (M0 (a), . . . , MN (a)). This
is called d-uple embedding of Pn in PN .
15

37. Let : k[y0 , . . . , yN ] k[x0 , . . . , xn ] be the homomorphism defined by sending


yi to Mi (in the notation of the above definition). Let a be the kernel of
. Show that a is a homogenous prime ideal, and V (a) is an irreducible
projective variety in PN .
Solution.
Recall that an ideal is called homogenous if it is generated by homogenous polynomials.
A relation among Mi s is zero if and only if that relation is a homogenous polynomial.
For example, if M0 = x20 , M1 = x0 x1 , M2 = x22 , then M0 M2 M12 = 0 which is of course
y0 y2 y12 . This is because each Mi has the fixed degree d. Therefore, the kernel of
is generated by homogenous elements. The image of is a subring of k[x0 , . . . , xn ]
generated by Mi s. Obviously being an integral domain is preserved for the subrings
of integral domains. Therefore the quotient k[y0 , . . . , yN ]/ ker is an integral domain
showing that ker is a prime ideal.

38. Show that the variety, d (Pn ) defined in Problem 37 has the ring of global
sections equal to k[y1 , . . . , yN ]/ ker , namely it is given as the locus of ker .
Solution.
The kernel of is generated by the polynomials that are zero when evaluated in the
monomials M0 , . . . , MN in place of y0 , . . . , yN . Therefore these are exactly the generators of the vanishing ideal on the set of points (M0 (a); . . . ; MN (a)) in PN . Therefore
the variety d (Pn ) is cut out by the kernel of .
39. Show that d is a homeomorphism of Pn onto V (ker ).
Solution
First, let us see that d is an injection. Suppose (M0 (a); . . . ; MN (a)) = (M0 (b); . . . ; MN (b)),
namely d (a) = d (b) for some a, b Pn . Observe that ai s cannot be all zero (otherwise Mi (a) = 0 for all i). Say ai 6= 0. Among monomials of degree d we have
adi and ad1
aj for j = 0, . . . , n. Since (M0 (a); . . . ; MN (a)) = (M0 (b); . . . ; MN (b)) in
i
N
P , we have (M0 (a)/adi , . . . ; MN (a)/adi ) = (M0 (b)/bdi ; . . . ; MN (b)/bdi ) in AN . Therefore we have ad1
aj /adi = bd1
bj /bdi for j = 0, . . . , n. But then aj /ai = bj /bi hence
i
i
(a0 ; . . . ; an ) = (b0 ; . . . ; bn ) in Pn . Thus d is injective. Since a map is surjective onto
its image. Therefore d is a bijection. It is clear that it is continuous (on each open
piece yi 6= 0 in PN it is given by continuous maps). It is enough to show that it has a
continuous inverse. The inverse map 1
d maps (M0 (a); . . . ; MN (a)) to a. Since at each
1
affine open xi 6= 0, this reduces to d ((M0 (a)/adi , . . . , MN (a)/adi )) = (a0 /ai , . . . , an /ai )
and among Mj (a)/adi we have aj /ai for j = 0, . . . , n, we see that the inverse is basically
a projection. Therefore, it is continuous hence d is a homeomorphism.

16

40. Find the degree of d-uple embedding of Pn in PN .


Solution
By definition, we want to compute the Hilbert polynomial of the k-vector space
k[y0 , . . . , yN ]/ ker ,
where (yi ) = Mi (x). This ring is isomorphic to the subring k[M0 , . . . , MN ] of k[x0 , . . . , xn ]
generated by all the monomials of degree d. Therefore it suffices to compute
L the Hilbert
polynomial of the graded (by degree) vector space k[M0 , . . . , MN ] =
Si . Obviously
Si = 0 if i is not a multiple of d. Let i = rd for some nonzero r, then Si is equal to
 the
n+rd
ith piece of k[x0 , . . . , xn ]. Thus the Hilbert polynomial is equal to n+rd
=
as a
rd
n
function of r = 0, 1, . . . . This is a polynomial in r;


n + rd
(n + rd)(n + rd 1) (rd + 1)
=
.
n!
n
The leading coefficient of this polynomial is dn /n!. Therefore, the degree (leading term
times n!) of d (Pn ) is dn .

Fact 0.9. If partially ordered set S has the property that every chain has an upper
bound in S, then the set S contains at least one maximal element.
Definition 0.10. The nilradical of a ring is the set of all nilpotent elements of the
ring.
41. Show that a nilradical is an ideal. Furthermore, show that the nilradical is
equal to the intersection of all prime ideals of the ring.
Solution.
Let n denote the nilradical of the ring R. Let x and y be two elements from n,
and suppose n, m N are such that xn = y m = 0. Binomial theorem implies that
(x + y)n+m = 0, hence n is closed under addition. Let a R. Since R is commutative,
(ax)n = 0, hence ax n. Therefore, n is an ideal. To prove that n is the intersection
of all prime ideals, we first make the following simple observation: If P is prime ideal
P R, then the quotient ring R/P (which is an integral domain) does have any
nilpotent elements. Therefore, n P . This proves that n P : prime P .
For the opposite inclusion, it suffices to show that for any non-nilpotent element x,
there is some prime ideal that does not contain x. Towards this contradiction we
assume that P : prime P n is non-empty. Let S be the set of all ideals that does not
contain any power of x. Since n S, we know that S is non-empty. Furthermore,
if I1 I2 is a nested sequence of elements from S, then Ii S. Hence, by
Zorns lemma (Fact 0.9) S has a maximal element M S. Notice that if we prove M
17

is a prime ideal, then we succeed in our goal that there is a prime ideal that does not
contain x.
Assuming M is not prime, we take two elements a, b R M such that ab M .
The ideals Ma generated by a and M , and Mb , generated by b and M both properly
contain M , hence both of them contains a power of x: ay + m1 = xp and bz + m2 = xq .
Since (ay + m1 )(bz + m2 ) = abyz + aym2 + bzm1 + m1 m2 lies in M we conclude that
xp xq = xp+q is contained in M also. This contradiction shows that M is a prime ideal,
hence our proof is complete.

Fact 0.11. Recall that the Jacobson radical Jac(R) of a ring R is the intersection of
all maximal ideals of R. Given an ideal I Jac(R) and a finitely generated R-module
M , Nakayamas Lemma says that
IM = M implies M = 0.
An R-module P is projective if there exists a module K such that P
some free module F . Equivalently, every short exact sequence

K ' F for

0 N M P 0
h

splits; there exists P M such that f h = idP .


Remark 0.12. Being projective is transitive in the following sense: if M is a projective
B-module (hence M is a direct summon of a free B-module F ) and if B is an A-algebra
with free A-module structure, then M is a projective A-module as well. The reason
for this transitivity is that F is a free A-module, hence M is a direct summand of a
free A-module.
42. Show that the following are equivalent for a finitely generated Z-module M :
(a) M is projective;
(b) M is torsion free;
(c) M is free.
Solution. (c) = (b) and (c) = (b) are obvious. Suppose that M is torsion free and
let {m1 , . . . , mr } be a generating set for M . Define : M Zr by (mi ) = ei and
extend by linearity, where ei is the ith standard vector in Zr . Since none of the mi s are
torsion, this is a well defined map of Z-modules. Furthermore, it is an isomorphism.
So (b) = (c). Finally, since M is a direct summand of a free module, it is torsion
free; (a) = (b). So, we are done.

18

Definition 0.13. For an integral domain R, a fractional ideal is an R-submodule A


of the fraction field K of R such that for some nonzero element d R the following
inclusion holds:


da a
:
A R.
dA=
b
b
Equivalently, A = d1 I for some ideal I R and a nonzero d R. In particular,
any ideal of R is a fractional ideal. The product of two fractional ideals A = d1 I,
B = r1 J is another fractional ideal AB = (dr)1 IJ. This operation defines a monoid
structure on the set of all fractional ideals of R. Indeed, the identity fractional ideal is
the ring R itself.
The unit group of this monoid contains all principal ideals. Obviously, the set of all
fractional principal ideals forms a subgroup of the unit group of the monoid. Invertible
fractional ideals modulo its subgroup of principal fractional ideals is called the class
group of R. Thus the class group of an integral domain R measures how far is R from
being a PID.

43. Show that in an integral domain R with fraction field K, if a fractional ideal
A is invertible, then it is a projective R-module.
Solution.
Assume that A is an invertible fractional ideal. Let A1 be its inverse. Then a1 a01 +
+ an a0n = 1 for some ai A and a0i A1 since AA1 = R. Let S be a free R
module of rank n say generated by y1 , . . . , yn . Define : S A by (yi ) = ai and
extend it by linearity. Define also : A S by (c) = c(a01 y1 + + a0n yn ). This
makes sense because ca0i R. Obviously, = idA , so A is a direct summand of F .
In other words, A is a projective module.
Remark 0.14. The converse of this problem is true also: if a fractional ideal A is
projective, then it is invertible

44. Show that the ideal (2, 1 + 5) of Z[ 5] is a projective Z[ 5]-module but


not free.
Solution.

Note that Z[
5] = 1 Z 5

Z,
therefore
Z[
5]

is a free Z-module of rank 2.


If I := (2, 1 + 5) were free Z[ 5]-submodule of Z[ 5], then it would be of rank
one. Thus there would be a
single generator;
I ' Z[ 5] for some I. But it

is easy to see that 2 and 1 + 5 are Z[ 5]-linearly


independent; Z[ 5] cannot

generate I. Therefore I cannot be a free Z[ 5]-module. However, I 2 is the ideal (2).


19


Since (2) is an invertible
ideal
with
inverse
(1/2)Z[
5], I is an invertible ideal with

inverse I (1/2)Z[ 5], hence I is a projective module.

45. Use Nakayamas Lemma to prove that a finitely generated projective module over a local ring is free.
Solution.
Let {w1 , . . . , wn } be a minimal set of generators for a finite projective module M and
let F be a free module of rank n. Consider the following surjection : F M :
Xn
Xn
(
ai ei ) =
ai w i ,
i=1

i=1

where ei s are the generators


for F . Since we have declared wi s to be the minimal
P
generating set for M , if ni=1 ai wi = 0, then we should have ai s in the maximal ideal
m of the local ring. Otherwise, they would be units and that would contradict the
minimality of wi s. Thus ker mF . Now, for M being a projective module and
having a surjection on it, we get a splitting of F via an injection : M F
such that = idM . Therefore we can write F = (M ) ker . Since we have
the containments m(M ) (M ) and ker mF and since ker is disjoint from
(M ), we see that ker m ker . So, ker = m ker . Now, by Nakayamas Lemma,
ker = 0, hence is an isomorphism.

Definition 0.15. Left Derived Functors: Let C denote the category of R-modules
and suppose F is a right exact, covariant, additive (preserving addition of homomorphisms) functor from C to C. The left derived functors Li F (N ) of an R module N are
defined as follows: take a projective resolution of N (for instance, a free resolution)
and apply the functor F to the exact sequence omitting N . The new sequence is a
complex and Li F (N ) is defined as the ith homology of this complex.
46. Let C denote the category of graded C[x]-modules and let F stand for the
functor C[x] (C/x) on C. Compute the left derived functors Li F of N =
C[x]/x2 for all i.
Solution.
Here is a projective resolution for N :
x2

0 (x2 ) C[x] N 0
Applying F to it, we get the following complex, say P :
x2 1

01

0 C[x] C[x]/x (x2 ) C[x] C[x]/x C[x] C[x] C[x]/x 0


20

Then
L0 F (N ) = H0 (P ) = (C[x] C[x] C[x]/x)/x2 C[x] C[x]/x
= (C[x]/x)/(x2 C[x]/x)

= (C[x]/x)
because (x2 C[x]/x) = 0. Therefore L0 F (N ) = (C[x]/x).
Next,
L1 F (N ) = H1 (P ) = ker(x2 1)/im(0 1)
= ((x2 ) C[x] C[x]/x)/0
= (x2 C[x]/x)/0 = 0.
Of course, Li F (N ) = Hi (P ) = 0 for all i > 1.

Definition 0.16. Tor. Left derived functors of the tensor products are called Tor.
What we have computed in the previous problems are T orCi [ x] (N, C[x]/x).
i
(R/r, M ).
47. Let M be an R-Module and let r R. Compute the R-modules T orR

Solution.
Consider the free resolution
r

0R
R R/r 0
Apply R M to this by omitting R/r to get
1

0 R R M R R M 0
r

i
This is equal to 0 M
M 0. Therefore T orR
(R/r, M ) = M/r M and
1
T orR
(R/r, M ) = {m M : r m = 0}.

i
48. Let M be an R-module and let r R. Compute the R-modules T orR
(R/r, M ).

Solution.
Consider the free resolution
r

0R
R R/r 0
Apply R M to this by omitting R/r to get
r1

0 R R M R R M 0
21

0
This is equal to 0 M
M 0. Therefore T orR
(R/r, M ) = M/r M and
1
T orR (R/r, M ) = {M M : r m = 0}.

49. Prove that for a homogenous polynomial f of degree d and a homogenous


ideal I R, there is a graded exact sequence:
0 R(d)/I : f R/I R/hI, f i 0.
Hint: Clearly, 0 hI, f i/I R/I R/hI, f i is exact. How can you get a graded
map from R to hI, f i/I?
Solution.
Obviously the exact sequence given in the hint is graded. Consider the graded map
f
R(d)
hI, f i/I. The kernel is {g R : f g I} : f after a glance at the definition
of I : J. Thus, R(d)/I : f
= hI, f i/I. Thus, replacing hI, f i by R(d)/I : f in the
exact sequence given in the hint, we get the graded exact sequence:
0 R(d)/I : f R/I R/hI, f i 0

50. Show that a given f R is a nonzero divisor on R/I if and only if I : f = I.


Solution.
Recall that I : J = {h R : hJ I}. Suppose now that I : f = I, therefore if for
some h R it happens that hhf i I, then h I. So, f (h mod I) = f h mod I 0
mod I implies that h I. Thus, f is a nonzero divisor. Conversely, if f is a nonzero
divisor on R/I then for any h R with the property that hhf i I we must have
h I.

Remark 0.17. We defined the Hilbert polynomial of a ring in Problem 35. Suppose
the initial term of the Hilbert polynomial of a quotient ring R/I is given by
am m
i +
HP (R/I, i) =
m!
If f is a homogenous linear form which is a nonzero divisor on R/I, then we have
HP (R/hI, f i, i) = HP (R/I, i) HP (R/I, i 1)
am
=
im1 + . . .
(m 1)!
We deduce that, by slicing with the hyperplane defined by f the dimension drops by
one while preserving the degree.
22

Definition 0.18. Let M be an R module. An element x of R is called M -regular if it


is not a zero divisor on M . An ordered sequence of elements x = x1 , . . . , xn from R is
called M -regular if the following two conditions hold:
(a) xi is M/(xi , . . . , xi1 )-regular for i = 1, , n.
(b) M/xM 6= 0
A weak M -regular sequence is defined by requiring only the first condition.
51. Show that given an R-module M over a local ring (R, m), a weak M -regular
sequence x m is always M -regular.
Solution.
Let x m be a weak M -regular sequence and assume M/xM = 0 that is M = xM .
But then by Nakayamas Lemma M must be trivial.

52. Show that x, y(1 x), z(1 x) is an R-sequence, but y(1 x), z(1 x), x is not
where R = k[x, y, z] a polynomial ring. Here, by an R-sequence we mean an
R-regular sequence for R regarded as a module over itself.
Solution.
Multiplication by y(1 x) on R/xR is the same as multiplication by y on k[y, z](
=
R/xR). So y(1 x) is R/xR regular. And multiplication by z(1 x) on R/(x, y(1
x))R is the same as multiplication by z on k[z](
= R/(x, y(1 x))R. So, z(1 x) is
R/(x, y(1 x))R-regular. Furthermore, the ideal I = (x, y(1 x), z(1 x)) is equal to
(x, y, z). Therefore R/IR
= k 6= 0. Thus x, y(1 x), z(1 x) is an R-sequence.
Since (y(1 x), z(1 x), x) is the same ideal with I above, we must show that this
sequence fails to be a weak R sequence. Multiplication by z(1 x) on R/(y(1 x))R is
not regular: z(1 x) y = 0 in R/(y(1 x))R. Therefore (y(1 x), z(1 x), x) cannot
be an R-sequence.

53. Let R be a Noetherian local ring, M a finite R-module, and let x be an


M -sequence. Show that every permutation of x is an M -sequence.
Solution.
Since every permutation is the product of adjacent transpositions, it suffices to show
that x1 , . . . , xi1 , xi , , xn is M -regular. Note that for the module M/(x1 , , xi1 )M ,
(xi , . . . xn ) is a regular sequence. Therefore, by induction, it will suffice to handle the
case of n = 2. We want to show that x2 , x1 is M -regular assuming X1 , x2 is M -regular.
Of course, we may assume that xi is not a unit, otherwise x1 M = M , hence we would
23

be done. Now, look at the kernel of multiplication by x2 on M . Since x2 must be


regular on M/xi M , ker(x2 ) xi M . If z ker(x2 ), then z x1 m for some m M.
Since xi (x2 m) = x2 (x1 m) = 0, and x1 is regular on M, (x2 m) must be 0. But then
m ker(x2 ). Therefore ker(x2 ) = xi ker(x2 ). Since x1 is a non unit, it is in the
maxima ideal, hence any element 1 + (x1 ) is regular on M. Finally we have to show
that x1 is regular on M/x2 M . Assume otherwise; there exists m M/x2 M such that
xi m x2 M hence xi m = x2 m0 for some m0 M . Here m0 cannot be in xi M otherwise
the equation would imply m x2 M. So, x2 is not regular on M/x1 M, contradiction.
Thus, x2 , x1 is a regular sequence too.
Definition 0.19. Let A be a subring of a ring B and b B. Then b is call integral
over A if b is a root of a monic polynomial with coefficients from A, that is if there is
a relation of the form bn + a1 bn 1 + + an = 0 with ai A.
54. Let A B be rings. Show that an element b B is integral over A if and
only if there exists a ring C between A and B such that b C and C is
finitely generated as an A-module
Solution.
() Let b B be integral over A. The let C be the ring generated by A and b, that is
C := A[b]. Obviously A CsubsetB. Let us see that C is indeed a finitely generated
module over A. Obviously any element of C is a polynomial in b with coefficients of
A. Since bn = (a1 bn 1 + + an ), any element of C can be written as an A-linear
combination of 1, b, b2 , . . . , bn1 hence C is finitely generated A-module.
() Suppose there exists an intermediate ring C which is a finitely generated
PA module.
Then C = Awi + . . . Awn for some wi C. Let b C. Then wi b =
j aij wj for
n
n1
i = 1, . . . , n. But then we get a relation b + a1 b
+ + an = 0 (by the Cayleys
theorem; expanding det(bij aij )).

55. Let B be an integral domain and A B a subring such that B is integral


over A. Then A is a field if and only if B is a field.
Solution.
() Suppose B is an integral domain which is integral over a field A. Then any element
b in B satisfies a monic polynomial over A i.e, bn + a1 bn1 + + an = 0 for some
ai A. Since an is invertible, from this equation we see that b is invertible. Therefore,
every element of B is invertible, hence it is a field.
() Suppose B is a field and A B a subring such that B is integral over A. It is
enough to show that for a A, the inverse a1 of a also lies in A. Assume otherwise;
b := a1 6 A. But then it satisfies a monic polynomial over A, that is bn + a1 bn1 +
24

+ an = 0 for some ai A. Multiplying this relation by an1 , we see that b A


which is a contradiction. Therefore A is a field, also.

56. Let B be an extension ring of A which is integral over A. Let P B be a


prime ideal. Then P is maximal if and only if P A is maximal in A.
Solution.
P A is obviously a prime ideal. The composition A , B B/P gives the injection
A/A P B/P. Note that B/P is integral over A/A P. Therefore, by the previous
problem that A/A B is a field if and only if B/P is a field. Hence A P is maximal
in A if and only if P is maximal in B.

57. Let A B be rings and suppose B is integral over A. Let m be a maximal


ideal in A. Show that there exists a prime P in B lying over m, that is
P A = m. Furthermore, any such P is maximal ideal of B.
Solution.
Let us first see that mB
B. Assume contrary that mB 6= B, then there exists bi B
P6=
n
and i m such that i=1 bi i = 1. Set C := A[b1 , ...bn ], then C is finite
P over A and
mC = C. Let C = Au1 + +Aur for some uj C. Then, we get ui = ij uj for some
ij m. Therefore := det(ij ij ) satisfies uj = 0 for every j. Hence C = 0.
But since 1 C, = 0. On the other hand mod m simply by its expansion.
Hence 1 m, a contradiction. Now, since mB 6= B, it is contained in a prime ideal,
say P. By the previous problem we know that P must be a maximal ideal.

58. Let A B be rings ans suppose B is integral over A. Let p be a prime ideal
of A. Show that there exists a prime p B such that P A = p. Furthermore,
there is not inclusion between primes in B that lie of p.
Localizing the exact sequence 0 A B at p, we get an exact sequence
0 Ap Bp = B A Ap
in which Bp is an extension ring of Ap and integral over Ap . Using the following commutative diagram:
Ap

Bp

25

we see that the prime ideals of B lying over p corresponds bijectively with the maximal
ideals of Bp lying over the maximal ideal pAp Ap . Hence by the previous problem
we are done.

59. Let A B be rings and suppose B is a finitely generated A-module( hence


integral over A.) Show that for a prime ideal of A, there are only finite
number of prime ideals in B that lie over p.
Solution.
First we assume A to be a local ring with maximal ideal m. Since B is finitely generated
module over A, B = Awi + + Awr for some wi B. Then, we the quotient ring
B/mB becomes a vector space over A/m with a generating set {wi }. Every prime
P B containing mB gives a vector space P/mB , B/mB. Also note that by the
previous problems we know that if any prime lying over a maximal must be maximal
too. Therefore these primes are coprime to
Q each others. But then by the Chines

Remainder Theorem we see that B/mB = i Pi /mB. Since each Pi /mB contributes
to the dimension of the vector space, there must be finitely many Pi 0 s. So, in the local
case we are done. For the general case we make use of the diagram:
Ap

Bp

Since primes containing p A correspond to the maximal ideals of Bp over pAp in the
local ring Ap , we reduce to the local case. Hence we are done.

26

Note: Next two problems have been added to the file on January 2015.
Definition 0.20. Let p be a prime number and consider the ring of p-adic integers
i+1
Zp , which is constructed as the inverse limit lim Z/pi Z. Let AQ
Z for
i denote Z/p

simplicity. By definition, this is the ring of sequences (ai )i0 i=0 Ai which satisfy
the compatibility condition with respect projections ji : Aj Ai defined by
ji (aj

mod pj+1 ) = ai

mod pi+1 for all j > i.

60. Prove that there exists a bijection between Zp and the set of all formal
power series of the form
0 + 1 p + 2 p2 + , where i {0, 1, . . . , p 1}.

Solution.
i+1
Z, i = 0, 1, . . . , we assume that
Let (ai )
i=0 be an element from Zp . As ai Ai = Z/p
i+1
ai is a positive integer less than p . Set 0 = a0 {0, . . . , p 1}.

Since a1 = a0 mod p, a1 a0 is divisible by p. Since a1 < p2 and a0 < p, there exists


unique 1 {0, . . . , p 1} such that a1 = a0 + p1 .
Similarly, by using projections ji , we have that a2 = a1 mod p2 , or equivalently
that a2 a1 = 2 p2 for a unique integer 2 . Since 0 a2 < p3 , we see that 2
has to be less than p. Therefore, we see that a2 = a1 + p2 = 0 + p1 + p2 2 .

Continuing
Pnin thisi manner, we conclude that the nth term of the sequence (ai )i=0 is
equal to i=0 i p for some non-negative integers 0 j < p, uniquely determined
by a0 , a1 , . . . , an1 . Therefore, the data of the element (ai ) Zp is represented by the
infinite series 0 + 1 p + 2 p2 + in a unique way.
Let us revert this process to compute i s in terms of ai s. We already know that
a0 = 0 . 1 = (a1 a0 )/p. More generally
n =

an (0 + n1 pn1 )
.
pn

61. What are the ring operations on the series representation of p-adic integers?
Solution.
P
P i
Suppose (ai ), (bi ) are two p-adic integers represented by the series
i pi and
i p ,
respectively. We first analyze what happens to addition in the
notation. Let
P series
(ci ) = (ai ) + (bi ) = (ai + bi ) be represented by the summation
i pi .

27

Since 0 = a0 , 0 = b0 , we see that the constant term 0 of the series representation


of (ci ) has to be equal to 0 + 0 mod p. Note that 0 + 0 = 0 + p0 for a unique
0 {0, 1}.
Next we determine 1 . Since c1 = a1 + b1 = (0 + 0 ) + p(1 + 1 ) Z/p2 Z is equal
to 0 + 1 p, by the uniqueness of the power series coefficients we see that 1 has to be
equal to 0 + 1 + 1 mod p. Since 0 + 1 + 1 < 2p, we see that 0 + 1 + 1 = 1 + p1
for a unique 1 {0, 1}. Thus we write 1 + 1 = 1 0 + p1 .
Similarly,
a2 + b2 = 0 + 0 + (1 + 1 )p + (2 + 2 )p2 mod p3
= 0 + (0 + 1 + 1 )p + (2 + 2 )p2 mod p3
= 0 + (0 + 1 0 + 1 p)p + (2 + 2 )p2 mod p3
= 0 + 1 p + (1 + 2 + 2 )p2 mod p3
Similar to the previous case, we write 2 for 1 + 2 + 2 mod p, hence the equality
2 + 2 p = 1 + 2 + 2 implies that 2 {0, 1}. Thus
a2 + b2 = 0 + 1 p + 2 p2

mod p3 ,

where 2 is equal to 1 + 2 + 2 mod p, and 1 is found from 0 + 1 + 1 = 1 + p1 .


P
P
P
More generally, if an = ni=0 i pi and bn = ni=0 i pi , then an + bn = ni=0 (i + i )pi ,
and n is equal to n1 + n1 + n1 mod p, where n1 is found (inductively) from
n2 + n2 + n2 = n1 + pn1 .
Next we look at what happens to the nth term of the product (ai )(bi ) = (ai bi ). Suppose
P
i pi corresponds to this product. Clearly 0 = a0 b0 mod p = 0 0 mod p. For 1
we look at a1 b1 modulo pZ, which has to be equal to a0 b0 modulo pZ. Since a1 b1 =
0 0 + (1 0 + 0 1 )p in Z/p2 , the equality a1 b1 = a0 b0 mod p is straightforward. On
the other hand, to compute 1 we need to look at the carry over from 0 0 . Indeed,
writing 0 0 as
P20 + pu1 , we see that 1 must be 0 1 + 1 0 + u1 mod p. Similarly, 2
must be u2 + i=0 i 2i mod p, where u2 is the carry-over from the previous parts.
Indeed,
a2 b2 = 0 0 + (0 1 + 1 0 )p + (0 2 + 1 1 + 2 0 )p2 mod p3
= 0 + (0 1 + 1 0 + u1 )p + (0 2 + 1 1 + 2 0 )p2 mod p3
= 0 + 1 p + (u2 + 0 2 + 1 1 + 2 0 )p2 mod p3 ,
where u2 is found from the equation 0 1 + 1 0 + u1 = u2 p + 1 . Let 2 denote
u2 + 0 2 + 1 1 + 2 0 mod p2 , hence we write 2 = u2 + 0 2 + 1 1 + 2 0 + u3 p2 .
Iterating this process we see how to multiply power series representations of p-adic
integers by carry-overs.

28

Grbner Bases:
The monomials xa : = xa11 xann in a polynomial rings k[x1 , . . . , xn ] over any field k
can be identified with the lattice points (a1 , . . . an ) Nn . A total order on Nn is
a term order if the zero vector 0 is the unique minimal element, and a b implies
a+c b+c for all a, b, and c in Nn . Given a term order , every nonzero polynomial
f k[x] has a unique initial monomial, denoted in (f ). If I is an ideal in k[x], then its
initial ideal is the monomial ideal generated by the initial monomials of the elements
of I. The monomials which do not lie in in (I) is a Grobner Basis for I with respect
to , if in (I) is generated by {in (g) : g G}. Given any polynomial f k[x], by
division algorithm, we can write f = h + r uniquely for h I and for an r k[x]
with no term of r is divisble by any in (g) for g G. Therefore, any polynomial f
can be uniquely written as a linear combination of the standard monomials. It is clear
that the set of standard monomials makes a k-vector space basis for the quotient ring
k[x]/in (I). Two important term orderings are
Pure Lexicographic Order: > if the leftmost nozero entry of is positive.
P
P
P
P
Graded Lexicographic Order: > if
>
or
=
and the
leftmost nonzero entry of is positive.
62. Fix a term order . Show that a finite subset G = {g1 , . . . gn } I is a Gr
obner
basis for I if and only if S(gi , gj ) reduces 0 mod G where
S(f, g) =

l.c.m(in (f ), in (h))
l.c.m(in (f ), in (h))
f
h
in (f )
in (h)

Solution. Type it later..

63. Fix a term ordering on R := k[x1 . . . xn ]. Let I be an ideal in R. Show that


there exists a basis B of the vector space R/I consisting of the images of
the standard monomials,the monomials that do not lie in in (I).
Solution.
Let G be a Gr
obner for I, so, by definition in (I) is generated by the monomials n (I)
for g G. We know by the division algorithm that any polynomial f is congruent
modulo I to a polynomial r which does not posses any term divisible by in (I) for
g G (hence with none of in (I) for f (I)). Note that this implies that r is in the
span of the standard monomials (monomials that do not lie in in (I)). therefore any
polynomial f R, the image R/I is a linear combination of the images of the standard
monomials.P
Assume for a second that a linear combination of standard monomials lie
in I; s := a x I. Then in (I) would be a standard monomial inside in (I)
which is absurd. Therefore we must have a = 0 for all Nn . This shows that
29

the standard monomials are linearly independent mod I. So we have shown that the
images of the standard monomials is a basis for the vector space R/I.

64. Show that a Gr


obner Basis G of an ideal I generates the ideal.
Solution.
Recall that a finite subset G I is called a Gr
obner basis for I if the initial ideal of I
is generated by the initial terms of G. Denote by the ideal generated by G. We want
to show that G = I. Observe that (in (g) : g G) = in (G) = in (I). Now, by the
previous problem we know that R/I is a vector space spanned by the monomials that
do not lie in in (I) = in (G). Therefore R/I = R/G. Since G I, this shows that
I = G.

65. Let I = hx2 + y, x + xyi. Find a Gr


obner basis for the ideal I.
Solution.
This is an application of Buchbergers Algorithm. I will use the Pure Lex with x > y.
Let fi = x2 + y and f2 = xy + y. And let G0 = {f1 , f2 }. Using pure lex, in(f1 ) = x2 and
in(f2 ) = xy. Then S(f1 , f2 ) = y f1 x f2 = y 2 x2 . Reduce y 2 x2 mod G0 : since
y 2 x2 + f1 = y 2 + y and in(y 2 + y) = y 2 is not divisible by the initial monomials of f1
and f2 , the result is f3 = y 2 y. Now, let G1 = {f1 , f2 , f3 }. Then S(f1 , f3 ) = y 3 yx2
and modulo G1 this is 0. and S(f2 , f3 ) = 0. Therefore algorithm stops here and we get
a Gr
obner basis {f1 , f2 , f3 }.

30

USEFUL GEOMETRIC INTERPRETATIONS

We repeat; all rings are assumed to be commutative unless otherwise stated.


Given a ring R we denote by Spec(R) the set of all prime ideals of R. Note that we
exclude the whole ring itself from this set, but if R is an integral domain, the zero-ideal
is assumed to be an element of Spec(R). There is a useful topology on Spec(R), called
the Zariski topology defined as follows:
For each ideal I R, let V (I) Spec(R) denote the set of all prime ideals containing
I. A closed set in Spec(R), by definition, is either
intersection of arbitrarily many sets of the form V (I), or
union of finitely many sets of the form V (I).

P
66. Show that (a) V ( Ii ) = V (Ii ); (b) V (I J) = V (I) V (J); (c) V (IJ) =
V (I) V (J) .
Solution.
We start with the first claim. If P contains the sum of the ideals
P Ii , then it contains each
ideal Ii as well. Therefore, P V (Ii ) for all i, hence V ( Ii ) V (Ii ). Conversely,
if aP
prime ideal contains all Ii s, then it contains their sum as well, hence the equality
V ( Ii ) = V (Ii ) follows.
For (b), let P V (I J) be a prime ideal containing I J. Assume that P does not
contain neither I nor J. Let a I and b J be two elements such that a I P and
b J P . Since ab belongs to both of I and J, it lies in I J, hence it lies in P . P
is a prime ideal, therefore, either a P , or b b, both of which gives a contradiction.
Therefore, either P V (I), or P V (J). Conversely, if P lies in V (I), then it of
course lies V (I J). This finishes the proof.
The proof of (c) is similar to that of (b).

Observe that a prime ideal is pulled back to a prime ideal by ring homomorphisms:
Let : A B be a ring homomorphism, P B be a prime ideal, and let a, b be two
elements from A. If ab 1 (P ), then (ab) P , hence either (a) P , or (b) P .
It follows that either a 1 (P ) or b 1 (P ). Therefore, there exists an induced
map in the opposite direction: : Spec(B) Spec(A), defined by (P ) = 1 (P ).
Definition 0.21. A basic open set in a spectrum Spec(R) is, by definition, the complement D(f ) := Spec(R) V ((f )), where (f ) is the principal ideal generated by an
31

element f R. Thus D(f ) is the set of all primes ideals which do not contain f . The
collection {D(f )}f R forms a basis for the Zariski topology.
67. Show that the induced map : Spec(B) Spec(A) of the ring homomorphism : A B pullbacks distinguished open sets to distinguished open
sets.
Solution.
Let D(a) Spec(A) be the distinguished open set associated with an element a of
A. The pre-image 1 (D(a)) consists of prime ideals Q in Spec(B) that are mapped
into D(a). If Q 1 (D(a)), then (Q) = 1 (Q) is an element of D(a). Hence,
a
/ 1 (Q), or (a)
/ Q. In other words, 1 (D(a)) = D((a)) and this is what we
wanted to prove to begin with.
Remark 0.22. Our conclusion from Problem 67 is that the induced map : Spec(B)
Spec(A) is continuous with respect to Zariski topology.

Definition 0.23. Let X be a topological space. A presheaf F (of rings, groups,


modules, or anything) on X is an assignment (of rings, groups, modules, or anything)
to each open set and satisfying certain compatibility conditions with respect to restrictions: For each pair of open sets U1 , U2 with U2 U1 , there exists a restriction map
r1,2 = rU1 ,U2 : F (U1 ) F (U2 ) that satisfies
(a) If U1 = U2 , then r1,2 is the identity map;
(b) If U3 U2 U1 , then the following diagram of restriction maps commute:
F (U1 )

r1,2

r1,3

F (U2 )
r2,3

F (U3 )
A sheaf is a presheaf F such that for every collection of open sets {Ui } with U =
the following two conditions are satisfied:

Ui ,

(a) If x1 , x2 are two elements from F (U ) and rU,Ui (x1 ) = rU,Ui (x2 ) for all Ui s, then
x1 = x2 ,
(b) If a collection elements xi F (Ui ) satisfies rUi ,Ui Uj (xi ) = rUj ,Ui Uj (xj ) for all
i, j, then there exists x F (U ) such that xi = rU,Ui (x) for all i.

32

As an example, we define a sheaf of rings OR on Spec(R). For each prime ideal P , let
RP denote the localization of R at the multiplicative set (submonoid) R P . For each
open set U of Spec(R), we define OR (U ) to be the set of functions
G
s:U
RP
(4)
P U

such that
s(P ) RP for all P U ;
for each P U there exists an open neighborhood P U 0 U and two elements
a, b in R satisfying 1) b is not contained in any prime Q U 0 , hence a/b lives in
RQ , 2) a/b from 1) is equal to s(Q) for all Q U 0 .
It is straightforward to verify that OR is a sheaf on Spec(R).
Finally, let us define the notion of the stalk at a point. Let F be a presheaf (of
groups, rings, ..) on a topological space X and let x X be a point. The stalk of F
at x, denoted by Fx is the set of germs of sections of F at x. In other words, each
element of Fx is an equivalence class pairs (U, s), where U is an open set containing
x and s F (U ) a section; the pair (V, t) is equivalent to (U, s) if there exists an open
set W U V such that s|W = t|W .
A locally ringed space is a ringed space for which stalks of the sheaf are local rings.
68. Show that (Spec(R), OR ) is a locally ringed space and furthermore the stalk
of OR at P is nothing but the local ring RP .
Solution.
Let (OR )P denote the stalk of OR at P and define : (OR )P RP by ((U, s)) = s(P ).
We claim that is an isomorphism. To prove the surjectivity, let a/b be an element
of the local ring RP , hence, a, b R and b
/ P . Then the distinguished open set
U = D(b) contains P . We define a section s OR (U ) by s(Q) = a/b for all Q D(f ).
In particular s(P ) = a/b. Therefore, is surjective.
Next we prove that is injective. Let (U, s) and (V, t) be two germs with the same
image a/b at P . Then we need to show that (U, s) and (V, t) represent the same
equivalent class. By definition given in (4) we know that for both section s and t,
there are open sets U 0 and V 0 around P and elements a1 , b1 and a2 , b2 from R such
that for all Q1 U 0 and Q2 V 0 we have b1
/ Q1 , b2
/ Q2 with s(Q1 ) = a1 /b1 and
t(Q2 ) = a2 /b2 . In particular, a1 /b1 = s(P ) = t(P ) = a2 /b2 in RP which is true if and
only if c(a1 b2 a2 b1 ) = 0 for some unit c from R P . This equality is true for all
prime ideals Q such that b2 , b1 , c
/ Q, or equivalently, for all Q D(b1 ) D(b2 ) D(c)
which is an open set. Therefore (U, s) and (V, t) represents the same germ, hence is
injective.

33

69. Given (Spec(R), OR ) let OD(f ) denote the restriction of the sheaf OR to the
open set D(f ), f R. Show that the locally ringed space (D(f ), OD(f ) ) is
isomorphic to (SpecRf , ORf ).
Solution.
Let : R Rf denote the homomorphism defined by (a) = a/1. A prime ideal of
Rf is of the form P Rf where P is a prime ideal of R such that f
/ P . It is clear
1

that (P Rf ) = P . The image of the associated map : SpecRf Spec(R). The


image of is the set of all primes in R that does not contain f . In other words,
(SpecRf ) = D(f ) in Spec(R). The induced map on the stalks is given by the
localized homomorphism
P : RP (Rf )P Rf .
Since P does not contain f , the localization (Rf )P Rf is isomorphic to RP , hence the
canonical map P is an isomorphism. Since we have isomorphisms between their stalks,
the isomorphism SpecRf D(f ) is an isomorphism between locally ringed spaces
(SpecRf , ORf ) and (D(f ), OD(f ) ).

Definition 0.24. An affine scheme is a locally ringed space (X, F ) which is isomorphic
to a pair (Spec(R), OR ) for some ring R. The isomorphism here is a local isomorphism.
A scheme is a locally rings space such that around every point x X there exists a
neighborhood x U for which the pair (U, F |U ) is an affine scheme. Here, F |U denotes
the restriction of the sheaf F to U .

Definition 0.25. A scheme (X, OX ) is reduced if for every open U X the ring
OX (U ) has no nilpotent elements.
70. Show that reducedness is a local property: (X, OX ) is reduced if and only if
for every p X the local ring (OX )p has no nilpotent element.
Solution.
Let (U, s) and (V, t) be two germs from the stalk at a point P X. Multiplication
and/or addition of these germs are done in in O(W ), where W U V is a neighborhood of P . Therefore, a germ (U, s) is nilpotent if and only if sn = 0 in a sufficiently
small neighborhood W of P . It follows that OX (U ) has nilpotent elements if and only
if the stalk (OX )P has nilpotent elements.

34

71. Let (X, O) denote an affine scheme and Ored denote the sheaf associated
with the presheaf U 7 (O(U ))red . Here, the subscript red means we are
taking the quotient of the ring O(U ) by the ideal of nilpotent elements.
Show that (X, Ored ) is a scheme. Let us denote it by Xred . Show that there
is a morphism : Xred X which a homeomorphism of the underlying
topological spaces.
Solution.
It is straightforward to verify that U 7 (O(U ))red is a sheaf. The second claim
that Xred is homeomorphic to X boils down to the fact that the nilradical (ideal
of nilpotent elements) of a ring is the intersection of all prime ideals in the ring.
See Problem 41. Indeed, let n denote the nilradical of R. The canonical quotient
homomorphism p : R R/n gives a homeomorphism between topological spaces

: Spec(R/n)
Spec(R).

(5)

72. Let R be a reduced ring and suppose we have a homomorphism f : R0 R


from another ring R0 into R. Show that there exists a unique ring homomor0
phism g : Rred
R such that f = g p, where p : R0 R0 /n is the canonical
projection into quotient of R0 by its nilradical.
Solution.
We claim that the map g : R0 /n R defined by g(p(a)) = f (a) is a well-defined
homomorphism. if p(a) = p(b), then a b is a nilpotent element in R0 . Since R has
no nilpotent elements, a b is mapped to 0 R via f . Therefore, g(p(a)) g(p(b)) =
f (a) f (b) = f (a b) = 0. In other words, g is well defined. g is a homomorphism
because
g(p(a) + p(b)) = g(p(a + b)) = f (a + b) = f (a) + f (b) = g(p(a)) + g(p(b)) for all
a, b R0 ;
g(p(a))g(p(b)) = f (a)f (b) = f (ab) = g(p(ab)) = g(p(a)p(b)) for all a, b R.
Uniqueness is clear from the definition.

73. Let f : Xred Y be a map from a reduced scheme into another scheme. Show
that there exists unique a scheme map g : Xred Yred such that f = g.
Solution.
Reducedness is a local property. The solution follows from Problem 72.

35

74. Let x be an element of a ring R. Show that the distinguished basic open
set D(x) Spec(R) is empty if and only if x is nilpotent.
Solution.
Recall that D(x) is the set of all prime ideals that do not contain x. Therefore,
D(x) = , then x lies in all prime ideals of R, hence it belongs to the nilradical.
Conversely, if x is nilpotent, it lies in all of the prime ideals, hence D(x) = .

Definition 0.26. A topological space X is called irreducible if it is not a union of two


proper closed subsets.
75. Show that a scheme is irreducible if and only if every open subset is dense.
Solution.
Assume that X is irreducible and let U X be an open subset. Since U (X U ) = X
and since X U 6= X, we must have U = X. In other words, U is dense.
Conversely, suppose that every open subset in X is dense. If X = A B for two proper
closed subsets, then X = (X A) (X B). In this case (X A) (X B) 6=
since open sets are dense. But taking complement once again we see that A B 6= X
which is absurd.

76. An affine scheme X = Spec(R) is reduced and irreducible if and only if R is


an integral domain.
Solution.
() First, assume that X is irreducible. Let a, b R be two non-zero elements such
that ab = 0. Since any prime ideal contains 0, we have D(a) D(b) = D(ab) = D(0) =
. Since D(a) and D(b) are open sets, we have a contradiction. Therefore, R is an
integral domain.
() If R is an integral domain it cannot have any nilpotent elements. Hence, Spec(R)
is a reduced affine scheme.

77. Spec(R) is irreducible if and only if R has a unique minimal prime ideal.
Solution.
Suppose X = Spec(R) is irreducible. If S is the set of all minimal prime ideals of
R, then P S V (P ) = Spec(R). This contradicts with the irreducibility of X unless
S is a singleton. Conversely, if there exists unique minimal prime ideal P , then it is
36

necessarily the nilradical of R. In this case R/n is an integral domain. Since Spec(R/n)
is homeomorphic to Spec(R), the irreducibility of the latter scheme follows from Problem 76.

Definition 0.27. Let : F G be a map (morphism) of sheaves. The presheaf


defined by U 7 ker |U is a sheaf and denoted by ker . The morphism is called
injective if the associated kernel sheaf ker is the constant sheaf 0. Equivalently, is
injective if and only if for all open sets U the homomorphisms (U ) : F (U ) G (U )
are injective.
78. Let : A B be a ring homomorphism and let f : Spec(B) Spec(A)
denote the associated morphism between affine schemes. Show that if B
is an integral domain, then is injective if and only if the corresponding
sheaf map f ] : OA f OB is injective.
Solution.
() Suppose f ] : OA f OB is injective. By definition, f ] (U ) is a ring map (induced by ) from OA (U ) to OB (f 1 (U )). If U = Spec(A), then OA (U ) = A and
OB (f 1 (U )) = OB (Spec(B)) = B, and the corresponding ring map is injective.
() Suppose that : A B is an injective ring homomorphism. We are going to
show that any homomorphism f ] (U ) : OA (U ) OB (f 1 (U )), where U FSpec(A)
is injective. Recall that the elements of the ring OA (U ) are the s : U P U AP
satisfying for each P U , there exists a neighborhood D(g) and f A such that
s(P ) = f /g AP . After this refreshing, we observe that the value of f ] (U ) on a
section s OA (U ) is defined by the compositions:
f

f 1 (U )
U

G
P U

U
AP

BQ .

Qf 1 (U )

The last map is defined as follows: For each Q f 1 (U ) Spec(B) there exists
unique prime ideal P U Spec(A) such that P = f (Q) = 1 (Q), or equivalently
(P ) = Q. Since g
/ P , and is injective, we see that (g)
/ (P ). Therefore
U (f /g) = (f )/(g) BQ . Now we are ready to check that f ] (U )(s) = f ] (U )(t)
implies that s = t. Indeed, if (f )/(g) = (f 0 )/(g 0 ), then c(f g 0 f 0 g) = 0 for
some c B Q. Since B is an integral domain, and since is injective f /g = f 0 /g 0 ,
hence f ] (U ) is injective.

Definition 0.28. A morphism of schemes is called dominant if its image is dense.

37

79. Let : A B be an injective ring homomorphism. Show that the induced


map f : Spec(B) Spec(A) is dominant.
Solution.
We are going to show that every non-empty distinguished open set D(x) in Spec(A)
intersects f (Spec(B)). Assume contrary that there exists x A such that D(x)
f (Spec(B)) = . Let Q Spec(B) be a prime ideal in B. Then f (Q) is the prime
ideal P = 1 (Q). Notice that x must be contained in P , otherwise, P D(x). It
follows that (x) is contained in every prime ideal in B, therefore, (x) is nilpotent.
Since is injective we x must be nilpotent. Then D(x) = a contradiction to our
initial assumption.

80. Let : A B be a surjective ring homomorphism. Show that f : Spec(B)


Spec(A) is a homeomorphism onto a closed subset of Spec(A) and furthermore f ] : OSpec(A) f OSpec(B) is surjective.
Solution.
The image of a prime ideal of B under f is a prime ideal in A that contains the kernel
of . Therefore, the image of is nothing but the closed set V (ker ) Spec(A). It is
clear that f is homeomorphic onto its image.
To prove the surjectivity of the sheaf map we look at the corresponding claim on stalks.
For P Spec(A) let Q Spec(B) be the prime ideal such that 1 (Q) = P . Since the
stalk of OA at P is AP and the stalk of f OSpec(B) at Q is BQ , for each a/b BQ with
e
b
/ Q, it is enough to find c/d AP such that (c/d)
= (c)/(d) = a/b. To this end,
1
let d be an element from (b). It is clear that d
/ P . Similarly, let c 1 (a). Then
e
c/d lies in AP , and furthermore, (c/d)
= a/b.

81. Show that the converse of the above problem is true: if f : Spec(B)
Spec(A) is a homeomorphism onto a closed subset of Spec(A) and if f ] :
OSpec(A) f OSpec(B) is surjective, then : A B is surjective.
Solution.
Type later.

82. Show that for any OX -module F, HomOX (E, F)


= E OX F.
Solution.

38

Consider it locally; on the left hand side we have HomOXx (Ex , Fx ) and on the right
HomOXx (Ex , OXx ) Fx . Thus
HomOXx (Ex , OXx ) Fx
= HomOXx (Ex , OXx Fx )

= HomOXx (Ex , Fx ),
so we are done.

83. Show that for any OX -modules F, G, and E,


HomOX (E F, G)
= HomOX (F, HomOX (E, G))

Solution. Think local once again. It from Problem 26.

84. If f : (X, OX ) (Y, OY ) is a morphism of ringed spaces, and if F is an


OX -module and E is a locally free OY -module of finite rank, then there is a
natural isomorphism
f (F OX f E)
= f (F) OY E.

Solution.
Type later.

Definition 0.29. A morphism f : X Y is called "locally of finite type" if there


exists a covering of Y by open affine subsets Vi = Spec(Bi ), such that for each i,
f 1 (Vi ) can be covered by open affine subsets Uij = Spec(Aij ), where each Aij is a
finitely generated Bi -algebra. The morphism f is of finite type if in addition each
f 1 (Vi ) can be covered by a finite number of the Uij .
85. Show that a morphism f : X Y is locally of finite type if and only if for
every open affine subset V = Spec(B) of Y , f 1 (V ) can be covered by open
affine subsets Uj = Spec(Aj ), where each Aj is a finitely generated B-algebra.
Solution.
Suppose that f is locally of finite type and V = Spec(B) is an affine open subset
of Y . Then it is clear from definition that there exists an open covering of the form
Vj = Spec(Aj ) for which Aj is a finitely generated B-algebra.
39

For the converse first recall the definition of a scheme: a locally ringed space (X, OX )
in which every point has an open neighborhood U such that the topological space
U , together with the restricted sheaf OX |U is an affine scheme, that is isomorphic to
(Spec(B), OSpec(B) ) for some ring B.
Thus by definition of a scheme we get an open covering consisting of affine open subschemes Vj = Spec(Bj ). By the hypothesis we have a covering of f 1 (Vj ) with affine
open subsets Uij = Spec(Aij ), where Aij is a finitely generated Bj -algebra. But this is
exactly the definition of being locally of finite type. We are done.

86. Let f and g be two elements from a ring A. Show that the basic open
subsets D(f g) = D(f ) D(g).
Solution.
By definition D(f g) is the set of all primes in A that do not contain f g. But then
none of those primes can contain neither f nor g. Therefore D(f g) D(f ) D(g).
Conversely, a prime that does not contain both f and g cannot contain f g. Therefore
D(f ) D(g) D(f g) hence we get the equality.

87. Show that {D(f )}, a family of distinguished open subsets of Spec(R), is a
covering if and only if 1 is in the ideal generated by f s.
Solution.
Suppose Spec(R) =

D(f ). Then
Spec(R) =

D(f )

(V (f ))c

!c
\
(V (f ))

= (V (hf i))c
Therefore = V (hf i). But then hf i = R.

88. Show that Spec(R) is quasi compact, that is any cover has a finite subcover.
Solution.

40

Suppose {U } is an open cover for Spec(R). Each U is covered by basic opens U =


S
D(f ). Therefore {D(f )} is an open covering for Spec(R). By the previous
problem, 1 is in the ideal geberated by f that is 1 = a1 f1 1 + . . . + ar fr r for some
ai R. Then {D(fi i )}i=1...r hence {Ui }i=1...r is an open covering by the previous
problem. Therefore Spec(R) is quasi compact.

89. Let A be a ring. Show that the following conditions are equivalent:
(a) Spec(A) is disconnected.
(b) There exists nonzero elements e1 and e2 in A such that e1 e2 = 0, e21 = e1 ,
e22 = e2 , and e1 + e2 = 1 (these elements are called orthogonal idempotents).
(c) A is isomorphic to a direct product A1 A2 of two nonzero rings.
Solution.
(a) (b) Suppose Spec(A) is disconnected that there are two disjoint open (hence
closed) proper subsets U1 and U2 with U1 U2 = Spec(A). Therefore there are distinguished opens D(f ) that cover U1 and distinguished opens D(g ) that cover U2 .
Since Spec(A) is quasi-compact, and totality of those basic opens is a covering for A,
we can choose a finite subcover: D(f1 ), . . . , D(fr ), D(g1 ), . . . , D(gs ). We can safely
assume that none of the fi s or gj s are nilpotent or unit. Otherwise corresponding
basic open would be the whole space or empty. One remark here is that for any ring
and for any element a of it, D(an ) = D(a). This is because a prime containing some
power of a will also contain it. Thus we may replace each basic open D(fi ) by D(fin )
if needed. Now since these basic opens cover Spec(A), 1 is in the ideal generated
by f1 , . . . , fr , g1 , . . . , gs . Say 1 = a1 f1 + . . . + ar fr + b1 g1 + . . . + bs gs . Before going further, we observe that D(fi ) and D(gj ) are disjoint for any i and j. Therefore
D(fi ) D(gj ) = D(fi gj ) = . Thus fi gj is nilpotent. Thus (fi gj )mij = 0 for some mij .
Let m be the maximum of those natural numbers for i = 1, . . . , r and j = 1, . . . , s.
By the previous remarks we can replace each fi and gj by fim and gjm and still get a
covering of Spec(A). Now let e1 = a1 f1 + . . . + ar fr and e2 = b1 g1 + . . . + bs gs so that
e1 + e2 = 1 and e1 e2 = 0. Furthermore, e1 e2 = e1 (1 e1 ) implies that e1 = e21 and
e2 = e22 .
(b) (c) Since we can write ae1 + ae2 = a, we have the following homomorphism

of rings A
A/(e1 ) A/(e2 ) via a 7 (ae2 , ae1 ). This homomorphism is injective
because (ae2 , ae1 ) = (be2 , be1 ) implies that (a b)e2 + (a b)e1 = 0 thus a = b. This
homomorphism is surjective since for any (x, y) A/(e1 )A/(e2 ), we have the equality
(x, y) = (e2 x, e1 y) from e1 + e2 = 1 and hence (e2 x + e1 y) = (e2 (e2 x + e1 y), e1 (e2 x +
e1 y)) = (e2 x, e1 y). Therefore A is the direct sum of two nonzero rings A1 = A/(e1 )
and A2 = A/(e2 ).
41

(c) = (a) Suppose A = A1 A2 , and let e1 = (1, 0) and e2 = (0, 1). Therefore
e1 e2 = 0, e1 + e2 = 1, e21 = e1 and e22 = e2 . Now let V1 = V (e1 ) and V2 = V (e2 ) that is
the closed subsets of Spec(A) defined by e1 and e2 respectively. If a prime p contains
e1 then it cannot contains e2 otherwise it would contain e1 + e2 = 1. Therefore V1 and
V2 are disjoint proper subsets of Spec(A). Thus Spec(A) is disconnected.

90. A morphism of schemes f : X Y is quasi compact if there is a cover of Y


by open affines Vi such that f 1 (Vi ) is quasi compact for each i. Show that
f is quasi compact if and only if for every open affine subset V Y , f 1 (V )
is quasi compact.
Solution.
Suppose that for every affine open subset V Y , the preimage f 1 (V ) is quasi
compact. Since Y is a scheme, every point has an affine open neighborhood which is
isomorphic to a spectrum of a ring. These affine neighborhoods give us a covering that
is required.
Suppose that f is quasi compact so that we have an open affine cover {Vi =
Spec(Ri )} of Y such that f 1 (Vi ) is quasi compact. Let V Y be an arbitrary
affine open subset. Since affine schemes are
Sn quasi compact, we can choose finitely
many {V1 , . . . , Vn } affine opens such that i=1 Vi = V . Now if we can show that
each open f 1 (Vi V ) is quasi compact, then we are done because a finite union of
quasi compact sets is quasi compact. Let Z := f 1 (Spec(Ri )) be the preimage of
Vi in X and let f |f 1 (Z) = g. Now we have a morphism g : Z Spec(Ri ) with
g 1 (Spec(Ri )) = f 1 (Spec(Ri )) = Z is quasi compact, and we would like to show
that any open subset V 0 Spec(Ri ) has quasi compact preimage.
S Since Z is quasi
compact we can cover it by finitely many open affines, say Z = kj=1 Spec(Bj ). So,
if we can show that the preimage of V 0 in Spec(Bj ) is quasi compact, then we are
done. Repeating it one more time we want to show that the preimage of any open
V 0 Spec(Ri ) under morphism h = g|Spec(Bi ) : Spec(Bi ) Spec(Ri ) is quasi compact. Any open subset V 0 of Spec(Ri ) is covered by finitely many basic open D(f ).
Therefore, we would like to show that these have quasi compact preimages in Spec(Bj ).
Note that for any two rings A and B, the preimage of a basic open D(f ) Spec(A)
under a morphism h : Spec(B) Spec(A) is D((f )) where : A B is the
induced homomorphism by h on the rings. But we know from problems above that
D((f )) = SpecB(f ) . Therefore the preimage of a basic open in an affine scheme is
affine hence quasi compact and we are done.

Definition 0.30. Recall that a morphism f : X Y is called "locally of finite


type" if there exists a covering of Y by open affine subsets Vi = Spec(Bi ), such that
for each i, f 1 (Vi ) can be covered by open affine subsets Uij = SpecAij , where each
42

Aij is a finitely generated Bi -algebra. The morphism f is of finite type if in addition


each f 1 (Vi ) can be covered by a finite number of the Uij .
91. Show that a morphism f : X Y is of finite type if and only if it is locally
of finite type and quasi compact.
Solution.
If a morphism f : X Y is of finite type then it is necessarily of locally finite type.
Let us see why it is quasi compact. By definition of being of finite type, there exists a
covering of Y by open affine subsets Vi = Spec(Bi ), such that for each i, f 1 (Vi ) can
be covered by a finite number of open affine subsets Uij = SpecAij . Since affine sets
are quasi compact and finite number of quasi compact subsets is still quasi compact
we see that the preimages f 1 (Vi ) are quasi compact hence one direction is done.
Conversely suppose we have the locally of finite type and quasi compact situation.
So, we have a covering of Y by open affine subsets Vi = Spec(Bi ), such that for each
i, f 1 (Vi ) can be covered by open affine subsets Uij = SpecAij , where each Aij is a
finitely generated Bi -algebra. Since f is quasi compact, by the previous question the
preimages f 1 (Vi ) are quasi compact hence can be covered by finite number of open
affine subsets Uij = SpecAij , hence we are done.

92. Conclude from the above question that f is of finite type if and only if for
every open affine subset V = Spec(B) of Y , f 1 (V ) can be covered by a finite
number of open affines Uj = Spec(Aj ), where each Uj is a finitely generated
B-algebra.
Solution.
If for every open affine subset V = Spec(B) of Y , f 1 (V ) can be covered by a finite
number of open affines Uj = Spec(Aj ), where each Uj is a finitely generated B-algebra,
then by the previous two questions we know that f is of locally finite type and quasi
compact therefore it is of finite type. Conversely if it is of finite type, then it is of locally
finite type and hence by the first question about locally finiteness above, for every open
affine subset V = Spec(B) of Y , f 1 (V ) can be covered by open affines Uj = Spec(Aj ),
where each Uj is a finitely generated B-algebra. Since f is quasi compact too, we have
finite number of Uj = Spec(Aj )s. Hence we are done.

93. Show that if f is of finite type, then for every open affine subset V =
Spec(B) Y , and for every open affine subset U = Spec(A) f 1 (V ), A is a
finitely generated B-algebra.
Solution.

43

S
This is rather long. Let ni=1 Spec(Ai ) be a finite cover of f 1 (V ) by affine open
Ui := Spec(Ai ) where each Ai is a finitely generated B-algebra. Since V Ui is an open
subset of V , it is covered by the basic open subset D(gV ) for gV V . Each of those
basic open is open in Ui so they are unions of basic opens of Ui . But D(gV )
= SpecBgV
is quasi compact as being affine. Therefore, each D(gV ) is a finite union of the basic
open from Ui , say D(gV ) = D(gVi 1 ) D(gVi r ). But this last union is equal to
D(gVi 1 gVi r ). This last one is in turn isomorphic to Spec(Ai )gVi 1 gVi r hence a finitely
geerated B-algebra. So is SpecAgV . Now note that we can cover Spec(A) by those
SpecAgV . Since Spec(A) is quasi compact, we can cover it by finitely many SpecAgi ,
where each Agi is a finitely generated B-algebra. Since they cover Spec(A), we have
the relation g1 + + gr = 1. Therefore we have a purely algebraic question; given
finitely many gi A generating the unit ideal, and each Agi is a finitely generated
B-algebra, we would like to conclude A is a finitely generated B-algebra. To show
e and show it is equal to A. To
that we will construct a finitely generated B-algebra A
start with, first put gi s in it. Then, for each i put a finitely many elements from A
such that adding 1/gi to the algebra gives Agi . Since each Agi is a finitely generated
e
B-algebra, we need finitely many elements altogether. Therefore, this finite set A,
e generates A as a B-algebra. Let
when 1/gi added, generates Agi . Let us see why A
e and
A be an element of A. Then, in Agi it is of the form ai /gim for some ai in A
l m
integer m . Therefore gi (gi ai ) = 0 for some integer l . Since gi and ai are in
e it follows g m +l A.
e Now choose a sufficiently large power m of gi for each i so
A,
i
e for each i. Now since g1 + . . . + gn = 1, we can take sufficiently
that gim will be in A
large power r of 1, so that in each term we get one of powers of the gi s gets larger
e hence A
e = A.
than m. Therefore = 1 A

Definition 0.31. Recall that a morphism is finite morphism if there exists a covering
of Y by opening affine subsets Vi = Spec(Bi ), such that for each i, f 1 (Vi ) is affine,
equal to Spec(Ai ) where Ai is a Bi -algebra which is a finitely generated Bi -module, in
other words, Ai is finite over Bi .
94. Show that a morphism f : X Y is finite if and only if for every open
affine subset V = Spec(R) of Y , f 1 (Y ) is affine, equal to Spec(A), where A
is a finite B-module.
Solution.
The direction is obvious. Conversely suppose that there exists a covering of Y
by open affine subsets Vi = Spec(Bi ), such that for each i, f 1 (Vi ) is affine, equal
to Spec(Ai ) where Ai is a Bi -algebra which is finitely generated Bi -module. Let us
first analyze the easy case that V Y is a basic open subset D(g) = Spec(Bi )g
Spec(Bi ) for some open Vi = Spec(Bi ) in the covering of Y . Let : Bi Ai be
the corresponding ring homomorphism. Then f 1 (D(g)) = D((g)) = Spec(Ai )(g) .
44

Since Ai is a finitely generated Bi -module, (Ai )(g) is a finitely generated (Bi )g -module.
Thus, for the basic open subsets of the affine opens of the covering Vi of Y , we are OK.
In general, for an affine open V = Spec(R) of Y ,
???????????????????? Answer is in Mumfords book on page 145. I will type later

Definition 0.32. A morphism f : X Y between schemes is called quasi-finite if


preimage f 1 (y) of a point y Y is a finite subset of X.
95. Show that a finite morphism is quasi-finite.
Solution.
Suppose that f : X Y is a finite morphism. Let y Y be a point and U = Spec(B)
is an affine neighborhood of x such that f 1 (U ) = Spec(A) is affine open with the
property that A is a finite B-algebra (that is finitely generated as a B-module). We
can restrict f to Spec(A) and loose nothing about the problem. Therefore we may
assume that we have a surjective morphism f : Spec(A) Spec(B) where A is a
finitely generated B-module. Since f is onto, any prime p of B is a preimage 1 (q)
of a prime q in A where : B A is the ring homomorphism inducing f . Therefore
we may say that every prime ideal of B contains the kernel of . After this remark we
may well assume that is an injection; because Spec(B) is identical to Spec(B/ker)
for our purpose. Furthermore, we may identify B with its image in A, that is B A.
Now what we exactly want to show here is that for each prime p B, there are only
finitely many prime ideals of A that lie above p. Well, this is already done as one of
the exercises above; we are done.

Definition 0.33. A sheaf of OX -modules F on a scheme X is called quasi-coherent if


X can be covered by open affine subsets Ui = Spec(Ai ), such that for each i there is
fi . It is called coherent if furthermore each Mi can be
an Ai -module Mi with F|Ui
=M
taken to be a finitely generated Ai -module.
96. Show that a sheaf of OX -,modules F on a scheme X is quasi coherent if and
only if every point of X has a neighborhood U such that F|U is isomorphic
to a cokernel of a morphism of free sheaves on U . If X is Noetherian, then
F is coherent if and only if it is locally a cokernel of a morphism of free
sheaves of finite rank.
Solution.
This solution is taken from Mumfords (red)book at page 139. Let us name some
properties that will be equivalent at the end of the proof.

45

(a) For all x X, there exists a nbhd U of x and an exact sequence of OX |U -modules
OX |U (I) OX |U (J) F|U 0
Note that this is exactly the statement that we want to prove.
fi for some
(b) There is an open affine cover {Ui } of X such that for all i, F|Ui
=M
(Ui , OX )-module Mi .
This is exactly the definition of a quasi coherent sheaf.
f for some (U, OX )-module M .
(c) For all U X affine open, F|U
=M
(d) For all V U open affines, the canonical map
(U, F) (U,OX ) (X, OX ) (X, F)
is an isomorphism.
Strategy is to prove (d) (a) (b) (c) (d). Assume (d) and let x X.
Take any open affine nbhd U of x and let R = (U, OX ). The R-module (U, F)
has a presentation
R(I) R(J) (U, F) 0
g
(I) = O | (I) , we get the exact sequence
e is nothing but OX |U and also R
Since R
X U
of sheaves
^
OX |U (I) OX |U (J) (U,
F) 0
^
Therefore we must show the isomorphism F|U
F). It is enough to do
= (U,
^
that around basic opens. Since (Uf , (U, F)) = (U, F) R Rf and we have
^
the obvious map (Uf , (U,
F)) (Uf , F), this obvious map becomes an
isomorphism by (d). This proves (a).
Next, we show (a) (b). Cover X by affine opens Ui = Spec(Ri ) on which there
are exact sequences
i

OX |Ui (Ii ) OX |Ui (Ji ) F|Ui 0


g
]
(I )
(J )
Since Ri i = OX |Ui (Ii ) , and Ri i = OX |Ui (Ji ) , the cokernel of i is a module of
e because of the previous problem. But, then we are done for (b). Next
type K
(b) (c) then (c) (d). I will show those later.

Regular functions on Pnk :


97. Let A be a commutative ring with unity and S = A[x0 , . . . , xn ] be a graded
polynomial ring over A, graded by degree. Show that (Sxi )0 = S[x1
i ]0 =
46

A[x00 , . . . , x0n ], the polynomial ring with generators x0j = xj /xi .


x0i = 1, hence we can omit it.

Of course

Solution.
Note that the first equation is definition of localization at an element xi , so there is
1
nothing to prove; it is the definition. For the second, let f /xm
i S[xi ]0 . Then f
m
piece) polynomial
must be a homogenous (this
P is crucial for f/xi to be in the zeroth
m
of degree m. That is f = x with deg(x ) = m. Then f /xi becomes a polynomial
in xj /xi . Converse is clear because xj /xi is of degree 0.

Fact 0.34. Let R = k[x0 , . . . , xn ] and let Uxi Pnk be the open subset defined by the
non-vanishing of the ith coordinate (or the complement of the closed subset V (xi ) in
Pnk ). Then, regular functions on Uxi are OPnk (Uxi ) = (Rxi )0 the 0th graded piece of the
localized ring k[x0 , . . . , xn ]xi = k[x0 , . . . xn , 1/xi ] at the element xi . Therefore, a regular
function on Uxi is of the form g/xdi where g is of degree d. By the above discussion
OPnk (Uxi ) can be identified with the polynomial ring k[x00 , . . . , x0n ] where x0j = xj /xi on
the affine open Uxi = k n (= Spec(k[x0 , . . . , xn , 1/xi ])).
98. Justify the above description of regular functions on Pnk in the case of k = C
and show that global section of Pnk are constants.
Solution.
Lets assume k = C and let us work on P1k . On Ux0 6=0 regular functions are of the form
g/xd0 because, by definition a regular function is a quotient g/h of two polynomials with
a non-vanishing condition on h. Of course on any subset of Pn , these two polynomials
must be homogenous and of the same degree in order for making a well defined function.
Since we want h to be non-vanishing on Uxi , it cannot contain any variable other than
xi because otherwise it would always have a root as a polynomial over C. Therefore,
it must be a power of xi only. This justifies that regular functions on Ux0 6=0 P1 are
of the prescribed form g/xd0 with degree(g) = d. Now let us check if there is a nonvanishing global regular function on P1 (idea will generalize to all n automatically).
So, we are looking for a function f which is like g/xd0 on Ux0 and like g 0 /xl1 on Ux1 .
Consider the overlap Ux0 Ux1 . Points (1; x1 ) for x1 6= 0 all lie in the overlap. Hence as
x1 becomes unbounded, the value of g/xd0 at such points becomes unbounded (unless
g is constant). But the value of g 0 /xl1 is either constant or goes to 0. Similarly looking
at the points of the form (x0 ; 1) for x0 6= 0 we see that f must be constant function.

Remark 0.35. Recall that the construction of the sheaf of regular functions OPnk on
Pnk depends on the graded structure of the polynomial ring. What if we use shifted
grading on the polynomial ring. Then we get a sheaf of OPnk -modules in a very special
47

way. This is very close to the sheaf of regular functions itself and its notation OPnk (m) if
we shift it by m. Explicitly, the construction is as follows: on the open Uf , OPnk (m)(Uf )
is the OPnk (Uf ) -module of elements in the 0th piece of the graded k[x0 , . . . , xn ]f -module
k[x0 , . . . , xn ](m)f , that is (k[x0 , . . . , xn ](m)f )0 .
99. Describe the space global sections of OPnk (m) explicitly.
Solution. They are homogenous polynomials of degree m in k[x0 , . . . , xn ].

100. Prove that a nonzero section of the line bundleOP1C (m) has total sum of
orders of vanishing m over P1C . Hence show that no two such bundles are
isomorphic.
Solution.
Let R = k[x0 , . . . , xn ] be the graded (by degree) polynomial ring over the field k.
Recall that OPnC (m)(Uf ) for a basic open Uf in PnC is defined to be ((Rf )0 )(m); the
mth piece of the graded ring (Rf )0 . In particular, on Uxi P1 , OPnC (m)(Uxi ) is the
1
OPnC (Uxi ) = k[x0 , . . . , xn , x1
i ]-module (or algebra) k[x0 , . . . , xn , xi ]0 (m). Therefore, a
section s OPnC (m) on Uxi is an element of k[x0 /xi , . . . , xn /xi ](m) hence deg(f ) = m+r
for some nonnegative integer r. In particular , on Ux0 P1 , it is a polynomial of degree
m + r in the variable x1 /x0 . Hence, its order of vanishing is m + r on Ux+0 . On Ux1
this section is of the form (x0 /x1 )m f . But this has a pole of order r. Therefore, sum
of the order of vanishing of s over P1 is m + r r = m.

101. Let X be the variety P2 minus an irreducible conic. Let OX (m) be the
restriction of OP2 to X. Show that OX (2) is trivial, but that OX (1) is not.
Hence OP2 is a 2-torsion element of P icX.
Fact 0.36. Suppose L is an invertible sheaf on a variety X, and that there is a global
section s of L vanishing no where. Then L is isomorphic to OX , the trivial sheaf.
Solution.
Since X is missing a conic in P2 , the equation of that conic is a non-vanishing 2 form on
X. This is an element of OP2 (2) and hence when restricted to X an element of OX (2).
By the fact above, OX (2) is trivial. How about OX (1)? Elements (sections) of OX (1)
are 1-forms. Of necessity they vanish at a point of X (because they must intersect
nontrivially). Therefore OX (1) is not a trivial sheaf. Since OX (2) = (OX (1))2 we are
done.
Remark 0.37. OX (1) is called hyperplane bundle; it attaches a hyperplane (a line
in P2 ) to every point of X. Since a hyperplane is a linear space in Pn , elements of
48

OX (1) are 1-forms. And the intersection of these hyperplanes with the variety gives
the vanishing of section on X. Since varieties of proper dimension must intersect in
Pn , the hyperplane bundle OX (1) has no non-vanishing section in it.

Interlude on ech cohomology of sheaves.


Let us assume that we have a sheaf of modules F at hand. The ech n-cochains of
F over an open cover U is the module
Y
C n (U, F) =
F(Ui0 Uin )
{i0 ,...,in }

where product runs over all indices of the opens of U. The coboundary operators
d : C n (U, F) C n+1 (C, F) are defines as follows:
d((fi0 ,...,in )) = (gi0 ,...,in+1 )
where
gi0 ,...,in+1 =

n+1
X

(1)k fi0 ,...,in |Ui

0 ,...,ik ,...,in

k=0

Thus, using previous cochain data, d gives a new cochain on the smaller opens (further
intersections). It is easy to check that d2 = 0. Thus we get a chain complex. The
cohomology modules are called the ech cohomology modules. Let us work those on
a specific example.
The constant sheaf on a space X is defined by giving Z the discrete topology, and
then setting Z(U ) to be the continuous functions from U to Z. (note: there is an easy
extension of this definition from Z to any abelian group).
102. Is it true that Z(U )
= Z for any open set U ? If not, what additional assumption on U would make this true?
Solution.
It is not true unless U is a connected open set. And otherwise it is
copies of Z as the connected components of U .

Z with as many

103. Compute the ech cohomology of the constant sheaf Z on S 2 using the
cover U consisting of U1 , the top hemisphere, U2 and U3 bottom two quarterspheres (all open are slightly enlarged to have non empty overlap).
Solution.

49

The ech complex for the open cover U and the constant sheaf Z is
d1

0 Z(U1 ) Z(U2 ) Z(U3 )


Z(U1 U2 ) Z(U2 U3 )
d2

Z(U1 U2 U3 ) 0
Since each Ui is connected, Z(Ui ) ' Z with the generators say f1 , f2 and f3 for
U1 , U2 and U3 respectively. In fact, it is obvious that fi = 1 for each i. So, an
arbitrary ech cochain looks like (hi )i=1..3 where hi = ni fi for some ni Z. Let
fij denote the restriction of fi on the open Ui Uj . Let us look at the kernel of
d : C 0 (U, Z) C 1 (U, Z). Since d((fi )) = (fj fk ), we see that kernel of d is
generated by (f1 , f2 , f3 ) because the differences fi Q
fj is 0 on the overlaps of the
domain. Therefore kerd is a rank 1 free submodule of Z(Ui ). So, H 0 (Z, U)
= Z.
Next, let f12 = (f12 , 0, 0), f13 = (0, f13 , 0) and f23 = (0, 0, f23 ) be the generators of
C 1 (U, Z) = Z(U1 U2 ) Z(U1 U3 ) Z(U2 U3 ): what is the kernel of the next
coboundary map d then? By definition d(fij ) = fij |U1 U2 U3 . Therefore images are
the restrictions of the constant functions, fij on a space of two connected component
U1 U2 U3 (check that U1 U2 U3 has two connected component and hence Z(U1
U2 U3 )
= Z Z). Thus the rank of the image of d will be 1 in Z Z. Thus, the kernel
must be of rank 2. But observe also that the image of d : C 0 (U, Z) C 1 (U, Z) must
be of rank 2. And since d2 = 0, that is the image of d : C 0 (U, Z) C 1 (U, Z) must lie
in the kernel of d : C 1 (U, Z) C 2 (U, Z), they must be the same. Thus H 1 (Z, U)
= 0.
Finally, since the image of the second d is of rank 1 in C 1 (U, Z), we get Z2 , H 2 (Z, U)
=
Z.
Given a short exact sequence of sheaves (short exact on the stalks)
0 F1 F2 F3 0
there is a long exact sequence in sheaf cohomology
. . . H i1 (F3 ) H i (F1 ) H i (F2 ) H i (F3 ) H i+1 (F1 ) . . .
Proof of this fact along the others is in Ch.III.4 of Hartshornes book. An important
instance of this short exact sequence is the ideal sheaf sequence; let X = V (I) be the
projective variety in Pn defined by the ideal I. Then we have the exact sequence of
modules 0 I R R/I 0. Exactness on the stalks gives us the exact
sequence on the sheaves on Pn , 0 IX OPn OX 0.

104. Prove that for any m, H i (OPn (m)) = 0 unless i = 0 or i = n. Use the definition
of ech cohomology and the fact that the standard open cover of Pn is Leray.
Solution. See Hartshornes book Chapter 3, Section 5, Theorem 1.
50

Highlights
(a) Localization is an exact functor.
(b) Localization commutes with quotients.
(c) HomR (, N ) is a left exact functor and it need not be exact.
(d) R M is a right exact functor and it need not be exact.
(e) Direct limit commutes with tensor products.
(f) Let s OP1C (m) be a global section. Then, on the affine open Ux0 , s is in the 0th
piece of k[x0 , x1 ](m)x0 . 0th graded piece of k[x0 , x1 ](m)x0 is {f /xd0 : deg(f ) =
m + d}. Therefore, s|U0 = f /xd0 for some homogenous polynomial f of degree
m + d. On U1 , this section will be of the form g/xr1 for some g k[x0 , x1 ] of
degree m + r. Note that on the overlap they must be divisible by xd0 and hence s
is a polynomial of degree m.
(g) PicPn = Z. In general regular sequences are dependent on the ordering of the
sequence.
(h) Let f : Spec(B) Spec(A) be a morphism of affine schemes. Then f is a continuous map between topological spaces as well as a map of sheaves OSpec(A)
f OSpec(B) on Spec(A). Let U be an open subset of Spec(A). Then f OSpec(B) (U ) =
OSpec(B) (f 1 (U )). Therefore at every open U in Spec(A) we have a ring homomorphism OSpec(A) (U ) OSpec(B) (f 1 (U )).

51

INTEGER THEORY
Next problems are added in January, 2015.
105. Suppose R is a unique factorization domain and K is its fraction field. Let
K be a root of a monic polynomial p(x) R[x]. Show that lies in R.
Solution.
Suppose p(x) = xn + an1 xn1 + + a1 x + a0 for some a0 , . . . , an1 R and = a/b
with relatively prime a, b R. Since
an
an1
a
0 = p() = n + an1 n1 + + a1 + a0 ,
b
b
b
the following is immediate:
an = b(an1 an1 bn2 a1 a bn1 a0 ).
Therefore b is a unit and a/b R.

106. Let p(x) = an xn + + a1 x + a0 be a polynomial over an integral domain R.


Let K denote the fraction field of R. If a/b K is a root of p(x), then show
that a|a0 and b|an . Here, we assume that a and b are relatively prime.
Solution.
n

Since p(a/b) = 0 we have an abn + + a1 ab + a0 = 0, or


an an = an1 ban1 + a1 abn1 a0 bn = b(an1 an1 a0 bn1 ).
Thus b divides an . Similarly,
a(an an1 an1 ban2 a1 bn1 ) = bn a0
implies that a divides a0 .

Notation: Fq denotes the finite field with q elements. In particular, if q is a prime


number, then Fq = Z/qZ.
107. Show that x3 + x + 1 is irreducible over F2 and let be a root. What are the
powers of in F2 ().
Solution.
Checking that p(x) = x3 + x + 1 is irreducible over F2 can be done in several ways, the
easiest here is to substitute 0 and 1 into p(x) and see if the results is 6= 0.
52

Next, calculate a few values inductively.


3
4
5
6
7

= 1
= 2
= 3 2 = 2 + + 1
= 3 + 2 + = 2 + 2 + 1 = 2 + 1
= 3 + = 1 = 1

Since we have reached 7 = 1 the process (exponentiation) repeats periodically with


period 7.

108. Show that a finite field F has cardinality pn for some prime number p and
n N.
Solution.
Since the characteristic of F is p, F contain the field Z/p. More precisely, the subfield
generated by 1 in F is isomorphic to Z/p. Since F is a finite dimensional vector space
over this field, by choosing a basis e1 , . . . , en for F, we see that the cardinality of |F| is
a prime power pn .

109. Show that the group of nonzero elements of a finite field F with pn elements
is isomorphic to the cyclic group Zpn 1 with pn 1 elements.
Solution.
Since non-zero elements of F is a finite abelian group, by the Fundamental Theorem
of Finitely Generated Abelian Groups, we have
F
= Za1 Za2 Zar

(6)

for some integers a1 , a2 . . . . , ar that satisfy the divisibility condition: ai |ai+1 for i =
1, . . . , r 1. (Here, Zai is the cyclic group of order ai and it is isomorphic to additive
group Z/ai Z.) It follows that any element x F satisfies the polynomial equation
xar = 1. Since xar 1 has at most ar roots over a field, |F| ar + 1. Since |F | ar
by (6) we see that |F| = ar + 1, thus F
= Zar = Zpn 1 .

110. Let x F be an element from a finite field of characteristic p. Prove that


the map F r : F F defined by F r(x) = xp is an automorphism of F .
53

Solution.
Let x, y F be two distinct elements. It is clear that xp = y p if and only if xp y p = 0,
or (x y)p = 0. Since F has no zero divisors, x y = 0, or x = y. Therefore, we
conclude that F r : x 7 xp is an injective map, hence it is a bijection. It is easy to
check that F r is a homomorphism, hence it is an automorphism of F .

Definition 0.38. Let F K be a field extension. The minimal polynomial of an


element K over F is a monic polynomial min,F (x) F [x] of smallest degree such
that min,F () = 0.
A useful and simple fact about minimal polynomial of an element K is the following:
Fact 0.39. Let F K be a field extension. The degree of the minimal polynomial
min,F (x) F [x] is the dimension of the F -vector space F ().
111. Show that min,F (x) is irreducible and uniquely determined by and F .
Solution.
Assume that min,F (x) = a(x)b(x) for some non-constant polynomials a(x), b(x)
F [x]. Without loss of generality we assume that a(x) and b(x) are monic. Since
a()b() = 0, and F is an integral domain, either a() = 0, or b() = 0. This contradicts with the degree of min, F (x) being minimal. Hence, min,F (x) is irreducible.
Uniqueness follows from the fact that the set of polynomials that vanish at is a
principal (in F [x]) hence it has a unique minimal irreducible generator up to invertible
multiples.

112. Compute the minimal polynomial of 1 + i over Q where i C is the second


root of unity.
Solution.
First we manipulate some equations to find a candidate. (i + 1)2 = 1 + 2i + 1 = 2i.
So, (i + 1)2 2(i + 1) = 2. We conclude that f (x) = x2 2x + 2 has i + 1 as a root.
Since f (x) is of degree 2, to show that it is irreducible it is enough to prove that it has
not rational root. If = a/b Q is a root of f (x), then by Problem 106 a divides 2
and b divides 1. Possibilities for a/b are then 2. But f (2) = 2, f (2) = 10.

54

113. Prove that Q( 2, 3) = Q( 2 + 3).


Solution.
The inclusion is obvious. To prove theopposite
inclusion2 we compute
the degrees
of these two fields over Q. Let denote 2 + 3. Since = 5 + 2 6 we see that
(2 5)2 = 24, or that is a root of
p(x) = x4 10x2 + 1.
By Problem 106 we see that p(x) has no rational root. Also, by Problem 115 it suffices
to show that p(x) is irreducible over Z. Assume that p(x) is reducible, hence, it is of
the form p(x) = (a + bx + cx2 )(d + ex + f x2 ) for some a, b, c, d, e, f Z. Equating
coefficients we have the following equalities and implications:
ad = 1 = a = d = 1
cf = 1 = c = f = 1
ae + bd = 0 = e = b
bf + ce = 0 = e = b
af + be + cd = 10 = 1 e2 1 = 10
The last equality is absurd since there is no square equal to 8 or 12. Therefore, p(x) is
irreducible, hence it is the minimal polynomial of over Q. In other words, thefield
extension (Q, Q()) has degree 4. On the other hand, the field extension (Q, Q( 2, 3))
has degree
at most 4, the degree of both of the field extensions

(Q, Q( 2)) and


(Q, Q( 3)) is 2. Since the Q() is a vector subspace of Q( 2, 3), the following
inequalities finishes our argument:

dimQ Q() = 4 dimQ Q( 2, 3) 4.

Next problem is a nice application of the field theory to the superpositions of functions.
114. Let f (x) be an irreducible polynomial of degree n over F and let g(x) F [x]
be any polynomial. Show that every irreducible factor of the composition
f g(x) has degree divisible by n.
Solution.
Without loss of generality we assume that f (x) is monic. Since f (x) is irreducible,
it is the minimal polynomial of any of its roots. Let be a root of an irreducible
factor h(x) F [x] of f (g(x)). In particular f (g()) = 0, hence g() is a root of f (x).
Let K and K 0 denote respectively the field extensions F () and F (g()). Clearly
F (g()) F (). Without loss of generality we assume that h(x) is monic, hence h(x)

55

is the minimal polynomial of over F . The degree of the field extension F K = F ()


is equal to deg h(x). On the other hand, the following implication is easy to verify:
F K 0 K = F () dimF K = dimF K 0 dimK 0 K.
Therefore, deg h(x) is divisible by deg f (x) = dimF K 0 .

115. Let R be a UFD (unique factorization domain) and K denote its fraction
field. Show that an element f (x) of R[x] is irreducible in R[x] if and only if
it is irreducible over K[x].
Solution.
If f (x) is irreducible in K[x] but of the form f (x) = a(x)b(x) for some a(x), b(x)
R[x] , K[x], then obviously we have a contradiction. Let us prove the converse that
the irreducibility in R[x] implies irreducibility in K[x].
First of all, let us recall some basic facts: Since R is a UFD, R[x] is a UFD, also. In a
UFD, irreducible elements are prime that is to say, the principal ideal generated by an
irreducible element is a prime ideal. Now, towards a contradiction, assume that f (x)
factorizes as in f (x) = a(x)b(x) for two polynomials a(x), b(x) from K[x]. Let c1 be
the product of the denominators of coefficients of a(x) and let c2 denote the product
of the denominators of coefficients of b(x). Define c = c1 c2 . Then cf (x) = a0 (x)b0 (x),
where a0 (x) = c1 (x) R[x] and b0 (x) = c2 b(x) R[x]. Since (f (x)) R[x] is a prime
ideal, and since cf (x) (f (x)), it follows that either a0 (x) (f (x)) or b0 (x) (f (x)).
Without loss of generality we assume that a0 (x) = d(x)f (x) for some d(x) R[x].
Then cf (x) = a0 (x)b0 (x) = d(x)f (x)b0 (x) implies c = d(x)b0 (x). Hence, we conclude
0
that both d = d(x) and b0 = b0 (x) are from R, thus invertible in K. But then b(x) = cb2
is a unit in K[x], hence f (x) is irreducible in K[x].

116. Let F be a field and F (x) denote the field of rational functions over F for
some variable x. Let t F (x) be a rational function t = P (x)/Q(x) where
P (x) and Q(x) are two relatively prime polynomials from F [x].
(a) Show that the polynomial P (X) tQ(X) as an element of F (t)[X] is
irreducible and has x as a root.
(b) Show that the dimension of the field extension F (t) F (t)(x) = F (t, x)
is equal to dimF (t) F (x) = max{deg P (x), deg Q(x)}.
Solution.
(a) It is clear that P (x) tQ(x) = 0. Let us prove the irreducibility of P (X) tQ(X).
By Problem 115 it is enough to prove it in R = F [t] for which K = F (t) is the
56

fraction field. To this end, assume that P (X) tQ(X) is reducible over R that there
exists two polynomials At (X) and Bt (X) from F [t][X] = F [t, X] = F [X][t] such that
P (X) tQ(X) = At (X)Bt (X). Without loss of generality we assume that the t-degree
of At (X) is 0, and the t-degree of Bt (X) is 1. Thus we write Bt (X) = tU (X)+V (X) for
some U (X), V (X) F [X]. In particular P (X) tQ(X) = At (X)V (A) + tAt (X)U (X),
or P (X) = At (X)V (X) and Q(X) = At (X)U (X). Since P (X) and Q(X) are relatively
prime, At (X) must be a unit, which implies that P (X) tQ(X) is an irreducible
polynomial.
(b) It is obvious that the X-degree of P (X) tQ(X) as a polynomial over F (t) is the
maximum of the degrees of P (X) and Q(X). By Fact 0.39, it is enough to compute
the degree of the minimal polynomial of = x over F (t). If c(t) F (t) denotes the
coefficient of the highest X-degree term in any of the polynomials P (X) or tQ(X), then
1
(P (X) tQ(X)) F (t)[X] is monic, and furthermore,
the polynomial ft (X) := c(t)
irreducible by Part (a). Therefore, ft (X) is the minimal polynomial of x over F (t),
hence its X-degree, which is max{degX P (X), degX Q(X)}, is the dimension of the
field extension F (t) F (t)(x).

Notation and a simple fact: Let K be a field and let K1 and K2 be two subfields. The
smallest subfield of K that contains both K1 and K2 is denoted by K1 K2 .
117. Let K1 and K2 be two finite extensions of a field F all contained in a large
field K. Prove that the F -algebra K1 K2 is a field if and only if
dimF (K1 K2 ) = (dimF K1 )(dimF K2 ).

Solution.
Let a1 , . . . , am and b1 , . . . , bn denote F -vector space bases for K1 and K2 , respectively.
Clearly {ai bj }i,j spans K1 K2 as an F -vector space.
If K1 F K2 is a field, then its action on K1 K2 , which is defined by linearly extending
x y ai bj = (xai )(bj y), where x K1 , y K2 ,
gives a K1 K2 -vector space structure on K1 K2 . Since K1 K2 is an F -vector space,
the dimension of K1 K2 as an F -vector space is (dimF K1 K2 )(dimK1 K2 K1 K2 ). Since
dimF K1 K2 mn, and since dimF K1 K2 = mn we see that mn = dimF K1 K2 .
Next we prove the converse. Since dimF K1 K2 = mn, then we know that {ai bj } is a
basis for K1 K2 . Define f : K1 F K2 K1 K2 by extending a b 7 ab by linearity.
We first show that f is an F -vector space isomorphism. Its linearity
P is clear. It is also
clear that f is surjective. To prove the injectivity, assume that f ( (i,j)I ri,j ai bj ) =
57

ri,j ai bj = 0 for some ri,j F , (i, j) I [m] [n]. Since {ai bj } is a basis of
K1 K2 we must have that ri,j = 0 for all (i, j) I, hence f is an injective linear map.
Since F -dimensions of K1 F K2 and K1 K2 are the same, f is a linear isomorphism.
Next we show that f is a ring isomorphism. With what we have so far, it suffices to
show that f respects multiplication:

X
X
X
f
ri,j ai bj
si0 ,j 0 ai0 bj 0 = f
ri,j si0 ,j 0 ai ai0 bj bj 0
(i,j)I

(i0 ,j 0 )J

(i,j)I

(i,j)I,(i0 ,j 0 )J

ri,j si0 ,j 0 ai ai0 bj bj 0

(i,j)I,(i0 ,j 0 )J

X
(i,j)I

ri,j ai bi

si0 ,j 0 ai0 bj 0 ,

(i0 ,j 0 )J


 P
0
0
0
0
which is equal to f
(i0 ,j 0 )J si ,j ai bj . Therefore, f is an
(i,j)I ri,j ai bi f
isomorphism between the rings K1 F K2 and K1 K2 . Since the latter is a field, we
conclude that K1 F K2 is a field, also.
P

118. Prove by an example that in general the tensor product of two field extensions does not need to be an integral domain.
Solution.

Let K1 and K2 both be equal to Q( 1),


and let x and y be the
non-zero
elements in
K1 F K2 defined by x = 1 1 + 1 1 and y = 1 1 1 1. In this
case,

xy = (1 1)2 ( 1 1)2
= 1 1 1 1 = 0.

Definition 0.40. Let F be a field and f (x) F [x] be a polynomial over F . The
splitting field of f (x) is the smallest field extension K of F such that f (x) splits
completely into linear factors in K[x] but not over any proper subfield F
L
K.
A splitting field of f (x) is easy to construct in inductive manner by using roots of
the irreducible factors of f (x), and furthermore, ant two splitting field of f (x) are
isomorphic to each other.
An algebraic extension K of F is called a splitting field over F if K is the splitting
field of a collection of polynomials over F .
58

119. Determine the splitting field and its degree over Q of the polynomial f (x) =
x4 + x2 + 1.
Solution.
By using Problem 106 we see that f (x) does not have a root in Q. However, this
does not mean it is irreducible over Q. It could be equal to a product of two degree 2
polynomials. Indeed, let 6 denote the primitive 6th root of unity, exp(2i/6). Since
f (x) = x4 + x2 + 1 = (x2 )2 + (x2 ) + 1
(x2 )3 1
=
x2 1
x6 1
= 2
x 1
(x3 1)(x3 + 1)
=
(x 1)(x + 1)
x3 1 x3 + 1

=
x1 x+1
= (x2 + x + 1)(x2 x + 1).

(7)

(8)

we see that all 6th root of unities, except 1 and 63 (which makes the denominator of
eqn 7 0) are roots for f (x). Therefore, f (x) = (x2 + x + 1)(x2 x + 1) = ((x 62 )(x
64 ))((x 6 )(x 65 )). Clearly Q(6 ) contains all roots of f (x), and furthermore, it is of
degree 2 over Q. To see this, it is enough to observe that 6 is the root of the irreducible
polynomial x2 x + 1. Thus Q(6 ) is the smallest field in which f (x) = x4 + x2 + 1
splits into linear factors.

Definition 0.41. An algebraically closed field is a field L such that every polynomial in L[x] has a root in L. An algebraic closure of a field F , denoted by F is an
algebraic extension of F that is algebraically closed.
120. Let K be an algebraic extension of F . Suppose we have an injective field
homomorphism (embedding) : F , F . Then there exists an injective
e
homomorphism e : E F such that (x)
= (x) for all x F .
Solution.
Follows immediately from Zorns lemma by considering the poset of all pairs (E 0 , ),
where E 0 is an algebraic extension of F and : E 0 F is an injective homomorphism.
The partial ordering is given by (E 0 , ) (E 00 , ) if E 0 E 00 and |E 0 = .

59

121. Let K be an algebraic extension of F . Suppose K is contained in an algebraic


closure F of F . Prove that K is a splitting field over F if and only if
any injective field homomorphism : K , F that is constant on F is an
automorphism of K.
Solution.
() Let {fi (x)}iI be a collection of polynomials over F for which K is a splitting
field over F and let K be a root of fi (x) for some i I. Without loss of generality
we assume that fi (x) is irreducible over F . On one hand if g(x) denotes the image
of fi (x) under (applied to coefficients), then () is a root of g(x). On the other
hand (fi (x)) = g(x). Therefore, the subfield F () of K is mapped injectively into
itself in F . Since F () is a finite extension (finite dimensional vector space) of F ,
: F () F () is a surjection. In particular it is an isomorphism. We repeat
this process for all roots , , , . . . of all fi (x), i I to conclude that restricts to
isomorphisms on all F (), F (), F (), . . . , hence it is an isomorphism of F (, , , . . . )
onto itself. Since F (, , , . . . ) is the splitting field K, we are done.
() Let K be an element and let p(x) denote its irreducible polynomial over F . If
there is any other root of p(x), it lies in F . Let 0 denote the field map F () F ()
defined by 7 and c 7 c for all c F . By Problem 120 we know that 0 extends
to an injection : K , F , hence, by our assumption to an automorphism of K. If
follows that is an element of K. Repeating this process for all other roots we see
that p(x) splits completely in K. In particular we see that K is the splitting field of
the set of all such polynomials over F .

122. Let K be an algebraic extension of F . Prove that K is a splitting field over


F if and only if every irreducible polynomial over F that has a root in K
splits completely in K[x].
Solution.
() Since K is algebraic over F it is contained in an algebraic closure of F . Let
f (x) F [x] be an irreducible polynomial and suppose K of f (x). Let F be
another root of f (x) and define : F () F () by () = and (c) = c for all
c F . By Problems 120 and 121, extends to an automorphism of K. Therefore,
K, hence f (x) splits in K[x].
() Conversely, for K let p (x) denote the irreducible polynomial over F of .
Then p (x) splits completely in K[x]. Hence K is the splitting field of {p (x)}K .

123. Let F K be a field extension and let K1 and K2 be two intermediate fields
which are finite extensions of F . Suppose both of K1 and K2 are splitting
60

fields of some polynomials over F . Show that both of K1 K2 and K1 K2 are


splitting fields over F .
Solution.
Let : K1 K2 , F denote an injective morphism, constant on F , into the algebraic
closure of K. Since K1 and K2 are splitting fields over F and since the restrictions
K1 and K2 are injective, by Problem 121 these restrictions are automorphisms of K1
and K2 . Let x1 y1 + + xn yn , xi K1 , yi K2 denote an element of K1 K2 . Then,
(x1 y1 + + xn yn ) = (x1 )(y1 ) + + (xn )(yn ) lies in K1 K2 also. Therefore,
is an automorphism of K1 K2 , hence, K1 K2 is a splitting field by Problem 121.
To prove that K1 K2 is a splitting field over F we use Problem 122: let K1 K2
be a root of an irreducible polynomial p(x) over F . Since both of the fields K1 and
K2 are splitting fields, p(x) splits completely over them, hence it splits completely over
K1 K2 . In particular, K1 K2 is a splitting field.

124. Let d and n be two positive integers. Prove that xd 1 divides xn 1 if and
only if d divides n.
Solution.
() Suppose n = dk for some k N. If d is the primitive dth root of unity, then
dn = (d )dk = 1k = 1, hence every root of xd 1 is a root of xn 1 proving that xd 1
divides xn 1.
() Conversely, suppose that xd 1 divides xn 1 and suppose r {0, 1, . . . , d 1}.
We claim that xr 1 is not divisibly by xd 1. Indeed, for every m = 1, . . . , d the mth
power of d would be a root of xr 1. In particular d and dr+1 would be roots. But
then 0 = (d )r+1 1 = (d )r d 1 = d 1, which is absurd, hence xd 1 - xr 1 for
any 0 r < d. Next we assume that d does not divide n, hence by division we have
n = dq+r for some q, r N and 0 < r < d. Since xd 1 divides xqd+r xr = xr (xqd 1),
by the first part of our solution we obtain a contradiction from the equation
xn 1 = xqd+r 1 = (xqd+r xr ) + (xr 1).

125. Improve the previous problem by showing that for any fixed integer a > 1,
d|n (ad 1)|(an 1).
Solution.
Let us validate our hint: by Problem 124, we know that d|n (xd 1)|(xn 1), in
particular ad 1 divides an 1 by evaluation. Conversely, assume that ad 1 divides
an 1 but d - n, that there exists 0 < r < d such that n = qd + r for some nonnegative integer q. By the same trick we used in the proof of Problem 124 we write:
61

an 1 = aqd+r 1 = (aqd 1)ar (ar 1). Equivalently, (an 1)+((ad )q 1)ar = ar 1.


Since (ad )q 1 = (ad 1)((ad )q1 +(ad )q2 + +(ad )1 +1) the left hand side is divisible
by ad 1 but (ar 1)/(ad 1) is not an integer because 0 < r < d. This contradiction
show shat d|n.

126. Let Fpd denote the finite field with pd elements. Show that if Fpd Fpn , then
d divides n.
Solution.
Recall that Fq the group of non-zero elements of the finite field with q elements is
isomorphic to the cyclic group of order q 1. On the other hand, if Z1 and Z2
are two cyclic groups then Z1 , Z2 if and only if |Z1 | divides |Z2 |. Therefore, the
Fpd Fpn , then Fpd is a subgroup of Fpn hence pd 1|pn 1. Now the result follows
from Problem 125.

127. Let F be a field of characteristic p. Show that for any polynomial f (x) F[x]
we have f p (x) = f (xp ).

 p1
p
Solution. It suffices to observe (a + b)p = ap + p1 ap1 b + + p1
ab
+ bp which
p
p
is equal to a + b in a characteristic p field.

Fact 0.42. Eisensteins Criterion: Let P be a prime ideal in an integral domain R.


Suppose that a monic polynomial f (x) = xn +an1 xn1 + a1 x+a0 from R[x] satisfies
the following properties:
each coefficient ai belongs to P ;
constant coefficient a0 does not belong to the square of P , that is a0
/ P 2.
Then f (x) is irreducible over R (hence over K, the fraction field of R).
128. Let p and ` be two prime numbers, and let [`]x denote the `th cyclotomic
polynomial 1 + x + + x`1 .
(a) Show that [`]x is an irreducible element of the polynomial ring Q[x].
(b) Show that [`]x is divisible by x 1 in the polynomial ring Fp [x] if p = `.
Here Fp is the finite field Fp = Z/pZ.
(c) Suppose now that p is different than `, and let a be the order of p in
F` , that is to say, a is the smallest positive integer such that pa = 1 in
F` . Show that a is the first value of m for which the group of invertible
62

m m matrices with entries from Fp contains an element of order `.


Hint: Use the formula for number of elements in GLm (Fp ).
(d) Show that [`]x is not divisible by any polynomial of degree smaller than
a in Fp [x]. Here, a is as in the previous part. Hint: Consider the
companion matrix of any such divisor and use the previous part.
(e) Let A denote a matrix of order ` from GLm (Fp ). Assume that m is
smallest possible number a for which such A exists (see part (c)). If
mA (x) Fp [x] is the minimal polynomial of A, then show that mA (x) is
irreducible of degree a, and it divides [`]x in Fp [x].
(f ) Prove that [`]x is irreducible in Fp [x] if and only if ` 1 is the smallest
power of p which is congruent to 1 modulo ` (in other words, a = ` 1
is the smallest positive integer such that pa 1 = 0 in F` ).
Solution.
(a) First we make an easy observation: a polynomial f (x) is irreducible over Q if and
only if f (x + 1) is irreducible over Q. Thus, it suffices to prove that
 `1

`
x` + `1
x + + 1` x1
(x + 1)` 1
=
[`]x+1 =
(x + 1) 1
x


 
 
`
`
`
`1
`2
=x +
x + +
x+
`1
2
1
`(` 1)
x+`
= x`1 + `x`2 + +
2
is irreducible. This follows from Eisensteins criterion.
(b) Suppose p = `. If p = 2 then [2]x = 1 + x which is equal to 1 x in F2 [x]. If p > 2,
then
(x 1)p
xp 1
=
= (x 1)p1 ,
[p]x =
x1
x1
hence [p]x is divisible by x 1 in Fp [x].
(c) Let us compute the number of elements in GLm (Fq ), where q is a prime power: Let
e1 , . . . , em denote the standard basis for Fm
q . If A GLm (Fq ), then Ae1 , . . . , Aem is a
m
basis for Fq . Conversely, any basis for the vector space Fm
q put together as the columns
of a matrix gives an invertible m m matrix over Fq . So, we count the number of bases
m
for Fm
q . The first element of a basis B = {f1 , . . . , fm } is any non-zero vector from Fq so
it can be chosen in q m 1 different ways. The second element f2 of B is any vector that is
not a scalar multiple of f1 , hence it can be chosen in q m q different ways. f3 is a vector
that is not in the span of f1 and f2 . In other words, f3 Fm
q {af1 + bf2 : a, b Fq }.
m
3
There are q q choices for f3 . Continuing
Pi1 in this manner we see that fi is one of
the q m q i possible vectors from Fm

{
q
j=0 ai fi : ai Fq }. Therefore;
|GLm (Fq )| = (q m 1)(q m q) (q m q m1 ).
63

(9)

Going back to the original problem, we are going to show that the order a of p in F`
is the smallest m such that GLm (Fq ) contains an element of order `. The necessary
m Q
i
condition for this to happen is that ` divides |GLm (Fp )| = p( 2 ) m
i=1 (p 1). Here,
m Q
i
since ` 6= p, we see that the first value of m that ` divides p( 2 ) m
i=1 (p 1) is when
m
` divides the highest term p 1 of the product for the first time. Obviously, this
happens exactly when pa 1 = 0 mod `.
(d) Suppose [`]x is divisible by a polynomial f (x) of degree s. Without loss of generality
we assume that f (x) has a non-zero constant term, hence its companion matrix A is
invertible (it belongs to GLs (Fp )). Since the characteristic polynomial of a companion
matrix A of a polynomial f (x) is the polynomial f (x) itself, we see that f (A) = 0.
Therefore, the value of the polynomial [`]x = (x` 1)/(x 1) on A is 0. It follows that
A` = id. In other words, A GLs (Fp ) is an element of order `. By Part (c) we see
that this is possible only if s a.
(e) Let A GLa (Fp ) be a matrix of order `, where a is smallest possible index for
which such A exists. If mA (x) is the minimal polynomial of A over Fp , the irreducible
generator of the principal ideal consisting of all polynomials that vanish on A. Since
A is of order `, x` 1 is divisible by mA (x). It follows that [`]x = (x` 1)/(x 1)
is divisible by mA (x). The degree of the minimal polynomial of a matrix is less than
or equal to the degree of the characteristic polynomial of a matrix. Since the degree
of the characteristic polynomial is the size of the matrix, we see that deg mA (x) a.
But by part (d), we know that a deg mA (x), therefore, we have the equality.
(f) Let A denote the companion matrix of [`]x , and let mA (x) denote the minimal
polynomial of A which divides the characteristic polynomial of A, which is [`]x . Since
mA (x) is irreducible we know that the degree of mA (x) is greater than or equal to a.
Since A satisfies [`]x we know that mA (x) divides [`]x . Therefore, if [`]x is irreducible,
then mA (x) = [`]x and furthermore its degree is ` 1, which is the smallest number a
such that pa = 1 mod `. Conversely, if ` 1 is the smallest number such that p`1 = 1
mod `, then m = ` 1 is the smallest number such that GLm (Fp ) has an element of
order m. In other words, A`1 = id for some A GLm (Fp ). It follows that [`]x vanishes
on A, hence it belongs to the ideal generated by the minimal polynomial of A. By part
(d), we know that the degree of mA (x) is a, hence deg mA (x) = a = ` 1. Since
deg[`]x = ` 1, we see that [`]x = mA (x), hence irreducible.

Definition 0.43. A polynomial f (x) over a field F is called separable if its roots in
F are all distinct from each other. Otherwise f (x) is called inseparable.
Let K be an extension of F and let x K be an element. Multiplication by x is
an F -linear operator on K. We denote its minimal polynomial by px (t) F [t]. If
for all x K the minimal polynomial px (t) is a separable polynomial, then K is
called a separable extension over F . If there exists at least one element x K with
64

inseparable minimal polynomial, then K is called, logically, an inseparable extension


of F . If minimal polynomial of any element of K is inseparable over F , then K is called
purely inseparable.
Fact 0.44. A straightforward to verify observation about having a root with multiplicity is this: a polynomial f (x) has a root with multiplicity 2 if and only if is
a root of f 0 (x), the derivative of f (x). In other words, a polynomial f (x) is separable
if and only if f (x) and f 0 (x) are relatively prime.
129. Let f = f (X) F [X] be an irreducible polynomial. Prove that f is separable
over k if and only if f 0 is non-zero.
Solution.
We already know that if f is separable, then f 0 and f are relatively prime, therefore,
f 0 cannot be zero. We prove the converse by contrapositive, so, assume that f is
not separable. Then f and f 0 have a non-unit common divisor. On one hand f is
irreducible, hence the common divisor of f and f 0 is f . On the other hand, the degree
of f 0 is less than that of f . Therefore, f | f 0 implies that f 0 = 0.

130. For any prime p and nonzero element a Fp , prove that the polynomial
f (x) = xp x + a is irreducible and separable over Fp .
Solution.
Let A(x) denote xp x a. First of all, A(x) is separable because A0 (x) = 1 is
non-zero. Second, A(x) has no root in Fp , since bp b a = a for any b Fp . Notice
that A(x + a) = (x + a)p (x + a) a = xp + ap x a a = xp x a = A(x).
Therefore, by iteration, we see that A(x + ba) = A(x) for all b = 1, 2, . . . . As b runs
over all positive integers, ba runs over {0, . . . , p 1} = Fp . This means that in the
splitting field K of A(x) over Fp , if is a root, then + k, k {0, . . . , p 1} is a
root, also. Since
the degree of A(x) is p, it follows these are all the roots of A(x)
Qp1
that is A(x) = k=0 (x k). If A(x) is not irreducible over Fp then there exist
r(x), s(x) Fp [x] such that A(x) = r(x)s(x) and the 0 < d = deg
Q r(x) < p. On the
other hand, being a divisor of A(x), r(x) has the form r(x) = kJ (x k) for
d1
some subset J {0, . . . , p 1}. The coefficient
of r(x) P
is the negativeP
of the
P of x
sum of all roots of r(x), hence it is equal to iJ + i = |J| + iJ i = d + iJ i.
Since this coefficient lies in Fp , and since the degree d of r(x) is non-zero, we obtain
the contradictory statement that Fp .

131. Show that for any prime p and a positive integer n there exists a finite field
with q = pn elements.
65

Solutions.
We know that Z/p, which we denote by Fp , is a finite field with p elements. Consider
n
n
f (x) = xp x over Fp . Since f 0 (x) = pn xp 1 1 = 1, f (x) is separable, hence it
has pn distinct roots. It is easy to verify that all of these roots form a field K; closed
under multiplication, inverses exist, and closed under addition. Therefore, there exists
a field K that contains F and has pn elements. Note: Fp is contained in K (since 1 is
a root of f (x), and 1 generates Fp ). In fact, K is the splitting field of f (x) over Fp .

132. In this problem we ask for an inverse of Problem 126. If d divides n, then
Fpd is a subfield of Fpn .
Solution.
By Problem 131 we know that both Fpd and Fpn exist. It suffices to show that the
splitting polynomial that defines Fpd is a divisor of Fpn . Indeed, the formers splitting
d
d
n
n
polynomial is xp x = x(xp 1 1) and the latters is xp x = x(xp 1 1). By
d
Problem 125 we know that pd 1 divides pn 1. Then by Problem 124, xp 1 1
n
divides xp 1 1. Therefore, Fpd is a subfield of Fpn .

133. Show that a finite field F does not contain any ring R which is not a field.
Solution. If R F is a subring, then the non-zero elements of R is a submonoid of the
group of invertible elements of F, which is a cyclic group {1, g, . . . , g d }. Any submonoid
of a cyclic group is a subgroup, hence it is cyclic. Therefore, R is a subfield of F.

134. Recall (from Problem 110) that in a finite field F of characteristic p any
element is a pth power. Find a field K of characteristic p which has an
element that is not a pth power.
Solution.
Let X be a variable and consider the rational function field K = Fp (X). Clearly
K is of characteristic p. Let us show that X is not a pth power in K. Assume
otherwise that X = (f (X)/g(X))p , for some polynomials f (X), g(X) and hence X
divides f (X). This leads to a contradiction as follows. Write f (X) = X r f1 (X), where
r 1 and X - f1 (X). Simplifying the polynomial equality g(X)p X = f (X)p , we obtain
g(X)p = X rp1 f1 (X)p . Thus X divides g(X). Write g(X) = X s g1 (X) with X - g1 ,
and s 1. It follows that X sp = X rp1 which is impossible.

66

135. Find an irreducible polynomial f (x) over a field F which is not separable.
Solution.
As we know from Problem 134, a variable X is not a pth power in F = Fp (X), hence
the polynomial f (Y ) defined by f (Y ) = Y p X F [Y ] has no root in F . Let K
denote the splitting field of f (Y ) over F , and let x K be a root. The characteristic
of K is p, therefore, it follows from binomial expansion that (Y x)p K[Y ] is equal
to f (Y ) = Y p X K[Y ]. In other words, the multiplicity of the root x is p, hence,
f (Y ) is not separable.
We claim that f (Y ) is irreducible in F [Y ]. First of all, R = Fp [X] is an integral domain
and the fraction field of R is F = Fp (X). Since R is a UFD, by Problem 115, to show
that f (Y ) is irreducible over F [Y ], it is enough to prove its irreducibility over R[Y ].
Let P R[Y ] denote the prime ideal generated by X in R[Y ] = Fp [X, Y ]. Since X P
but X
/ P 2 = (X 2 ), by Eisensteins criterion (Fact 0.42), f (Y ) is irreducible in R[Y ],
hence in F [Y ] = Fp (X)[Y ].

Definition 0.45. Problem 135 brings us to the notion of a perfect field. A field F
is called perfect if every irreducible polynomial is separable.
136. Let F be a characteristic 0 field. If f (X) F [X] is irreducible, then it is
separable. In other words, F is perfect.
Solution.
If f (X) F [X] is a non-constant irreducible polynomial, then its derivative f 0 is
non-zero in F [X], hence f is separable by Problem 129.

137. Prove that any finite field is perfect.


Solution
Let K be a finite field with |K| = pn , and let f (x) K[x] be an irreducible polynomial.
Assume towards a contradiction that f (x) is not separable, or equivalently that f 0 (x) =
0 (by Problem 129). Then each monomial in f (x) is of the form axn where n is divisible
by p. We analyze f (x) further by writing it explicitly:
f (x) = am xmp + am1 x(m1)p + + a1 xp + a0
= bpm (xm )p + bpm1 (xm1 )p + + b1 xp + bp0 (since every coefficient is a pth power)
= (bm xm + + b0 )p .
But this means f (x) is not irreducible, a contradiction.

67

Fact 0.46. Let M be an n n matrix with entries from a field F . If F contains all
the eigenvalues of M , then M is similar to a diagonal matrix over F if and only if the
minimal polynomial of A has no repeated roots. Here, by a minimal polynomial we
mean a polynomial of the smallest degree p(x) F [x] such that p(A) = 0. (See the
section on RATIONAL CANONICAL FORMS.)
138. Let A GLn (C) be an invertible matrix with entries from C such that Ak = id
for some k 1. Show that A is diagonalizable. Show that the matrix


1
B=
GL2 (Fp )
0 1
satisfies B p = id but it is not diagonalizable.
Solution.
Since A GLn (C) satisfies the polynomial equation xk 1 = 0 the minimal polynomial
of A is a divisor of xk 1, hence it has distinct roots. In particular, by Fact 0.46, A is
diagonalizable.


1
k
For the second part observe that, for all k 1 B k =
, hence B p = id.
0 1
Therefore, the minimal polynomial is a divisor of xp 1. Since xp 1 = (x 1)p
in Fp [x], and since B 6= id, we see that the minimal polynomial has a repeated root,
therefore, by Fact 0.46 B is not diagonalizable.

68

MODULES OVER PRINCIPAL IDEAL DOMAINS


We begin with some warm-up exercises.
Definition 0.47. An element m of a module M over an integral domain is called
torsion if r m = 0 for some r R. The set of all torsion elements of a module M is
denoted by tor(M ). It is straightforward to verify that tor(M ) is an R-submodule of
M (commutativity plays a role in the proof).
The rank of a module M over an integral domain R is the maximal number of R-linearly
independent elements.
139. Let M be a module over an integral domain R. Show that the rank of M is
the same as that of M/tor(M ).
Solution. We begin with showing that the rank of tor(M ) is zero. Indeed, there is no
R-linearly independent set of elements from tor(M ): for any set of elements x1 , . . . , xs
from tor(M ) there exists a nonzero r R such that s(x1 + +xs ) = sx1 + +sxs = 0.
Next, let B = {y1 , . . . , yr } M be a maximally R-linearly independent set of elements
from M . None of the yi s lie in tor(M ) otherwise elements of B satisfy
a1 y1 + + ar yr = 0
with not all ai s are zero, hence B is linearly dependent. Thus, the images of the
elements of B in M/tor(M ) form a maximally R-linearly independent elements.

Definition 0.48. An R-module M is called a torsion R-module, if for a given m M


there exists r R such that rm = 0.
140. Let R be an integral domain and let M be an R-module. (a) Suppose M
is of rank n. Prove that if B = {y1 , . . . , yn } M is a maximally R-linearly
independent subset, then the R-submodule N of M that is generated by B
is a free R-module of rank n, and furthermore M/N is a torsion R-module.
(b) Prove the converse: if N M is a free R-submodule of rank n and if
M/N is a torsion R-module, then the rank of M is equal to the rank of N .
Solution.
(a) Since B is an R-linearly independent subset it is clear that N
= Ry1 Ryn . In
other words, N is free of rank n. Let m M be a non-zero element. The set {m} B
is linearly dependent by the maximality assumption on B. It follows that there exists
not all zero elements r0 , r1 , . . . , rm from R such that r0 m+r1 y1 + +rn yn = 0. Hence,
r0 m N , or equivalently, r0 m = 0 in M/N . Since m is arbitrary, M/N is a torsion
R-module.

69

(b) Since M/N is torsion, for an element m M , there exists r R such that
rm N . Since N is free of rank n, it has a basis B = {y1 , . . . , yn } N . Thus
rm = r1 y1 + + rn yn for some r1 , . . . , rn R. It follows that we cannot add any
element of M to B and keep the R-linear independence. Therefore, B is a maximally
R-linearly independent subset of M , hence the rank of M is n, also.

141. Let R be an integral domain and N M be two R-modules. If the ranks of


M , N , and of M/N are given by, respectively, n, r and s, then prove that
m = r + s.
Solution.
Let x1 , . . . , xs be a list of elements from M such that their images x1 , . . . , xs in M/N is
a maximally R-linearly independent. If there is an R-linear relation among xi s, then
it carries into M/N , hence their images would not be linearly independent. Therefore,
x1 , . . . , xs are linearly independent in M . Let {y1 , . . . , yr } be a maximally R-linearly
independent subset of N . We claim that {x1 , . . . , xs , y1 , . . . , yr } is maximally R-linearly
independent
P
P in M . Assume otherwise that there exist a1 , . . . , as , b1 , . . . , br such that
ai xi + bj yj = 0. On the one hand we know that not all of ai s are simultaneously
zero, and similarly, not
P all bj s are simultaneously zero. On the other hand, reducing
mod N we see that
ai xi = 0 in M/N . By our initial assumption on xi s we then
have ai = 0, a contradiction. Therefore, {x1 , . . . , xs , y1 , . . . , yr } is maximally R-linearly
independent set and the rank of M is s + r.

142. Here is an example of an R-module of rank 1 which is not free: Let R denote
the polynomial ring Z[x] and M R denote the ideal generated by 2 and x.
Show that M is an R-module of rank 1 but it is not free. More generally,
if R is any integral domain and M R a non-principal ideal, then M is a
torsion-free R-module of rank 1 but not a free R-module.
Solution. Every ideal in R has a natural structure of an R-module. Let y1 , . . . , yn be
a maximal set of R-linearly independent elements from M . Since y1 , y2 belongs to R,
we define a1 = y2 , a2 = y1 , a3 = = an = 0. Then a1 y1 + a2 y2 + + an yn = 0 even
though not all of ai s are zero. We conclude that a maximal set of linearly independent
elements in M has only 1 element. In other words, M is of rank 1. Next we prove
that M is not a free R-module. Let N M be the subideal generated by 2. Since any
element of N is of the form 2f (x) for f (x) R, N is a free R-module of rank 1. Notice
that M/N is the Z[x]-module consisting of polynomials with odd integer coefficients,
hence it is non-zero, proving that M is not free.
This argument generalizes verbatim to the setting of an arbitrary integral domain and
a non-principal ideal.
70

143. Let R be an integral domain. Find an example of a torsion R-module M


such that the annihilator of M is 0.
L
i
Solution. The Z[x]-module M =
i=1 Z[x]/(x ) is a torsion module: If f = (f1 , f2 , . . . )
is an element of M , then only finitely many fi s are non-zero (meaning that fi
/ (xi )).
If j is sufficiently large positive integer, then xr f = (xr f1 , xr f2 , . . . ) = (0, 0, . . . ). On
the other hand, for any polynomial g(x) Z[x] of degree d, the element g(x)f M ,
where f = (f1 , f2 , . . . ) with fd+1 = 1, is non-zero. Hence, the annihilator of M is {0}.

144. Let R be an integral domain. Prove that if N is a finitely generated torsion


R-module, then the annihilator of N is nonzero.
Solution.
Let {y1 , . . . , yk } be a generating set for N . Since N is a torsion module, for each
i {1, . . . , k} there exists ri such that ri yi = 0. Then the nonzero element r = r1 rk
lies in the annihilator of N .

Definition 0.49. Let R be a PID, M a torsion R-module, p R a prime element.


(Recall that an element p is called prime if and only if the principal ideal generated
by p is a prime ideal. In a UFD an element is principal if and only if it is irreducible.)
The p-primary component of M is the set of all elements of M that are annihilated
by some positive power of p.
145. Let R, M and p be as in the definition.
(a) Prove that p-primary component of M is a submodule.
(b) Prove that M is the direct sum of its p-primary components as p runs
over all prime elements.
Solution.
(a) Let m1 and m2 be two elements from p-primary component of M and let r R be
an arbitrary element. This means pu m1 = pv m2 = 0 for some positive integers u and v.
Since pu+v (m1 + rm2 ) = pu+v m1 + rpu+v m2 = 0, the p-primary subset is a submodule.
(b) Given a prime element p M let us denote by M (p) the p-primary component.
We claim that M (p) M (q) = 0, where M (p) is the p-primary component of M
(M (q) is defined similarly). Assume otherwise that there exists a nonzero element
m M (p) M (q). Then there exist positive powers pa and q b that belong to the
annihilating ideal AnnR (m) of m in R. Since R is a PID, pa and q b are relatively
prime, the ideal is the whole ring, which is absurd (1 R cannot annihilate any
non-zero element).
71

Next, we show that any element m M is of the form m = m1 + + mk for some


mi M (pi ), where p1 , . . . , pk are distinct prime elements. Since M is a torsion module,
there exist r R such that rm = 0. Suppose that r = upa11 pakk is the irreducible
decomposition of r. It is clear that the list r1 , . . . , rk , where ri = r/pai i (1 i k)
consists of relatively prime elements. Therefore, there exists f1 , . . . , fk R such that
P
fi ri = 1. Denoting by mi the element fi ri m, we have the following decomposition:
m = m1 + + mk . Note that mi is annihilated by pai i , therefore, m ki=1 M (pi ) and
the proof is complete.

146. Let R be a PID and M be a free R-module. Show that any R-submodule N
of M is free, also.
Solution.
Note that we do not assume M is finitely generated. Let {ei } be a basis for M which
we assume to be totally ordered (by Axiom of P
Choice). Let pi : M R denote
the ith projection homomorphism defined by pi ( aj ej ) = ai . Clearly, the element
fi = ri ei M is in N Rei . We claim that {ri ei }Pis a basis for N . To this end,
let n N be an arbitrary element. Clearly, n =
si ei , where si = pi (n). Since
pi (n) P
(ri ) we see that si is a multiple of ri , say si = i ri for some i R. Thus,
n =
i ri ei showing that n lies in the span of {ri ei }. Since the set {ei } is Rlinearly independent, the set of scalar multiples {ri ei } is R-linearly independent, also.
Therefore, it is a basis for N .
Remark 0.50. Recall that one of the several equivalent definitions of a projective
module is that a module M is a projective R-module if it is a direct summand of a
free R-module. Problem 146 shows that over a PID a finitely generated R-module is
projective if and only if it is free.

147. Let R be a PID and let M be a finitely generated R-module. Prove that
M
= Rm tor(M ) for some (unique) m N.
Solution.
P
Let {ui }ni=1 be a generating set for M . Define : Rn M by ( ai ei ) = ai ui , where
e1 , . . . , en is the standard basis for the free R-module Rn = R R. It is clear
that is a surjective R-module homomorphism. Therefore, Rn / ker
= M . Since
n
ker is a submodule of the free module R , we know from Problem 146 that it is free
and has a basis of the form {ri ei }iI for some elements ri R, and I {1, . . . , n}.
We split this basis into two subsets: A1 = {ri ei : ri is a unit in R} and A2 = {ri ei :
ri is a non-unit}. Let m1 and m2 denote the cardinalities of A1 and A3 , respectively.

72

Thus, ker = Rm1

L L

ri ei A2


Rri ei Rn , and therefore
!

M
= R / ker
= (R
n

nm1

R/Rri

(10)

ri ei A2

L
It is clear that the second component ri ei A2 R/Rri gives the torsion submodule of
M . It is also clear from this argument that m = n m1 is equal to the rank of M ,
hence it is unique.

Definition 0.51. We know from Problem 147, specifically from eqn. (10) that any
finitely generated module M over a PID R is isomorphic to Rs R/(r1 ) R/(rl )
for some nonnegative integer s and some non-invertible elements r1 , . . . , rl from R. By
using Chinese Remainder Theorem we have a further decomposition of each factor:
R/(ri )
= R/(pa11 ) R/(pamm ),
where ri = pa11 pamm is the decomposition of ri into irreducible factors in R. An
elementary divisor of M is one of the prime powers pai i appearing in any of the
summands R/(ri ).
148. Let R be a PID and M be a torsion R-module. Prove that the decomposition
into elementary divisors of M can be re-organized so that we have
M
= R/(s1 ) R/(sk ),
for some (unique) elements si R such that si | si+1 for i = 1, . . . , k 1.
Solution.
Since R is a PID, it is a UDF. Therefore, each ri is a product of irreducible elements:
ri,d
r
r
ri = pi,1i,1 pi,2i,2 pi,dii for some positive exponents. By Chinese Remainder Theorem:


ri,d 
r
r
R/(ri )
= R/(pi,1i,1 ) R/(pi,2i,2 ) R/(pi,dii ) .
Next, we prepare a right justified array of the powers of the irreducible factors appearing
in any of the summands R/(ri ). Along each row we place the powers of pi (that appear
in any of R/(ri )) ordered so that the exponents increase from left to right. Obviously,
there are rows of various lengths. If the length of the longest row is k, then we complete
our array into an l k array by placing a 1 in each available entry. For example, for
M
= R/(q 4 t8 ) R/(p3 t) R/(p2 t2 ) R/(p3 t8 ) R/(t6 ), the list of prime powers is
2 3 3 4
p , p , p , q , t, t2 , t6 , t8 , t8 , hence, our array is:
1 1 p 2 p3 p3
1 1 1 1 q4
t t2 t6 t8 t8
73

Next, we multiply the entries along each column of the array and get k elements
s1 , . . . , sk with the property that si | si+1 for i = 1, . . . , k 1. Since their components
are rearrangement of each other, there is an isomorphism between the following direct
sums:
M
=

k
M

R/(si )
=

i=1

l
M

R/(ri ).

(11)

i=1

The uniqueness of the factors si are clear from the uniqueness of the array we constructed above for proving (11).

149. Let a be a nonzero element of a principal ideal domain R. If M = R/(a) and


p R is a prime element, then prove that
(
R/(p) if k n
pk1 M/pk M
=
0
if k > n
where n is the power of p dividing a.
Solution.
Let a = pn q be the factorization of a with p and q are relatively prime. There exists f, g R such that f p + gq = 1. Thus, for any x M = R/(a) we have the
decomposition x = f px + gqx.
If k > n, then k 1 n, hence pk1 x = f pk x + gpk1 qx = pk f x. In other words,
pk1 x = 0 modulo pk M .
If k n, then pk1 x = f pk x + gpk1 qx, hence pk1 x = pk1 gqx mod pk M . Define
: pk1 M/pk M R/(p) by (pk1 x mod pk M ) = x0 mod (p), where x0 R is
any representative of x R/(a) in R. Lets check that is well-defined: if pk1 x1
mod pk M = pk1 x2 mod pk M , then pk1 (x1 x2 ) pk M , hence x1 x2 is divisible
by p. It follows that x01 x02 = 0 in R/(p). Now we know that is well-defined we
prove that it is an isomorphism. The kernel of is clearly those pk1 x pk1 M with
any representative x0 of x is divisible by p, hence, x pk M . To prove the subjectivity,
let u R/(p) be a nonzero element. Hence p and u0 (a representative of u in R) are
relatively prime. Then pk1 u0 is not divisible by pk , and furthermore pk1 u0 is not zero
in M = R/(a) = R/(pn q) (since k 1 < n). Thus, pk1 u0 represents a nonzero element
in pk1 M/pk M which is mapped to u mod (p). It is clear that is an R-module map,
therefore, it is an isomorphism.

150. Let R be a PID, p R be a prime element. Prove that if M is a finitely


generated torsion R-module, then show that pk1 M/pk M
= F nk , where F is
74

the field F = R/(p) and nk is the number of elementary divisors ps of M such


that k s.
Solution.
Since M is a finitely generated torsion R-module, we know of various decompositions
of M as in eqn. (11). Write, for example,
M
= R/(r1 ) R/(rl )
for some ri R. Let us denote factor R/(ri ) by Mi so that M
= M1 Ml . Since
the sum is direct, we have
p

k1

M/p M
=
k

l
M

pk1 Mi /pk Mi .

i=1
k1

By Problem 149 we know that p Mi /p Mi is isomorphic to R/(p) for all k n(i),


where n(i) is the power of p that divides ri . In other words, pk1 Mi /pk Mi
= R/(p)

k1
k
if the elementary divisor p of Mi satisfy k . Thus, p M/p M has one copy of
R/(p) for each elementary divisor ps of M with s k.
Remark 0.52. In particular, if M1 and M2 are two isomorphic finitely generated
torsion R-modules, then for every k 0 M1 and M2 has the same the same set of
elementary divisors.

151. Let R be an integral domain with quotient field K and let M be an R-module.
Prove that the rank of M is equal to the dimension of the K-vector space
K R M .
Solution. Let {ui }iI be an R-linearly independent maximal set. We claim that {ui
1}iI is a linear independent set in K R M . Assume otherwise that there is a relation
among its elements: a1 (u1 1)+ +ar (ur 1) = 0 for some scalers ai K. Multiplying
by a common multiple of the denominators of ai s we P
assume that ai lie in R. On the
one hand, since tensor products are linear, we have ( ri ai ui ) 1 = 0. On the other
hand,
Pr since we work with an integral domain, this primitive tensor is 0 if and only if
i ai ui = 0, hence ai = 0 for all i = 1, . . . , r. Hence rankR M dimK K R M .
Conversely, if B = {ai /bi ui }iI is a vector space basis
Q for K R M , then for any
finite subset {a1 /b1 u1 , , an /bn un }, let b =
bi . We claim that the set
{ba1 /b1 u1 , . . . , ban /bn un } is an R-linearly independent
P set. Indeed, otherwise, there
exists not all zero elements c1 , . . . , cn R such that
ci bai /bi ui = 0. Then
bc1 (a1 /b1 u1 ) + + bcn (an /bn un ) = ((bc1 a1 /b1 ) u1 ) + + ((bcn an /bn ) un )
= (1 (bc1 a1 /b1 )u1 ) + + (1 (bcn an /bn un ))
= 1 (bc1 a1 /b1 u1 + + bcn an /bn un ))
= 1 0 = 0.
75

This contradiction shows that dimF F R M rankR M .

76

CANONICAL FORMS OF LINEAR OPERATORS


Let V be a finite dimensional vector space over a field F and T : V V be an F -linear
map. This data is equivalent toPgiving an F [x]-module structure on V via the action
p(x) v = p(T )v, where p(x) =
ai xi F [x] and so p(T )v = an T n v + + a0 v V .
Since V is a finitely generated F -module, it is finitely generated over F [x], as well.
Also, since any free F [x]-module is an infinite dimensional vector space over F , we see
that V is a finitely generated torsion F [x]-module. Therefore, by Problem 148 we have
V
= F [x]/(a1 (x)) F [x]/(a2 (x)) F [x]/(ak (x))

(12)

for some polynomials ai (x) F [x] such that ai (x) | ai+1 (x) for i = 1, . . . , k 1. Identifying the linear action of T on V by the multiplication by x on the right hand side of
eqn. 12, we represent T in an another matrix form. Since the images of 1, x, . . . , xdi 1
in F [x]/(ai (x)) is a vector space basis, where di = deg ai (x), we see that multiplication by x on this particular component has a pleasant matrix form, namely, by the
companion matrix of ai (x) = xdi + bdi 1 xdi 1 + b1 x + b0 :

0 0 0 0 0 b0
1 0 0 0 0 b1

0 1 0 0 0 b2

Ci = 0 0 1 0 0 b3
.. .. .. . . .. ..
..
. . .
.
.
.
.

0 0 0 1 0 bdi 2
0 0 0 0 1 bdi 1
The block diagonal matrix with diagonal blocks C1 , . . . , Ck , called the rational canonical form of T and denoted by rat(T ), determines the similarity class of T . Note that
the characteristic polynomial of the operator T is the product of the characteristic
polynomials of Ci s, which, in turn, are equal to ai (x)s. The minimal polynomial of
T is the largest invariant factor ak (x). Therefore, some power of the minimal polynomial is divisible by the characteristic polynomial, and conversely, the characteristic
polynomial is divisible by the minimal polynomial. In particular, this gives us various
useful information: minimal polynomial and characteristic polynomials have the same
roots.

152. Recall from Problem 110 that the Frobenius map : Fpn Fpn defined by
(a) = ap is an Fp -linear automorphism of Fpn . Prove that n is the identity
transformation on Fpn and no smaller power of is identity.
2

Solution. Since (a) = ap , a Fp we see that 2 (a) = (ap ) = ap , and by induction


i
n
i (a) = ap for all i = 1, . . . , n. Since Fpn is the splitting field of the polynomial xp x
n
over Fp , we see that n (a) = ap = a. To prove that no smaller power of gives
77

the identity transformation we need to show that for all i = 1, . . . , n 1, there exists
i
a Fpn such that ap a 6= 0. Assume otherwise that there exists 1 i < n such that
i
i
ap a = 0 for all a Fpn . But this is impossible because the polynomial xp x is of
degree pi and Fpn has pn elements.

153. Determine the rational canonical form of the Frobenius map : Fpn Fpn .
Solution.
By Problem 152 we know that n = id and for no i {1, . . . , n1} i = id. Therefore,
the minimal polynomial of is xn 1. In particular, its degree is n. Since Fpn is an
n-dimensional vector space over Fp , we see that the rational canonical form of is the
companion matrix of the minimal polynomial, which given by

0 0 0 0 0 1
1 0 0 0 0 0

0 1 0 0 0 0

0 0 1 0 0 0
rat() =
.
.. .. .. . . .. .. ..
. . .
. . . .

0 0 0 1 0 0
0 0 0 0 1 0

154. Determine the Jordan canonical form of the Frobenius map : Fpn Fpn
considered as an Fp -linear automorphism. Here we assume that Fp contains
all eigenvalues of .
Solution.
We know that the minimal polynomial and the characteristic polynomial of both are
the same and equal to xn 1. Since we assume that all roots of xn 1 lie in Fp , it
decomposes into linear factors as in
xn 1 = (x 1 )s1 (x k )sk
for some positive integers si such that s1 + + sk = n, and i Fp s are its distinct
roots. Thus, the Jordan canonical form of over Fp is of the form

J1 0 0 0
0 J2 0 0

0 0 J3 0

,
.. .. .. . .
..
. . .
. .
0 0 0 Jk
78

where Ji is the si si Jordan block matrix having i on its diagonal. We further


analyze the situation with respect to divisibility of n by p:
If p does not divide n, then xn 1 is separable (the derivative of xn 1 is non-zero in
Fp ), hence the Jordan canonical form is an n n diagonal matrix with distinct entries
as the roots of xn 1 in Fp .
e

If p divides n, say n = pe m (with e Z+ and p - m), then xn 1 = xp m 1 = (xm 1)p .


Therefore, the set of roots of xn 1 is precisely the set of roots of xm 1. In this case,
there are m distinct Jordan blocks J1 , . . . , Jm , each having the same size pe pe with
an mth root of unity along its diagonal.

Fact 0.53. Let A be an n n matrix with entries from a field F and assume that
F contains all eigenvalues of A. In this case, A is diagonalizable if and only if the
minimal polynomial of A has no repeated roots.
155. Determine the characteristic polynomial of the Frobenius map : Fpn Fpn
and prove that is diagonalizable over Fp if and only if n divides p 1, and
is diagonalizable over the algebraic closure of Fp if and only if gcd(n, p) = 1.
Solution.
We repeat the determination of the characteristic polynomial: Fpn is n-dimensional
over Fp . By Problem 152 we know that n = id and for no i {1, . . . , n 1} i = id.
Therefore, the minimal polynomial of is xn 1, hence it is equal to the characteristic
polynomial.
The operator is diagonalizable if and only if itsQminimal polynomial xn 1 factorizes
into distinct linear factors in Fp [x]. If xn 1 = S (x ) in Fp [x], then S forms a
subgroup of invertible elements of Fp . Therefore, n = |S| divides p 1 = |Fp |. Over
the algebraic closure of Fp , xn 1 always splits into linear factors. The only condition
that remains to be checked is if all roots are non-repeated, or if xn 1 is separable.
This is the case if n 6= 0 in Fp , or if gcd(n, p) = 1.

79

AUTOMORPHISMS OF EXTENSIONS
Notation. Given a ring extension A B, we denote by AutA (B) the group of all ring
isomorphisms : B B such that (a) = a. (Equivalently, AutA (B) is the group of
all A-algebra automorphisms of B.
156. Determine AutF (F [x]), where F is a field, x is a variable.
Solution. Since : F [x] F [x] is constant on F , it is uniquely determined by (x).
It is clear that x has to be mapped to an element of the form ax + b (it cannot be
mapped to a scalar, neither it could be mapped to a power xk with k 2) with a 6= 0.
Therefore,
G = AutF (F [x])
= {(a, b) F F : a 6= 0} = F F,
where the group structure on the last space is given by (a, b) (c, d) = (ac, ad + b).

157. Determine AutQ (R).


Solution.
If x R is a positive real number, then x = y 2 for some y R. Thus, (x) = (y 2 ) =
((y))2 , hence it is a positive number. If x R is a negative number, then x = y 2
for some y R, hence (x) = ((y))2 , hence it is a negative number. Let x, y R
be two real numbers such that x < y. Since y x is positive, (y x) = (y) (x)
is positive also. Therefore, x < y implies that (x) < (y) for all x, y R.
Next we prove that is a continuous map. To this end, it is enough to prove that an
infinitely small neighborhood of a point x is mapped to an infinitely small neighborhood
of the image (x). Equivalently, we prove that 1/m < a b < 1/m = 1/m <
(a) (b) < 1/m for every positive integer m and for any a, b R. Let x denote
a b. Since is order preserving we have (1/m) < (x) < (1/m), or equivalently,
1/m < (x) = (a) (b) < 1/m.
Now that we know is a continuous map, the function f : R R defined by f (x) =
x (x) is continuous also. If f (y) 6= 0 for some real number y R, then by continuity
there exists a rational number x near y such that f (x) 6= 0. This contradiction shows
that there are no non-trivial (6= id) continuous functions on R that fixes Q.

Notation. Occasionally we use the abbreviated statement f is an A-automorphism of


B to mean that f is an A-algebra automorphism of B.
158. Prove that the F -automorphisms of the field F (x) = {f (x)/g(x) : f (x), g(x)
F [x], g(x) 6= 0} are precisely the fractional linear transformations. In other
words, AutF F (x) if and only if there exits a, b, c, d F such that bcad 6= 0
for all f (x) F (x).
and (f (x)) = f bx+a
dx+c
80

Solution.
Let : F (x) F (x) be an F -automorphism. Since is a field isomorphism, it
is enough to determine its action on the element x F (x). Let us define the degree of an element f (x)/g(x) F (x) to be max{deg f (x), deg g(x)} and denote it by
deg(f (x)/g(x)). Here, we assume that f (x) and g(x) are relatively prime. We claim
that deg((x)) = 1. Assume otherwise that the degree m of (x) is different than
1. First of all, m cannot be 0, otherwise, (x) F hence (F (x)) = F . Suppose
(x) = f (x)/g(x), where f (x) and g(x) are two relatively prime polynomials with
g(x) 6= 0, r = deg f (x) and s = deg g(x). Assume that max(r, s) 2.
P
P
j
Let u(x) = ni=0 ai xi and v(x) = m
j=0 bj x be two relatively prime polynomials such
that
Pn


i
u((x))
u(x)
i=0 ai ((x))
=
= Pm
x=
j
v(x)
v((x))
j=0 bi ((x))
Pn

ai f (x)i g(x)ni (x)


g(x)n
Pm
j g(x)mj (x)
b
f
(x)
j=0 j
g(x)m
P
g(x)m ( ni=0 ai f (x)i g(x)ni )
,
P
m
j g(x)mj
b
f
(x)
g(x)n
j=0 j
i=0

=
=
hence
xg(x)n

m
X

!
bj f (x)j g(x)mj

j=0

= g(x)m

n
X

!
ai f (x)i g(x)ni

(13)

i=0

Let be a root of f (x). Since f (x) and g(x) are relatively prime, g() 6= 0. Evaluating
both side of the eqn. 13 at , we obtain g()n+m b0 = g()n+m a0 . Since u(x) and
v(x) are relatively prime both of their constant terms, a0 and b0 , cannot be zero
simultaneously. This means that f (x) has at most one root; = a0 /b0 . It follows
from our initial assumption that the degree s of g(x) is 2. Now that we know
0 r 1 < s, we are able to argue over the leading terms of the polynomials on both
sides of eqn. 13. The degree of the leading monomial of the left hand side is 1+(m+n
j) deg g(x) for some j {0, . . . , m1}, but the leading monomial of the right hand side
is (m + n i) deg g(x) for some i {0, . . . , n 1}. Modulo deg g(x) these two integers
are not equal, hence we have a contradiction. Therefore, 0 deg g(x), deg f (x) 1.
Since we have eliminated the possibility that deg f (x) = deg g(x) = 0, (if f (x) is
constant, then g(x) is not, and vice versa) we see that f (x) = bx + a, g(x) = dx + c
with different roots. This amounts to b0 c0 a0 d0 6= 0.

159. Determine the fixed subfield of the F -automorphism u : F (x) F (x) defined
by x 7 x + 1.
81

Solution.
We analyze the situation according to the characteristic of the underlying field. First
assume that the characteristic is 0 and let f (x) and g(x) be two relatively prime
polynomials such that f (x)/g(x) F (x) is u-fixed. Since f (x+1)/g(x+1) = f (x)/g(x),
replacing x by x + 1, or by x 1, we see that f (x + a)/g(x + a) = f (x)/g(x) for all
a Z. Define G(x, y) := f (x + y)g(x) f (x)g(x + y) Q[x, y]. It is clear that G(x, y)
vanishes on Z2 , therefore, it is identically zero. It follows that f (x)/g(x) is constant.
If F has characteristic p, then the answer is not as simple as in the previous case. We
claim that f (x)/g(x) is invariant under x 7 x + 1 if and only if it is of the form
Q
r( pi=0 (x i))
Q
s( pi=0 (x i))
for some polynomials r(x), s(x) F [x] with s(x) 6= 0. We prove this by induction
on the total degree n = deg f (x) + deg g(x). If n = 0, then there is nothing to
prove because f (x)/g(x) is a constant. Assume that the result holds for n < d, and
let f (x)/g(x) be a rational function that is invariant under translation x 7 x + 1
and deg f (x) + deg g(x) = d. Without loss of generality, we assume that deg f (x)
deg g(x), since otherwise its reciprocal satisfies the same property as well.
Now, since f (x)/g(x) is translation invariant, so is f (x)/g(x) f (a)/g(a) for any
a {0, . . . , p 1}. We fix one such a, and let c = f (a)/g(a). Since f (x)/g(x) c =
(f (x) cg(x))/g(x), we see that f (x)/g(x) c is translation invariant if and only if
f (x) cg(x) vanishes at every x = 0, . . . , p 1, or equivalently, it is divisible by u(x).
But then, (f (x)/g(x) c)/u(x) is a rational function of total degree d p < d and
still translation invariant. Hence, by induction hypothesis, it is a rational function in
u(x), hence so is f (x)/g(x). This finishes the proof of our claim and the answer to the
question.

160. If K is the splitting field of a polynomial f (x) over a field F , then


|AutF K| dimF K.

(14)

Moreover, eqn. (14) is an equality when f (x) is separable.


Solution.
We start with a simple observation. We fix a root K of an irreducible factor
f1 (x) of f (x). We claim that the degree of f1 (x) is greater than or equal to number of
embeddings : F () K such that (x) = x for all x F . Moreover, we claim these
two numbers are equal when f1 (x) is separable. To prove our claims, first, we extend
to an automorphism
: K K (see Problem 121). Since all coefficients of f1 (x) are
from F ,
(f (x)) = f (x), hence
permutes the roots of f1 (x) among themselves. In
82

other words,
() = for another root of f1 (x). Therefore, the number of injective
homomorphisms : F () K which are constant on F are bounded by the degree of
f1 (x), and simply equal to the degree when all roots of f1 (x) are distinct.
Now, we are ready to prove eqn. (14) by using induction on the degree of the extension
K over F . If the extension degree is 1, then there is nothing to prove. Assume otherwise
that dimF K = n and that our claim is true for all splitting fields of degree d < n.
Let be a root of an irreducible divisor f1 (x) F [x] of f (x) F [x]. By the above
observation we know that any AutF K gives us an embedding (that is to say, an
injective field homomorphism) : F () , K and the number of such embeddings is
dimF F () with equality when f1 (x) is separable.
Now, the question is how many different F -automorphisms of K restrict to the same
map ? Let 1 and 2 be two elements of AutF K such that 1 |F () = 2 |F () = . Since
21 1 fixes F () point-wise, we see that the number of distinct F -automorphisms of
K that restricts to the same embedding : F () , K is bounded by AutF () K.
Therefore,
|AutF K| |AutF () K| deg f1 (x)
= |AutF () K| dimF F ()
dimF () K dimF F ()
= dimF K.

(by induction)

161. Prove that if dimF K is finite, then K is the splitting field of a single polynomial f (x) over F .
Solution.
Let 1 , . . . , k K be a basis for K as a vector space over F and let f1 (x), . . . , fk (x)
denote their
polynomials over F of i s. We claim that K is the splitting field
Qminimal
k
of f (x) := i=1 fi (x). Indeed, if L is an intermediate field F L K such that f (x)
splits completely into linear factors over L, then i s are contained in L. Obviously,
this implies that L = K.

Fact 0.54. Let G be a subgroup of the automorphism group AutF K of a field extension.
The fixed field L = {x K : (x) = x for all G} of G has the right dimension:
|G| = dimL K.
In particular, if K is a finite extension of F , then |AutF K| dimF K.

83

162. Let F K be a finite extension. Any subgroup G of AutF K is equal to


AutE K, where E = K G is the fixed subfield of G. Furthermore, the map
G 7 E which assigns a subgroup G its fixed subfield E = K G is injective.
Solution.
By definition, G AutE K. Since |AutE K| dimE K = |G| by the above fact, we see
that AutF K = G. Of course, finiteness played a role in here. Now, our second claim is
obvious: if G1 , G2 are two subgroups of AutK F with the same fixed subfield E, then
G1 = AutE K = G2 .

163. Prove that Q( 2) is not isomorphic to Q( 3).


Solution.

from Q( 3) to Q( 2))
It is clear that any isomorphism
from Q( 2) to Q( 3) (or
has tomap Q to
Q is mapped
Q. If a + b 2, a, b
to 3 by an isomorphism
f : Q( 2) Q( 3), then f (a2 +2b2 +2ab 2) = 3, or f ( 2) = (3f (a2 +2b2 ))/f (2ab),
which is an element of Q. This contradicts with the fact
that f 1 , the inverse of f

maps the rational number (3 f (a2 + 2b2 ))/f (2ab) to 2.

164. Determine the splitting field of f (x) = (x2 2)(x2 3)(x2 5) over Q and
compute its degree over Q.
Solution.

knowthat
the splitting field of (x2
Roots of f (x) are
2, 3, 5. We already

3)(x2 2) is Q( 2, 3). Next we show


/Q( 2, 3). Assume otherwise that
5
that
there exists a, b, c, d Q such that 5 = a 2 + b 3 + c 6 + d. Taking the square of
both sides of the equation, we see that

5 = (a 2 + b 3 + c 6 + d)2

= 2a2 + 3b2 + 6c2 + d2 + 2ab 6 + 4ac 3 + 2ad 2 + 6bc 2 + 2bd 3 + 2cd 6

= 2a2 + 3b2 + 6c2 + d2 + 6(2ab + 2cd) + 2(2ad + 6bc) + 3(4ac + 2bd).


Thus, 1) ab + cd = 0, 2) ad + 3bc = 0, 3) 2ac + bd = 0 and 4) 2a2 + 3b2 + 6c2 + d2 = 0.
First three equations imply that a2 = 3c2 , 2a2 = d2 , 2a2 = 3b2 . By the 4th equation

we
see
that 8a2 = 5. Since this equation has no solution
in rationals, we see that 5
/

Q( 2, 3), and
the minimal polynomial
splitting
since

of 5 is
of degree two, the
field
of f (x) is Q( 2, 3, 5). Let K denote
Q(
2,
3,
5).
Since
dim
Q(
2,
3) = 4,
Q

and K is a degree 2 extension over Q( 2, 3), we see that the vector space dimension
of K over Q is 8.

84

165. Determine the automorphism group of the splitting field of f (x) = x3 3x + 1


over Q.
Solution.
f (x) is irreducible over Z (hence over Q as well). To see this we reduce mod 2 and
evaluate: f (0) = 1, f (1) = 1. Therefore, f (x) is irreducible.
Next, we analyze the derivative of f (x) to obtain information about its roots. f 0 (x) =
3x2 3 is positive outside the interval (1, 1) and negative in the interval (1, 1).
Since f (1) = 3 and f (1) = 1 we see that there are three real roots, denoted 1 , 2
and 3 . Since f (x) is irreducible of degree 3, dimQ Q(1 ) = 3.
Let K denote the splitting field of f (x). Thus, K is either degree 6 over Q, or it is degree
3 over Q. In the latter case the automorphism group is cyclic group of order 3. Let us
prove that the former case is not possible. In fact, in the former case the automorphism
group G = AutQ K is S3 , the symmetric group on 3 letters. Since the roots of f (x) are
permuted by the elements of G, we let S3 be the automorphism that maps 1 to 3 ,
3 to 1 and 2 to 2 . Write f (x) = (x1 )(x2 )(x3 ) = x3 3x+1. By taking the
derivative of f (x) we obtain (x2 )(x3 )+(x1 )(x3 )+(x1 )(x2 ) = 3x2 3.
Substituting 2 into this equation gives
(1 2 )(1 3 ) = 312 3
(2 1 )(2 3 ) = 322 3
(3 1 )(3 2 ) = 332 3
Q
On the one hand, multiplying right hand sides gives: 33 3i=1 (i 1)(i + 1) which is
equal to 27f (1)f
= 81. On the other hand, multiplying left hand
Q (1) = 27(1)3
2
sides gives 1i<j3 (i j ) . In summary, after simplification (canceling negative
signs and taking square roots) we have
Y
(i j ) = 9.
1i<j3

The right hand side is sent to its negative by the permutation we picked above, but 9
is obviously fixed by . Therefore, cannot be in G. It follows that the automorphism
group G is not equal to S3 , therefore, it must be the cyclic group of order 3. Moreover,
Q(1 ) = Q(1 , 2 , 3 ).

Definition 0.55. A field extension F K is called Galois, if it is


algebraic,
normal (splitting field of a family of polynomials over F ), and
separable.
85

The Galois group of a Galois extension is AutF K.


166. It follows from Problem 160 that the splitting field K of a polynomial f (x)
F [x] is Galois, then |AutF K| = dimF K. Prove more generally that for a finite
extension F K the following are equivalent:
(a) F K is Galois;
(b) |AutF K| = dimF K;
(c) K is the splitting field of a single separable polynomial over F .
Solution.
Let F K be a finite extension and let 1 , k be an F -vector space basis for K.
Let p1 (x), . . . , pk (x) denote the minimal polynomials of i s over F . Let S denote the
set
Q of linear factors appearing in any of pi (x)s, and let g(x) denote the polynomial
s(x)S s(x). In other words, g(x) is obtained from product of pi (x)s by reducing the
multiplicity of each repeated linear factor to 1. It is clear that g(x) is separable. Since
each AutF K permutes the roots of pi (x), the polynomial g(x) is invariant under
each AutF K, hence g(x) is defined over E, the fixed field of AutF K.
Now, we are ready to prove our claims.
((b) (a)) First, assume that |AutF K| = dimF K. By Fact 0.54 we know that
dimF E = |AutF K|. It follows that dimF E = dimF K, hence, F = E. Therefore, g(x)
is defined over F . Since g(x) is separable, and K is the splitting field of g(x) F [x],
K is a Galois extension of F .
((a) (b)) Suppose that K is Galois over F . By Problem 160, it is enough to show
that the polynomial g(x), which is defined in the first paragraph, is contained in F [x].
To this end, let K be a root of p1 (x). Since p1 (x) is irreducible in F [x], it is the
minimal polynomial of over F . Therefore, for all i = 2, . . . , k, we have pi ()
Q 6= 0. In
other words, p1 (x), . . . , pk (x) do not share any common root. Thus, g(x) = ki=1 pi (x),
hence it lies in F [x].
((c) (a)) This follows from definitions.
((a) (c)) This follows from the solution of ((a) (b)).

Fact 0.56. The fundamental theorem of Galois theory (finite case): Let F
K be a finite Galois extension, and let G denote AutF K. There exists an order reversing
isomorphism between the inclusion lattice of all subgroups of G and the inclusion lattice
of all intermediate fields F E K. Explicitly, such an isomorphism is given by
E 7 AutE K, and its inverse is H 7 fixed subfield of H. Furthermore, a subgroup H
of G is normal in G if and only if the corresponding subfield E is Galois over F . (Note
that K is always Galois over E by Fact 0.54 and Problem 166.)
86

167. Determine the automorphism (Galois) group of the splitting field of f (x) =
(x2 2)(x2 3)(x2 5) over Q.
Solution.


We know from Problem 164 that the splitting field of f (x) over Q is K = Q( 2, 3, 5)
and dimQ K = 8. Hence, the Galois group G of this extension is a finite group of order
8. LetH G be
generated
, Aut
the subgroup

by the
automorphisms

Q K defined

= 5, ( 3) = 3, ( 2) = 2, and ( 5) = 5, ( 3) = 3,
by( 5)
( 2) = 2. Then H fixes Q( 2) point-wise, and furthermore H
= Z/2 Z/2.
Since H is of index 2 in G, it follows that G is an abelian group. By the fundamental
theorem of finitely generated abelian groups, the only possibility for G is that G
=
Z/2 Z/2 Z/2. (G cannot be cyclic because H is not!.)

168. Determine the automorphism (Galois) group of the splitting field of f (x) =
x4 + 2x + 3 over Q.
Solution.
Unfortunately f (x) does not split as nicely as x4 + 3x2 + 2, so things are more complicated (but not horrible). In any case, we look at



2 8
2 + 8
2
y
y + 2y + 3 = y
2
2





= y (1 + 2i) y (1 2i)
p

Therefore,
four
roots
x
,
x
,
x
,
x
of
f
(x)
are
x
=
1 + 2i, x2 = x1 , x3 =
1
2
3
4
1
p

1 2i, and x4 = x3 . Thus, the splitting field of f (x) over Q is equal to


Q(x1 , x3 ).
First, we make some obvious observations:
Since f (x) is irreducible of degree 4, hence Q(x1 ) and Q(x3 ) are degree 4 extensions
of Q;

x21 x23 = 2i, hence Q(x1 ) Q(x3 ) contains Q( 2i);

x21 x23 = 3, hence x1 = 3/x3 ;


q
q

3
1
x1 = 2 + 2 + i 12 + 23 (this requires a bit of calculation but it is not so
bad).
Next, we claim that Q(x1 ) 6= Q(x3 ). To this end, let us assume otherwise
that x3

Q(x1 ) and find a contradiction.

Indeed, if x3 Q(x1 ), then x1 = 3/x3 implies that


3 Q(x1 ), hence that Q( 3) Q(x1 ).In this case Q(x1 ) is a degree 2 extension of
Q( 3). Therefore, there exists , Q( 3) such that x21 +x1 + = 0. We write =
87

3b, = c + 3d with a, b, c, d Q, and set x1 = u + iv with u = 12 + 23 and


q

v = 12 + 23 . Thus, we have the equation 1+ 2i+(a+b 3)(u+iv)+(c+d 3) = 0.


A complex number is equal
real and imaginary
parts are both
to 0 if and
only if the

0. Thus, 1 + u(a + b 3) + (c + d 3)= 0 and 2 + (a + b 3)v = 0. It suffices

4 = (a2 + 2ab 3 +
to use
the second equation: 2 = (a + b 3)2 v 2 , or equivalently,
3)(1 + 3). Simplifying the equation gives 0 = (a2 + 6ab 4) + 3(a2 + 2ab + 3). A
straightforward calculation shows that there are no rational numbers a and b satisfying
the last equation.

Now that we know Q(x1 ) 6= Q(x3 ), we know thatQ(x1 ) Q(x3 ) = Q( 2i) also.
Furthermore, we see that X 2 x23 = X 2 (1 2i) Q(x1 )[X] is the minimal
polynomial of x3 over Q(x1 ). Therefore, Q(x1 , x3 ) is a degree 2 extension of Q(x1 ),
which is degree 4 extension of Q.
a+

Finally we are ready to determine the automorphism group of K = Q(x1 , x3 ). Obviously an automorphism of K maps x1 to one of x1 , x1 , x3 , or x3 . In particular,
any permutation of these symbols extends to a Q-automorphism of K. Let us choose
: K K to be the one with (x1 ) = x3 (hence it maps x1 to x3 ) and (x3 ) = x1 .
Also, choose : K K to be the one with (x1 ) = x3 and (x3 ) = x1 . The relations
on and are as follows:
2 = id,

4 = id,

= 3 (this needs verification but it is tedious).

These are precisely the relations that are required to generate D8 , the dihedral group
of order 8.

169. Let F K L be three field extensions and assume L is Galois over F . We


know that there exists a subgroup H AutF L such that the fixed subfield
of H is K. Let SH denote the set of all coset representatives of H in AutF L
and define the norm of an element K by
Y
NK/F () =
()
SH

(a) Show that NK/F () F .


(b) Show that NK/F () = NK/F ()NK/F ().

(c) If K is the quadratic extension Q( D) for some square-free integer


D Z, then NK/Q (a + b D) = a2 Db2 .
P
(d) If min,F (x) = di=0 ai xi F [x] is the minimal polynomial of K over F ,
prove that a) d divides n = dimF K; b) each term = () in the product
Q
n n/d
SH () occurs with multiplicity n/d; c) NK/F () = (1) a0 .
88

Solution.
1) If AutF L, then (NK/F ()) is equal to NK/F () since permutes the coset
representatives among themselves. Therefore, the norm of an element is stable under
any F -automorphism of L, hence it lies in F .
2) The multiplicativeness is clear from the multiplicativeness of automorphisms SH .
3) Consider
D0 that is relatively prime to D and let L be the field ex a number

0
tension Q( D, D ). The automorphism
by the Q Q is generated

groupof L over
0
0
automorphisms

and defined by ( D) = D, ( D ) = D , and ( D) =

0
D, ( D ) = D0 . Since |AutQ (L)| = 4 = dimQ (L), L is a Galois extension of Q.
Let H denote the subgroup generated by . It is obvious that the fixed subfield of H
is K, and the minimal coset representatives for H in AutQ L are id, . Thus

NK/Q (a + b D) = id(a + b D)(a + b D) = (a + b D)(a b D) = a2 Db2 .


4.a) Let K 0 denote the subfield F () K. Since K 0 is isomorphic to F [x]/(min,F (x)),
which is a degree d extension of F , we see that d = dimF K 0 divides dimF K = n.
4.b) Since K L is a Galois extension, by Problem 166, |H| = dimK L, therefore,
dimF L = n |H|, hence n is the number of coset representatives of H in G = AutF L.
Let R = {1 , . . . , d } denote the set of all roots of min,F (x) with = 1 . The
full automorphism group G acts on R transitively, and H G acts trivially. In
other words, G = R and H StabG (). If , G are two elements such that
() = () = , then 1 is in the
Q stabilizer subgroup of . Therefore, the number
of occurrence of in the product SH () is equal to StabG () SH . This set
is nothing but the coset representatives of H in StabG (), hence its cardinality is
equal to |StabG ()|/|H|. It remains to compute |StabG ()|. Observe that K 0 is the
fixed field of StabG (), hence AutK 0 L = StabG (). By Problem 166, we know that
|AutK 0 L| = dimK 0 L and this is equal to dimF L/ dimF K 0 = n |H|/d. Therefore,
|StabG ()|/|H| = n |H|/(d |H|) = n/d.
Q
4.c) Consider the polynomial f (x) = SH (x ()). Clearly,
Y
f (0) = (1)|SH |
() = (1)n NK/F ().
SH

From part 4.b) we know that each linear factor in f (x) occurs with multiplicity n/d,
and of the form x i for some root i R of the minimal polynomial min,F (x).
P
n/d
Therefore, f (x) = min,F (x)n/d . Therefore, f (0) = ( di=0 ai xi )n/d |x=0 = a0 . Thus,
n/d
n/d
(1)n NK/F () = a0 , or NK/F () = (1)n a0 .

170. We keep the notation from Problem 169. The trace of K is defined by
X
TrK/F () =
().
AutF L

89

Prove that
(a) Show that TrK/F () F .
(b) Show that TrK/F ( + ) = TrK/F () + TrK/F ().

(c) If K is the quadratic


extension Q( D) for some square-free integer
D Z, then TrK/Q (a + b D) = 2a.
Pd
i
(d) TrK/F () = nd ad1 , where min,F (x) =
i=0 ai x F [x] is the minimal
polynomial of K over F .
Solution.
P
P
1) Since (TrK/F ()) = AutF L () = 0 AutF L 0 () = TrK/F () for all
AutF L, we see that TrK/F () F .
2) Additivity is clear from the additivity of the automorphisms.

3) The automorphism group AutQ Q( 2) is {id, }, where ( D) = D is defined


by

TrK/Q (a + b D) = id(a + b D) + (a + b D) = (a + b D) + (a b D) = 2a.


Q
P
4) Define f (x) = AutF K (x ()). On one hand, f (0) = AutF K () =
TrK/F (). On the other hand, as in the solution of Problem 169 4.b) L is a Galois
extension of F () with AutF () L = StabG (), and furthermore, since F () is Galois
over F , StabG () is a normal subgroup of L. Let S denote the set of coset representatives for StabG (). Then S is isomorphic to AutF F () and each () is a root of
min,F (x). Therefore,
f (x) =

(x ()) =
n
d

|StabG ()|(x ()) =

StabG (), S

It follows that f (x) =

i R (x

nX
(x ()).
d S

i ). Therefore,

TrK/F () = f (0) =

n X
n
i = (ad1 ).
d R
d
i

171. Let F K be a field extension of degree n. Show that the norm and
trace of K that are defined in Problems 169 and 170 are precisely the
determinant and the trace of the linear operator T : K K defined by
T () = .
Solution.

90

First of all, let us show that the minimal polynomial of the operator T : K K
is equal to the polynomial ofPminimal degree f (x) FP
[x] such that P
f (T ) = 0 as an
i
i
ai i for any
operator on K: If f (x) =
ai x , then f (T )() =
ai T () =
K. Therefore, f (T ) = 0 as an operator on K if and only if f () = 0. Since f (x)
has the smallest degree, f (x) is the minimal polynomial of the element K over F .
We recall some facts from linear algebra: the characteristic polynomial of an operator
on an n-dimensional vector space is a polynomial of degree n, and its constant term
is the determinant (of the operator) times (1)n . The minimal polynomial divides
the characteristic polynomial. Moreover, the roots of the minimal polynomial and the
roots of characteristic polynomial are the same, hence, the characteristic polynomial
divides some power of the minimal polynomial. Finally, the sum of all roots of the
characteristic polynomial is the trace of the operator.
In the case of our operator T we know that the minimal polynomial min,F (x) is an
irreducible polynomial over F , therefore, the characteristic polynomial charT (x) is a
power of min,F (x). In particular, if the degree of min,F (x) is d, then charT (x) =
(min,F (x))n/d . Therefore, the determinant of T is equal to
n/d

det T = (1)n charT (0) = (1)n (min,F (0))n/d = (1)n a0 .


For the trace, we use the equality charT (x) = (min,F (x))n/d once again: each root
of the minimal polynomial is repeated n/d times in charT (x). Since the sum of all
roots of min,F (x) is equal to ad1 , we see that the sum of all roots of charT (x) is
n
(ad1 ). By Problem 170, we know that this quantity is precisely the trace of the
d
element K.

172. Let K be a finite extension of F . For K let : K F denote the map


defined by () = TrK/F (). Here, TrK/F is as defined in Problem 170.
Prove that the assignment 7 is an injective linear transformation,
establishing an isomorphism between K and the space of all F -valued linear
functionals on K.
Solution.
Let us denote by the map 7 and let , be two elements from K. For c F ,
we have ( + c) = +c and
X
+c () = TrK/F (( + c)) =
(( + c)) = () + c ().
AutF K

It follows that is linear. To prove its injectivitiy of , we compute its kernel. Since
(1 ) = TrK/F (1) = |AutF K|, the image of is always non-zero, therefore it has no
kernel.

91

173. Suppose K is a Galois extension of F with AutF K a cyclic group of order n


generated by AutF K. Suppose K satisfies NK/F () = 1. Prove that
is of the form /() for some non-zero K.
Solution.
For i = 0, . . . , n 1, define i (x) = i (x), x K. It is clear that {0 , . . . , n1 }
is a set of distinct characters, hence, they are linearly independent. In other words,
for any list of not-all-zero elements a0 , a1 , . . . , an1 K there exists K such that
0 ()a0 + 1 ()a1 + + n1 ()(x) 6= 0. In particular, for the choice of a0 = 1,
ai := () 2 () i1 (), i = 1, . . . , n 1 we have an element K such that
+ () + (()) 2 () + + (() n2 ()) n1 () = ,
and is non-zero. Notice that, since n = id,
() = ( + () + (()) 2 () + + (() n2 ()) n1 ())
= () + () 2 () + () 2 () 3 () + + () 2 () n1 () n ()
Multiplying both sides with , we obtain
() = () + + () 2 () n2 () n1 () + () 2 () n1 () n ()
= + () 2 () n1 () n ()
= + NK/F () n ()
= + n ()
=

174. Here is a basic problem in group actions: Let G be a group acting transitively on a finite set B and let H be a normal subgroup of G. If A1 , , As
is the list of orbits of H in B, then prove that
(a) G acts on {A1 , . . . , As } transitively;
(b) if a is any element from A1 , then the cardinality of A1 is equal to
|H|/|H StabG (a)|,
and the number s of orbits of H is equal to
|G|/|H StabG (a)|.
Solution.
(a) Let a, b be two elements from A1 and let g G. We claim that g(a) and g(b)
belongs to the same set Ai , for some i {1, . . . , s}. To see this, let 1 j, k s be two
92

indices such that g(a) Aj and g(b) Ak . Since A1 is an H-orbit, there exists h H
such that a = h(b). Then g(a) = g(h(b)). Since H is normal in G, ghg 1 belongs to
H. Therefore, g(a) = g(h(b)) = ghg 1 g(b) = h0 g(b) for some h0 H. But h0 Ak = Ak ,
since Ak is an H-orbit. Therefore, g(a) belongs to the orbit to which g(b) is a member
of. Therefore, G acts on {A1 , . . . , As }. Since the action of G on B is transitive, its
action on the set of H-orbits is transitive, also.
(b) Since A1 is an H-orbit, H acts on A1 transitively. Let a A1 . Then StabH (a) =
H StabG (a), and therefore, |A1 | = |H|/|H StabG (a)|. For our last claim, we are
going to show that
StabG (a) H = StabG (A1 ).
The containment is obvious. By definition, if g StabG (A1 ), then g(b) A1
for any b A1 . In particular, g(a) A1 . Let h H be the element such that
h(g(a)) = a. Since H is normal in G, g 1 hg is an element h0 of H. Therefore,
a = hg(a) = gg 1 hg(a) = gh0 (a) implies that gh0 StabG (a), hence g StabG (a) H.

175. Let f (x) F [x] be an irreducible polynomial of degree n over F , and let L
be the splitting of f (x) over F . If L is a root of f (x), and K is any Galois
extension of F contained in L, then show that the polynomial f (x) splits
into a product of m irreducible polynomials each of degree d over K, where
m = dimF (F () K) and d = dimK K().
Solution.
Suppose f1 (x) fs (x) is the factorization of f (x) into irreducible factors over K. Let
A1 , . . . , As denote the sets of roots of the factors f1 (x), . . . , fs (x), respectively. We fix
an irreducible factor, say f1 (x), and let L be one of its roots. The degree d of
f1 (x) is the degree of the extension K K().
We know that both F L and K L are Galois extensions. Let G and H denote
AutF L and AutK L, respectively. Since K is Galois, H is normal in G. Since H fixes
K point-wise, each irreducible factor fi (x) is invariant under H, hence, Ai is invariant
under the action of H. Furthermore, since fi (x) is irreducible, H acts on Ai transitively,
hence Ai is an orbit of H. By Problem 174.b), we see that the cardinality of Al = j Ai
is the same as that of Ai . Thus, the degree of (any) irreducible factor fl (x) of f (x) is
equal to that of f1 (x).
On the other hand, by Problem 174, the number s of orbits of a normal subgroup H is
equal to the cardinality of the quotient G/HH1 , where H1 = StabG (), where A1 .
Note that H1 = StabG () is the automorphism group of F () over F , and F () is a
Galois extension. Therefore H1 is normal in G. The product of two normal subgroups
is normal as well. It follows that the fixed field K 0 of HH1 in L is a Galois extension
over F . Since K 0 = K F (), by Problem 166, we see that dimF K F () is equal to
|G/HH1 | = s.
93

176. Determine the splitting field of xp x a over Fp , where a 6= 0, a Fp .


Solution.
We already know from Problem 130 that A(x) = xp x a is irreducible and separable
over Fp . Let K denote its splitting field over Fp . Also,
Q we know from the solution of
Problem 130 that if K is a root, then A(x) = p1
k=0 (x k). Therefore, if
AutFp K is an automorphism, since A(x) is invariant under , we see that () is a
root of A(x), thus, () = +l for some l Fp . Clearly, in this case, (+k) = +k+l
for all k Fp . It follows that if is the automorphism defined by () = + 1, then
= k , hence generates AutFp K, as a cyclic group of order p. Since K is Galois
over Fp , we know now that p = |AutFp K| = dimFp K. On the other hand, since A(x) is
irreducible, Fp [x]/(A(x)) is a field extension of degree p, and has a root of A(x), hence
contained in the splitting field of A(x). Therefore, K is equal to Fp [x]/(A(x)).

Definition 0.57. Let x1 , . . . , xn be algebraically independent variables over Q. A


polynomial f (x1 , . . . , xn ) Q[x1 , . . . , xn ] is called symmetric if for any permutation
Sn , the polynomial f (x1 , . . . , xn ) = f (x(1) , . . . , x(n) ) is equal to f (x1 , . . . , xn ).
Let k be a number from {1, . . . , n}. The kth elementary symmetric polynomial in
variables x1 , . . . , xn is denoted by ek and defined by
X
ek = ek (x1 , . . . , xn ) =
xi 1 xi k .
(15)
1i1 <<ik n

The kth power sum symmetric polynomial in variables x1 , . . . , xn is denoted by pk and


defined by
pk = pk (x1 , . . . , xn ) = xk1 + + xkn .

(16)

177. Prove Newtons identities:


pk pk1 e1 + + (1)k1 p1 ek1 + (1)k kek = 0.

Solution.
Let y be a variable. Define E(y) =
that

Qn

i=1 (1

xi y). By expanding the product we see

n
Y
E(y) =
(1 xi y) = 1 e1 y + e2 y 2 + + (1)n en y n .
i=1

94

(17)

1
Since 1 + t + t2 + = 1t
= (1t)
= dtd ln(1 t), we see that ln(1 t) =
(1t)
t + t2 /2 + t3 /3 + . Therefore,

ln E(y) =

n
X

ln(1 xi y) =

i=1

n X

X
(xi y)j
i=1 j=1

X
pj j
j=1

Taking the derivative of both sides with respect to y gives

X
X
E 0 (y)
=
pj y j1 or E 0 (y) = E(y)
pj+1 y j .
E(y)
j=1
j=0

For good reasons, it is convenient to define e0 = p0 = 1. By comparing the coefficients


of y k1 on the expansion of E 0 (y) and the product on the right hand side of the last
equation, we obtain Newtons identity
k

(1) kek =

k1
X

(1)j ej pkj1 .

j=0

178. Prove that over a field F of characteristic 0, an n n matrix A is nilpotent


if and only if the trace of Ak is zero for all k = 1, 2, . . . .
Solution.
Assume that A is nilpotent, hence the minimal polynomial of A in F [x] is xk , where
k is the smallest positive integer such that Ak = 0. It follows that the characteristic
polynomial of A is equal to xn . The trace of A is equal to an1 , where an1 is the
coefficient of xn1 in the characteristic polynomial of A. Therefore, we see that the
trace of A is 0. Similarly, since Ar is nilpotent for any power r 2 of A, the traces of
Ar s are zero, as well.
Conversely, assume that trace(Ak ) = 0 for all k 1. If is an eigenvalue of A, then k
is an eigenvalue for Ak . Since the trace of an operator is the sum of all eigenvalues, we
see that the sums of powers of all eigenvalues of A are 0. On the other hand, by using
Problem 177 we see that the elementary symmetric polynomials e1 , . . . , en evaluated
at the eigenvalues of A are all zero, also. Since the coefficients of the characteristic
polynomial of an operator is precisely the elementary symmetric polynomials evaluated
at the eigenvalues, we see that the characteristic polynomial of A is equal to xn . Since
an operator is annihilated by its characteristic polynomial, we see that An = 0, hence
A is nilpotent.

95

179. Prove that over a field of characteristic 0 two n n matrices A and B have
the same characteristic polynomial if and only if trace(Ak ) = trace(B k ) for
all k 0.
Solution.
If A and B have the same characteristic polynomial then the elementary symmetric
polynomials in eigenvalues of A are equal to that of B. By Newtons identity, inductively, we express the power sums of eigenvalues of A in terms of the elementary
symmetric polynomials. Therefore, the trace of Ak , which is equal to the value of the
kth power sum symmetric polynomial evaluated at the eigenvalues of A is equal to that
of B. Conversely, if trace(Ak ) = trace(B k ) for all k 0, then the elementary symmetric polynomials evaluated at the eigenvalues of A is equal to that of B. Therefore, A
and B have the same characteristic polynomial.
Remark 0.58. For the last two problems, it is enough to restrict k to {1, . . . , n}.

180. By using the previous problem, show that the characteristic polynomial of
AB and that of BA are the same.
Solution.
Let us first assume that the trace of AB is the same as that of BA for any two n n
matrices A and B. It follows that the traces of (AB)m and (BA)m are the same (by
letting C = A(BA)m1 , and D = B) for all m 2. By Problem 179 we see that the
characteristic polynomials of AB and BA are the same.
Next, we show that trace(AB) = trace(BA). Let Aij and Bij , i, j = 1, . . . , n denote
the entries of A and B, respectively. The trace of AB is the sum of its diagonal entries:
trace(AB) =

n
n X
X

Akj Bjk

k=1 j=1

Interchanging the sums as well as the entries we have the same result:
n X
n
X
k=1 j=1

Akj Bjk =

n X
n
X

Bjk Akj = trace(BA).

j=1 k=1

Remark 0.59. This result is true over arbitrary field.

96

Definition 0.60. A (commutative, of course) ring R is called Artinian if any decreasing sequence of nested ideals I I1 I2 stops (becomes an equality) after
finitely many steps.
A (commutative, of course) ring R is called Noetherian if any increasing sequence of
nested ideals I I1 I2 stops (becomes an equality) after finitely many steps.
These two definitions extend to an R-module M , by replacing ideals by submodules.

181. Prove that in a Noetherian ring R every ideal is finitely generated and
conversely a commutative ring R in which every ideal is finitely generated
is Noetherian.
Solution.
Suppose R is a Noetherian ring and I R is an ideal. Let I0 = (a), a I be a sub
ideal of I generated by one element. If I0 = I, then we proved our claim. Otherwise,
let a1 I I0 be another element from I and let I1 be the subideal generated by a
and a1 . If I1 = I, then we proved our claim. Otherwise, we continue. Since we have
an increasing sequence of ideals I0 I1 I2 , this process stops at step n (by
our Noetherian assumption). Therefore, In = I.
Conversely, if every ideal in R is finitely generated, and I0 I1 is an increasing
chain of ideals in R, then we let I = Ii the union of all ideals in the sequence. It
straightforward to verify that I is an ideal. Therefore, I has finitely generated, say by
a0 , a1 , . . . , an . Let Im be one of the ideals of the sequence that contains all of these
elements. Then I = Im = Im+i for all i 0, hence R is Noetherian.

182. Determine if the ring C([0, 1]) of all real valued continuous function on the
unit interval [0, 1] is Noetherian or not.
Solution.
For n Z+ let An denote the interval [0, 1/n]. Let In denote the set of all continuous
real valued functions on [0, 1] such that f (x) = 0 for all x In . It is straightforward
to check that a) In is an ideal for all n Z+ and b) In Im for all n m.
On the other hand, not all functions in In+1 are contained in In . For example, let
g(x) : [0, 1] R denote the function that takes value 0 on [0, 1/n + 1] and
g(x) =

(n + 1)(x 1)
+ 1 for x [1/(n + 1), 1].
n

It is clear that g(x) is continuous, g(x) In+1 but g(1/n) 6= 0. Therfore, C([0, 1]) is
not Noetherian.
97

183. Show that the ring of all functions from X to Z/2 is not Noetherian if X
has infinite number of elements.
Solution.
This is similar to the previous problem: let xn , n = 1, . . . be a sequence of distinct
elements from X, An denote the set {xn , xn+1 , . . . }. Then the set In of all functions
f : X Z/2 such that f (x) = 0 if x An is an ideal. It is also clear that In In+1 .
On the other hand, the function defined by f (x) = 1 on {x1 , . . . , xn } and f (x) = 0 on
An+1 does not belong to In . Therefore, I1 I2 does not stabilize.

184. A module M over a field F is Artinian if and only if M is Noetherian if and


only if M is a finite dimensional vector space over F .
Solution.
Let us first prove that Artinian finite dimensional. The implication 00 is
obvious. Conversely, let M be an Artinian F -module. If B is a basis for M , then
let M1 be the proper vector subspace spanned by the set B1 = B {u1 } obtained
from B by omitting one of its elements u1 B, M2 be the subpace of M1 spanned
by B2 = B1 {u2 }, and so on. We construct Mi from Mi1 similarly by taking out
one of the basis elements of Mi . Since M is Artinian, there exists n 1 such that
Mn = Mn+k for all k 0 thus Bn = Bn+k = for all k 0. In other words, B is finite.
Next, we prove Artinian Noetherian.
An increasing sequence of vector subspaces
{0} M1 Mn1 Mn M
gives a decreasing sequence of quotients in the other direction
M/{0} M/M1 M/Mn1 M/Mn M/M
If M is Artinian, hence finite dimensional, by extending a basis of Mi to a basis of M ,
we see that there exists a subspace Wi (spanned by the newly added basis elements) of
M such that Wi
= M/Mi . Furthermore, there exists n 1 such that Wn+k = Wn for
all k 0. It follows that M/Mn+k = M/Mn for all k 0, or equivalently, Mn+k = Mn
for all k 0, hence M is Noetherian. Conversely, if M is Noetherian, let B be a basis,
and let Mi be the submodule of M generated by the first i elements of B. Clearly,
Mi Mi+1 for all i, and since M is Noetherian, the sequence stabilizes. Therefore, B
is finite, hence M is a finite dimensional vector space, implying that M is Artinian.
Remark 0.61. Our proof above relies on Axiom of Choice..

98

185. Show that if R is Artinian (respectively, Noetherian), I R is an ideal, then


R/I is Artinian (respectively, Noetherian).
Solution. Any nested decreasing (respectively, increasing) sequence of ideals in R/I
pulls back to a to nested decreasing (respectively, increasing) sequence of ideals in R
via the canonical projection : R R/I, therefore, it stabilizes after finitely many
steps.

186. Let 0 M 00
M
M 0 0 be a short exact sequence of R-modules.
Show that M is a Noetherian R-module if and only if both M 00 and M 0 are
Noetherian.
Solution.
First, assume that M is Noetherian. Any increasing subsequence of submodules in a
submodule M 00 of M is an increasing subsequence of submodules in M . Therefore,
any R-submodule of M is Noetherian. If 0 M 00 M M 0 0 is short exact,
then M 0
= M/M 00 . Any increasing sequence of R-modules in M/M 00 pulls back to an
increasing sequence of R-submodules containing M 0 in M . Since M is Noetherian, the
sequence of preimages stabilizes at finitely steps, so does the images in M 0 . Therefore,
M 0 is Noetherian, as well.
Conversely, assume that both of the R-modules M 00 and M 0 are Noetherian, and let
{Ni } be an increasing sequence of R-submodules in M . The set of R-submodules
{g(Ni )} of M 0 is an increasing sequence as well, hence, there exists a sufficiently large
m0 such that g(Nn+m0 ) = g(Nm0 ) for all n 0. Since {f (M 00 ) Nm } is an increasing
submodule of the Noetherian R-module f (M 00 ), we see that there exists a sufficiently
large integer m1 such that f (M 00 ) Nm1 +n = f (M 00 ) Nm1 for all n 0. Set m :=
max{m0 , m1 }. Now, on the one hand we have f (M 00 )Ns = ker gNs for all s 0, thus,
ker g Ns+m = ker g Nm for all s 0. On the other hand we have g(Ns+m ) = g(Nm )
for all s 0. Therefore, Ns+m = Nm for all m 0, hence M is Noetherian.

187. Let 0 M 00
M
M 0 0 be a short exact sequence of R-modules. Show
that M is Artinian if and only if both of the R-modules M 00 and M 0 are
Artinian.
Solution.
The arguments of the proof are the same as in that of Problem 186, where we replace
Noetherian by Artinian.

99

188. Let M be a maximal ideal in a ring R and suppose that M n = 0 for some
n 1. Prove that R is Noetherian if and only if R is Artinian.
Solution. As a convention we set M 0 = R. The sequence of R-modules
0 M i+1 M i M i /M i+1 0

(for all i)

is exact. Therefore M i is Artinian if and only if M i+1 and M i /M i+1 are Artinian.
Similarly, M i is Noetherian if and only if M i+1 and M i /M i+1 are Noetherian. In particular, at the last two steps: M n2 is Artinian (respectively, Noetherian) iff M n1 and
M n2 /M n1 are Artinian (respectively, Noetherian), and M n1 is Artinian (Noetherian) iff M n = {0} and M n1 /M n are Artinian (Noetherian).
We first assume that R is Noetherian and show that it is Artinian. If R is Noetherian, then for all i = 0, . . . , n 1, the modules M i+1 , M i /M i+1 are Noetherian. But
R-module structure on the quotient module M i /M i+1 is the same as R/M -module
structure on it. Since R/M is a field, by Problem 184, M i /M i+1 is Noetherian iff it is
Artinian. In particular, at the last step we have M n1
= M n1 /M n , therefore, M n1
n1
is Noetherian iff it is Artinian. Now, since M
and M n2 /M n1 are Artinian, by
Problem 187, M n2 is Artinian. Thus, going down in the indices, we see that M 0 is
Artinian.
The argument for the converse (R is Artinian = R is Noetherian) is identical except
we use Problem 186 instead of Problem 187 at a suitable place.

Definition 0.62. The Krull dimension of a ring R is the length of the maximum
possible length of chains P0 P1 Pn of distinct prime ideals in R.
189. Prove that if Krull dimension of an integral domain R is 0, then R is a field.
Solution. In an integral domain {0} is a prime ideal. If the Krull dimension of R is 0,
then it means there is no maximal ideal other than {0}, therefore, R is a field.

100

Definition 0.63. A discrete valuation on a field K is an integer valued function


: K Z {} such that
is surjective and (x) = x = 0;
(x) + (y) = (xy) for all x, y K;
(x + y) min{(x), (y)} for all x, y K.
The subset {x K : (x) 0} {0} is called the valuation ring of . A discrete
valuation ring R is an integral domain such that R is the valuation ring of a valuation
of a discrete valuation on its field fractions K. We abbreviate discrete valuation ring
by DVR.
190. Show that if R is a DVR, then for each element x from the fraction field K
of R either x R, or 1/x R.
Solution.
Suppose R is a DVR with valuation , and let x K R. Then (x) < 0. Observe
that (1) = (1 1) = (1) + (1), therefore, (1) = 0. It follows from 0 = (1) =
(x 1/x) = (x) + (1/x) that (1/x) = (x). In other words, 1/x R.
Remark 0.64. The converse of this problem is also true; if R is an integral domain
such that for all x K either x R, or 1/x R, then R is a DVR.

191. Let R be an integral domain which is local. Suppose that the maximal ideal
M of R is principal and of the form M = (t), t R, with n0 (tn ) = {0}. Prove
that R is a DVR.
Solution. Let x R be an arbitrary element. If x is a unit, then we define (x) = 0.
We claim that any element x of R is of the form utm for some m 0. Indeed, given
a non-zero element x R, let m denote the highest power of t that divides x, so that
we have x = tm a for some a that is not divisible by t. Notice that the finiteness of
m is guaranteed by the fact that n0 (tn ) = 0. Since t - a, a does not belong to M ,
hence invertible (since there is only one maximal ideal). Thus, it makes sense to define
(x) = m. In particular, (a) = 0 if a is invertible. Next, if x/y is an element of the
fraction field K, then there exists b R a unit, and s Z such that x/y = bts . Indeed,
m1 m2
if x = a1 tm1 , y = a2 tm2 with m1 , m2 0, a1 , a2 R are units, then x/y = a1 a1
.
2 t
0 0
Note that m1 m2 is uniquely determined by x/y. Indeed, if x , y R is another pair
such that x/y = x0 /y 0 , then xy 0 = x0 y (R is an integral domain). It follows that the
difference of the highest powers of t that divide x0 and y 0 is equal to the difference
of the highest powers of t that divide x and y, which is m1 m2 . Finally, we define
(0) = . Thus, we have a well-defined function from K onto Z {}.
101

It is clear that the valuation ring of is equal to R. First of all, since (ts ) = s for
any s Z, subjectiveness is evident. Secondly, if z, w K, then z = u0 ts , w = v0 tr
for some r, s Z and units u0 , v0 R. Therefore, (zw) = (u0 v0 tr+s ) = r + s =
(z) + (w). Next, without loss of generality assume that s r. In this case, z + w =
ts (u0 + v0 trs ). Since r s is positive, u0 + v0 trs is an element of M , therefore,
(u0 + v0 trs ) = 0. Thus, (z + w) = (ts ) + (u0 + v0 trs ) = s which is greater than
or equal to min{(z), (w)} = s.

Definition 0.65. A field extension K over F is called purely inseparable if for each
K the minimal polynomial of over F has only one root (with multiplicities
allowed).
192. Assume that K is an algebraic extension of F and the characteristic of K is
p. Prove that the following are equivalent:
(a) K is purely inseparable over F ;
(b) if K is separable over F , then F ;
n

(c) if K, then p F for some n = n , and min,F (x) = xp p .


Solution.
(a) (c): If K is purely inseparable over F and K then we have two cases; either
F in which case there is nothing to prove, or
/ F . In the latter case, the minimal
polynomial f (x) of is of the form f (x) = (x )m for some m. If is separable,
by definition, m = 1 so our claim follows. If is not separable, then f 0 (x) = 0 by
Problem 129. Since f 0 (x) = m(x )m1 , this is possible if and only if m is power pn
of p.
n

(c) (b): Let K be an element which is separable. By our assumption, p F


n
n
for some n = n and the minimal polynomial of is f (x) = xp p . Once again,
n
by Problem 129, we see that f 0 (x) must be non-zero. But f 0 (x) = pn xp 1 , which is 0,
unless n = 0. In this case, f (x) = x , hence F .
(b) (a): Let K be an element. If is separable, F , hence itsPminimal
i
polynomial f (x) = x has a unique root. If is not separable, and f (x) = m
i=0 ai x
is its minimal polynomial, then since f 0 (x) = 0 by Problem 129, the exponent of each
variable in any monomial in f (x) is divisible by p. Therefore, f (x) = f1 (xp ) for some
polynomial f1 (x) F [x]. Notice that f1 (x) is irreducible, otherwise f (x) would be
reducible. If f1 (x) is not separable, then its derivative vanishes, and we repeat the
above argument. Since f (x) has finite degree, this process eventually stops, hence we
arrive at a smallest possible positive integer k such that fk (x) F [x] is separable and
k
k
irreducible with f (x) = fk (xp ). Notice that p is a root of fk (x). Since fk (x) is
k
k
separable, by our initial assumption, we have that p F hence that fk (x) = x p .
102

Therefore, f (x) = xp p . Notice also that if 6= is another root for f (x), then
k
k
k
k
fk ( p ) = 0, or ( )p = p p = 0 implying that = . (Our proof proved (b)
(c), also.)

193. Prove that purely inseparable extensions are normal, that is to say, splitting
field of a family of polynomials.
Solution.
By Problem 122 we know an algebraic field extension K over F is a splitting field if
and only if any irreducible polynomial f (x) F [x] with a root in K splits completely
into linear factors in K[x].
Now, let us assume that K is purely inseparable over F , and let us let f (x) denote
an irreducible polynomial over F with a root in K. Since the minimal polynomial
n
n
min,F (x) divides f (x), they must be equal. By Problem 192, then f (x) = xp p ,
n
n
for some n, and furthermore, p F . Since the characteristic is p, f (x) = (x )p ,
hence f (x) splits completely, hence K is normal.

194. Suppose we have field extensions F K L, an element L, and f (x) =


min,F (x). Prove that K F F ()
= K[x]/(f (x)) as K-algebras.
Solution.
Let k and
map

Pm1
i=0

ai i be two elements from K and F (), respectively and let be the


(k

m1
X
i=0

ai ) =

m1
X

kai xi

mod (f (x))

i=0

extended by linearity to a K-algebra map : K F F () K[x]/(f


Ps(x)). iIn other
words, (k g()) = kg(x) mod (f (x)). In particular, if g(x) = j=0 bi x K[x]
with deg g(x) < deg f (x), then
(b0 1 + b1 + + bs s ) = b0 + b1 x + + bs xs .
Therefore, is surjective. Obviously, a polynomial g(x) K[x] is divisible by f (x),
then g() = 0. Therefore, is injective, hence it is an isomorphism of K-algebras.

195. Suppose we have field extensions F K1 L and F K2 L with K1 and


K2 are algebraic over F , and assume that L is separable. Prove that the
nilradical of F -algebra K1 F K2 is trivial.
103

Solution.
Let N denote the nilradical of K1 F K2 . We claim that it is enough to show that
N (K1 F F ()) = {0} for all nonzero K2 .
P
To see this, suppose x = ri=1 ai bi K1 F K2 is a nilpotent element. In this case,
x is a nilpotent element in K1 F F (b1 , . . . , br ). Since K2 is separable, F (b1 , . . . , br ) is
separable over F . By primitive element theorem, there exists F (b1 , . . . , br ) such
that F (b1 , . . . , br ) = F (). In other words, x is a nilpotent element in K1 F F ().
To see that K1 F F ()) is reduced, we use Problem 194: if x K1 F F () were
nilpotent, then its image (x) K1 [x]/(f (x)) is nilpotent also, where f (x) is the
minimal polynomial of . Since f (x) is irreducible, hence prime in K1 [x], K1 [x]/(f (x))
is an integral domain. Therefore, (x), hence x is not nilpotent.

196. Prove that F (x) is not a finitely generated F -algebra.


Solution.
Assume otherwise that the rational functions

(x)
f1 (x)
, . . . , fgnn (x)
g1 (x)

generates F (x) as an F -

1 (x)
algebra. Let f (x) denote g1f(x)+a
F (x), where a F is such that gcd(g1 +a, gi ) = 1 for
all i = 1, . . . , n as well as gcd(g1 +
 a, f1 ) = 1. Let P (y1 , . . . , yn ) be the polynomial with
(x)
(x)
coefficients from F such that P fg11 (x)
, . . . , fgnn (x)
= f (x). Equating the denominators
on the left hand side gives us the equality

H(f1 (x), . . . , fn (x), g1 (x), . . . , gn (x))


f1 (x)
=
s
s
n
1
g1 (x) gn (x)
g1 (x) + a
for some non-negative integers s1 , . . . , sn and for some polynomial H(z1 , . . . , z2n ) with
coefficients from F . This contradicts with our initial assumption that g1 (x) + a is
relatively prime with each of the polynomials f1 (x), g1 (x), . . . , gn (x).

Definition 0.66. A numerical semigroup is a subset of non-negative integers which


is closed under addition.
197. Show that numerical semigroups are finitely generated.
Solution.
Let S be a numerical semigroup. If all elements of S have a common divisor a Z+ ,
then S 0 := {b/a : b S} is a numerical semigroup, also. Furthermore, S 0 is finitely
generated, then so is S. Therefore, without loss of generality we assume that S contains
two relatively prime numbers a, b S.

104

The fact, which goes back to James Joseph Sylvester, is that the largest integer that
does not belong to the numerical semigroup generated by a and b is ab a b. In
other words, if n > c = ab a b, then n S. It follows that S is (finitely) generated
by its elements smaller than or equal to c.

198. Show that any subring R of F [x] that contains F is Noetherian, but not
necessarily a UFD.
Solution.
Let I R be a non-trivial ideal.
The degrees of elements from I forms a semigroup in Z+ , that we denote by SI . We
know that numerical semigroups are finitely generated by Problem 197. Let n1 , . . . , nk
denote the ordered set of generators for SI and let f1 (x), . . . , fk (x) be monic elements
from I such that deg fi (x) = ni for i = 1, . . . , k. We claim that fi s generate I. We
use induction on SI . To this end, let J denote the ideal generated by fi (x)s, let
g(x) I be a polynomial of degree n1 , and let b denote the leading term of g(x). Then
g(x) bf1 (x) lies in I and its degree is less than n1 , hence g(x) = bf1 (x).
Next we assume that our claim is true for all m SI with m < N and let f (x) I
be an element of order N . Since i1 n1 + + ik nk = N for some
QiN1 , . . . , ikis Z+ , there
exists b F (the leading coefficient of f (x)) such that f (x) b s=1 fs (x) has degree
< N , hence it lies in J by our induction hypothesis. Thus f (x) lies in J. Therefore,
J = I, hence I is finitely generated. Therefore, by Problem 181 R is Noetherian.
To give a counter example to the UFDness of R consider R = F [x2 , x3 ]. Obviously,
both x2 and x3 are irreducible elements in R. The factorizations x6 = x2 x2 x2 and
x6 = x3 x3 shows that R is not a UFD.

199. Prove that submodules, quotient modules, and finite direct sums of Noetherian modules are Noetherian as well.
Solution.
The fact that submodules and quotient modules of a Noetherian module are Noetherian
follows from Problem 186. To prove that a finite direct sum of Noetherian modules is
Noetherian, let M1 , . . . , Mn be a finite list of Noetherian R-modules, and let {Sj }j1
be an ascending chain of R-modules in M1 Mn . Clearly, for any submodule
S M1 Mn that is true:
S = (S M1 ) (S Mn ),
and that S Mi is a submodule of Mi for each i = 1, . . . , n is true. Therefore, for each
i = 1, . . . , n the ascending chain {Sj Mi }j1 stabilizes, say at step mi . Let m denote
105

max{m1 , . . . , mn }. Then
Sm = Sm M1 Sm Mn = Sm+k M1 Sm+k Mn = Sm+k
for all k 1. In other words, M1 Mn is Noetherian.

200. If R is a Noetherian ring, then an R-module M is Noetherian if and only if


M is finitely generated.
Solution.
Suppose M is a Noetherian R-module. Let M1 be a submodule generated by one
element f1 from M , and let Mi (for each i = 2, . . . ) be the R-module generated by
Mi1 and an element fi M , if there is any, that is not contained in Mi1 . Thus we
have an increasing sequence of R-modules: M1 M2 . Since M is Noetherian,
this chain stabilizes, say at i = n, hence Mn = M . In other words, the elements
f1 , . . . , fn+1 generate M .
Conversely, suppose M is a finitely generated R-module, say by the elements f1 , . . . , fn
M , and let M1 M2 be an ascending chain of R-submodules in M . Since R
is a Noetherian ring, it is a Noetherian R-module, also. Thus, by Problem 199, the
direct sum R R (n copies), denoted by Rn , is P
a Noetherian R-module as
n
well. The map : R M defined by (r1 , . . . , rn ) = ni=1 ri fi is a surjective Rmodule map. The pre-images of Mi s in Rn given an ascending chain of R-submodules:
1 (M1 ) 1 (M2 ) 1 (M3 ) . Since Rn is Noetherian (Problem 199) this chain
stabilizes, say at the kth step. Thus, their images in M stabilize at the kth step as
well. In other words, M is Noetherian.

201. Recall that rad(I), the radical of an ideal I is the set of all elements r R
such that rn I for some n N. Show that the rad(I) is equal to the
intersection of all prime ideals containing I.
Solution.
Recall that the radical of a ring is the intersection of all prime ideals of the ring and it
is equal to the set of all nilpotent elements (Problem 41). If I R is an ideal, then the
radical of R/I is the set of nilpotent elements in R/I and also equal to the intersection
of all primes of R/I. Since the prime ideals of R/I are in one-to-one correspondence
with the prime ideals of R that contain I in an inclusion preserving manner, we see
that the radical of R/I corresponds to the intersection of all primes containing I. At
the same time, the set of all nilpotent elements in R/I corresponds to the set of all
elements r of R with some power of r contained in I.

106

202. Let I and J be two ideals from R. Prove that the following are equivalent:
(a) V (I) V (J)
(b) J rad(I)
(c) rad(J) rad(I)
Solution.
If V (I) V (J), then a prime ideal containing I contains J also. Thus intersection
of primes that contain I contains the intersection of all primes that contain J. By
Problem 201, this shows that (a) (c).
If rad(J) rad(I), then since J rad(J) the implication (c) (b) is obvious.
Finally, if J is contained in rad(I), then J is contained in prime ideal that contains I.
Therefore, V (I) V (J).

203. Show that nilradical of R is equal to Jacobson radical of R if and only if


every open set contains a closed point.
Solution.
Recall that Jacobson radical is the intersection of all maximal ideals, and the radical
(also known as the nilradical) is the intersection of all prime ideals.
Suppose that nil(R) = rad(R). It is snough to prove our claim for basic open sets in
Spec(R). Let D(f ) be a non-empty basic open set. Recall that D(f ) is equal to set
of all prime ideals in R that does not contain f . Towards a contradiction assume that
D(f ) does not contain a closed point. Closed points corresponds to maximal ideals in
Zariski topology. In other words, we assume that D(f ) does not have any maximal
ideal. This means that all maximal ideals of R contains f . Therefore, f lies in the
Jacobson radical, hence it lies in the nilradical of R. Therefore, f is nilpotent. But
then D(f ) = by Problem 74.
Conversely, suppose that every open set contains a closed point and assume rad(J) 6=
Jac(R). In particular, it follows our hypothesis that all non-empty basic open sets
contain closed points. Equivalently, for each f R, if D(f ) 6= , then there exists a
maximal ideal Qf such that f
/ Qf . If f is contained in all maximal ideals but not
in a prime ideal, then it means D(f ) 6= . At the same time, if f Jac(R), f is
contained in all maximal ideals, in particular it has to be contained in Qf , which is a
contradiction. Therefore, our assumption is wrong.

204. Let : R S be a ring homomorphism. If I R is an ideal, then show that


(rad(I)) rad((I)). If is surjective and I contains the kernel of , then
show that (rad(I)) = rad((I)).
107

Solution.
Let x rad(I). Then xn I for some n. In this case, (x)n = (xn ) (I), therefore,
(x) rad((I)). Assume now that is surjective and its kernel is contained in I. Let
y rad((I)), and suppose x R is such that (x) = y. Since y m (I) for some m,
we see that (xm ) (I). Since the kernel of is contained in I we see that xm I.
In other words, x rad(I).

Definition 0.67. Let R S be a pair of rings. An element S that satisfies


f () = 0 for some monic f (x) R[x] is called an integral element of S over R. The
terminologies integral extension, integrally closed, integral closure are defined in a
way that they makes sense. An integral domain R is called normal if R is integrally
closed in its fraction field S = K.
205. Prove that UFDs are normal.
Solution.
Let R be a UFD and K be its fraction field. We have to show that any integral element
of K over R lies in R. This is the statement of Problem 105.

206. Let R denote the following quotient ring: R = F [x, y]/(x2 y 3 ) for some field
F . Show that
(a) R is an integral domain;
(b) if t denotes the element x/y in the fraction field K of R, then K is equal
to F (t);
(c) F [t] is the integral closure of R in K = F (t).
Solution.
(a) The ring F [x, y] is a UFD. Therefore, if x2 y 3 is irreducible, then the ideal
generated by (x2 y 3 ) is a prime ideal, hence R is an integral domain. To prove that
x2 y 3 is irreducible, it is enough to prove that it is irreducible in F (y)[x]. Assume
towards a contradiction that x2 y 3 is reducible. Since it is a quadratic polynomial in
(y)
is a root.
x over F (y), it is reducible if and only if it has a root in F (y). Suppose fg(y)

2
(y)
We assume that f (y) and g(y) have no common divisor. But fg(y)
y 3 = 0 implies
f (y)2 = g(y)2 y 3 . Thus an irreducible factor of g(y) divides f (y)2 , hence divides f (y),
a contradiction.

108

(b) Let t denote x/y in the fraction field K of R. Since x2 = y 3 in R, we see that y =
(x/y)2 = t2 in K. Also, x = yx/y = yt = t2 t = t3 . Thus, any element f (x, y) R =
F [x, y]/(x2 y 3 ) (viewed inside K) is a polynomial in t. Therefore, f (x, y)/g(x, y) K
with g(x, y) 6= 0 lives in F (t). On the other hand, F (t) K, hence, the equality
K = F (t) follows.

P
i
(c) Let h(t) = m
i=0 ai t be an element of F [t] F (t). Arguing as in part (b), we
see that h(t) = a1 t + g(x, y) for some g(x, y) from R (replace t2 with y and replace t3
with x). Since (h(t) g(x, y))2 = a21 t2 = a21 y, we see that h(t) is a root of the monic
polynomial X 2 2g(x, y)X + g(x, y)2 a21 y R[X]. This shows that F [t] is integral
over R. To prove that F [t] is the integral closure of R in F (t), it remains to show that
any integral element of F (t) over R lies in F [t]. To this end, let h(t) F (t) be an
integral element over R. In particular, since R F [t], h(t) is integral over F [t]. Since
F (t) is the fraction field of F [t], and since F [t] is a UFD, F [t] is integrally closed by
Problem 205. Therefore, h(t) belongs to F [t].

207. Let R be an integral domain and let i and j be two relatively prime numbers.
Show that (xi y j ) is a prime ideal in R[x, y].
Solution.
Let : R[x, y] R[t] be the ring homomorphism defined by (x) = tj and (y) = ti . It
is clear that the kernel of contains the ideal generated by xi y j . If f (x, y) R[x, y]
is mapped to 0 by we claim that it belongs to (xi y j ). To see this, we consider
f (x, y) in R[x, y]/(xi y j ): Replace each occurrence of y j in monomials of f (x, y) by xi
and write f (x, y) =P
f0 (x)+yf1 (x)+ P
+y j1 fj1 (x) for some fi (x) R[x, y]/(xi y j ).
j1
s
j
s
j
Since (f (x, y)) = s=0 (yfs (x)) = j1
s=0 t fs (t ), and since any monomial of t fs (t )
0
is distinct from that of ts fs0 (tj ) for 0 s 6= s0 < j, we see that ts fs (tj ) = 0 for all
s = 0, . . . , j 1. In other words, fs (x) = 0 for all s = 0, . . . , j 1, hence f (x, y) = 0
in R[x, y]/(xi j j ). It follows that the kernel of is equal to the ideal generated by
xi y j . Since R[ti , tj ] = im() is an integral domain, (xi y j ) is a prime ideal.

208. Let R denote the polynomial algebra F [x, y]/(xi y j ) for some field F and
relatively prime numbers i and j. We know from Problem 207 that R is an
integral domain. Find the normalization of R.
Solution.
We know from the solution of Problem 207 that the ring R = F [x, y]/(xi xj ) is
isomorphic to F [ti , tj ]. So we consider everything inside of the fraction field F (t) of
F [t].
109

By Problem 205, we know that F [t] is normal. We claim that the normalization S of
F [ti , tj ] is equal to F [t].
Since F [ti , tj ] F [t], it is clear that S is contained in F [t].
Conversely, let f (t) be a polynomial from F [t] and decompose it in the form f (t) =
h(t) + g(ti , tj ), where g(x, y) F [x, y] and h(t) F [t] with no monomial of h(t) is of
the form dtai+bj , a, b N, d F . In other words, no monomial of h(t) lies in F [ti , tj ].
Let N denote ij i j + 1. We know that any integer m greater than or equal to
N is of the form ai + bj for some a, b N (see the solution Problem 197). Therefore,
any monomial in h(t)N , in particular (f (t) g(ti , tj ))N = h(t)N , belongs to F [ti , tj ]. In
conclusion, f (t) is a root of the monic polynomial (X g(ti , tj ))N h(t)N F [ti , tj ][X],
hence it belongs to the normalization S of F [ti , tj ].

209. Let R denote the polynomial algebra F [x, y]/(y 2 x3 x2 ) for some field F .
(a) Prove that R is an integral domain;
(b) Compute the normalization of R.
Solution.
(a) We claim that y 2 x3 x2 is irreducible in F [x, y]. It is enough to prove that it is
irreducible in F (x)[y]. Assume towards a contradiction that y 2 x3 x2 is reducible.
Since the y-degree of y 2 x3 x2 is 2, it is reducible if and only if it has a root in
(x)
is a root. We assume that f (x) and g(x) have no common divisor.
F (x). Suppose fg(x)
2

(x)
x3 x2 = 0 implies f (x)2 = g(x)2 (x3 x2 ) = (g(x)x)2 (x 1). Thus,
But fg(x)
x 1 is a divisor of f (x), hence (x 1)2 divides f (x)2 . But since x is not divisible
by x 1, it follows that g(x) is divisible by x 1, which is a contradiction. Since
y 2 x3 x2 is irreducible in F [x, y], the ideal that it generates is prime. Therefore,
F [x, y]/(y 2 x3 x2 ) is an integral domain.
(b) Let t denote y/x in the fraction field K of R = F [x, y]/(y 2 x3 x2 ). In particular,
F (t) is contained in K.
On the one hand 0 = y 2 x3 x2 = x2 (y 2 /x2 x 1) = x2 (t2 x 1). On the other
hand K is an integral domain. Therefore, t2 x 1 = 0, or x = t2 1. In this case,
y = xy/x = (t2 1)t. It follows that any element from R is a polynomial in t, hence
R F [t]. Therefore, K F (t). Thus, K is equal to F (t).
Next we compute the integral closure S of R = F [x, y]/(y 2 x3 x2 ) in K = F (t).
P
i
Let f (t) = m
i=0 ai t be an element of F [t] F (t). Let s = 2k be an even number.

110

Then
s

2 k

t = (t ) = ((t 1) + 1) =

k  
X
k
i=0

(t 1) =

k  
X
k
i=0

xi .

If s = 2k + 1 is an odd number with s 3, by using eqn 18, we obtain


 !
s3 
X
s

3
ts = ts ts2 + ts2 = ts3 (t2 1)t + ts2 =
xi y ts2 .
i
i=0

(18)

(19)

Since s 2 is an odd integer, repeating this argument for ts2 , by induction we see
that ts is of the form g(x, y) + at for some a Z and a polynomial g(x, y) R. In
particular, combining (18) and (19) we see that f (t) is of the form h(x, y) + bt for some
b Z and a polynomial h(x, y) R. Then f (t) is a root of the monic polynomial
(X h(x, y))2 b2 b2 x2 R[X].
It follows that f (t) belongs to S. In other words, F [t] S.
But since R is contained in F [t], and F [t] is integrally closed, we know that S F [t].
Therefore, they are equal.

210. Let R S be a pair of rings and let s S. Prove that the following are
equivalent to each other:
(a) s is integral over R,
(b) the polynomial ring R[s] is a finitely generated R-module,
(c) there exists an intermediary ring R T S such that s T and T is
finitely generated as an R-module.
Solution.
(a) (b) If s is integral over R, there exist elements a0 , . . . , an1 R such that
a0 + a1 s + + an1 sn1 + sn = 0. In particular, if m n, then sm = smn (a0 +
a1 s + + an1 sn1 ). Therefore, by using this identity recursively we see that sm is an
R-linear combination of the elements 1, s, . . . , sn1 . Hence R[s] is a finitely generated
R-module.
(b) (c) Since R R[s] S, this claim is obvious.
(c) (a) Let s S be an element such that there exits an intermediary ring T
containing s which is finitely generated as an R-module. Let s1 , . . . , sn be a list of

111

generators for T as an R-module. In particular, we have ssi = ai1 s1 + + ain sn for


some coefficients ai1 , . . . , ain R. Thus, we have a matrix equation


s1
s1
a11 a1n

.. ..
.
.
..
.. ...
(20)
s . = .
.
sn

an1

ann

sn

Let A denote the coefficient matrix A = (aij ), and let u denote the column matrix of
si s. We re-write the matrix equation 20 in compact form as follows: (sI A)u = 0,
where I is the n n identity matrix. Notice that if det(sI A) is non-zero, then sI A
is invertible, hence u = 0. Since this is absurd, we see that det(sI A) = 0. But this
shows that there exists a monic polynomial with coefficients from R having s as a root.

211. Let R S be a ring extension, and let T S denote the set of elements that
are integral over R. Prove that T is a ring.
Solution.
Let r, s T be two elements. We are going to prove that r s T and rs T .
There exists elements s1 , . . . , sm and r1 , . . . , rm0 such that R[s] = Rs1 + Rsm and
P Pm0
R[r] = Rr1 + + Rrm0 . It is clear that S 0 = R[s, r] is equal to m
i=1
j=1 Rsi rj ,
hence finitely generated. By Problem 210, r s as well as rs are integral elements over
R since both of them belong to S 0 .

212. Let R S be a ring extension such that S is an integral domain and integral
over R. Prove that R is a field if and only if S is.
Solution.
Suppose R is a field. Since S is an integral extension of R, if s S, there exists a
finitely generated R-submodule T of S such that s T and T is a ring. In particular,
T is a finite dimensional vector space overPR, hence 1, s, s2 , . . . is linearly dependent:
there exists a0 , a1 , . . . , am R such that m ai si = 0. Without loss of generality we
assume a0 6= 0. Thus, a0 = s(a1 + + am sm1 ), so s is invertible, hence S is a field.
Conversely, if S is a field and r R, then we are going to show that 1/r belongs to
1
=0
R. Indeed, since 1/r S and S is integral over R, a0 + 1r a1 + + r1m am + rm+1
1
m
m1
for some ai R, i = 0, . . . , m. Equivalently, a0 r + a1 r
+ + am = r , which
implies that 1/r R.

213. Prove that if K is a finite extension of Q, then any element of K has an


integer multiple that is integral over Z.
112

Solution.
Let K be an element. Since K is algebraic over Q (K being finite dimensional
vector space over Q), there exist a0 , . . . , am Q such that a0 + a1 + + am m = 0.
Multiplying by the common denominator of ai s we assume that the coefficients are
integers. Multiplying by the m 1th power of am we obtain
m m
m3 2 2
+ a1 am2
a0 am1
m (am ) + a2 am (am ) + + am = 0.
m

Thus, am is an algebraic over Z.

214. Let K be a finite extension of Q, and assume that K is contained a field


L that is Galois over Q. Recall that the trace of an element K over
F is defined in Problem 170. Prove that if K is integral over Z, then
TrK/Q () is an integer.
Solution.
We know from Problem 170 that TrK/Q = nd ad1 , where n = dimQ K, d is the degree
P
of the minimal polynomial min,Q (x) = di=0 ai xi of over Q, and furthermore, d | n
(by Problem 169). Therefore, it is enough to prove that the minimal polynomial of
is defined over Z.
Since is integral over Z there is a monic polynomial f (x) with integer coefficients
such that f () = 0. Let f (x) be a smallest degree such polynomial, hence irreducible
over Z. But since Z is a UFD, by Problem 115 we know that f (x) is irreducible over Z
if and only if it is irreducible over Q. Since the minimal polynomial of divides f (x),
f (x) is not irreducible over Q unless it is equal to the minimal polynomial.

215. Prove that if K is a finite extension of Q, then the set S of integral elements
over Z of K forms:
(a) a ring;
(b) a free Z-module.
Furthermore, the rank of S as a Z-module is equal to n = dimQ K.
Solution.
The fact that S is a ring follows from Problem 211. Let 1 , . . . , n be a basis of K
as a Q-vector space. By Problem 213, there exists d Z such that d1 , . . . , dn S.
Obviously, the Z-module generated by di s is a free Z-submodule of S.
We claim that there are n elements that are related to di s (by some sort of duality)
and they form a basis for S as a Z-module, hence, S is free and furthermore its rank
is n = dimQ K.
113

By Problem 172 we know that K, as a vector space over F = Q, is isomorphic to


HomQ (K, Q) the space of Q-valued linear functionals on K via () = , where
() = TrK/F (). Let f1 , . . . , fn denote the dual of the basis d1 , . . . , dn . In other
words, fi = di , and fi (dj ) = 0 if i 6= j. We choose elements i0 from K such that
(
1 if i = j,
0
fj (i ) =
0 otherwise .
Now, let be an element from S and we expand it in the i0 -basis: If = a1 10 + +
an n0 for some ai Q. Multiplying by di and then applying fi we see that fi (di ) =
ai . Since fi (di ) is equal to TrK/Q (d) and since di S, by Problem 214 we know
that ai Z. Therefore, S is contained in the free Z-module spanned by 10 , . . . , n0 .
Since S does not have any torsion elements (as it is contained in a field of characteristic
0) S itself is a free Z-module. Since S is squeezed between two rank n free Z-modules,
its rank is n, also.

216. Suppose R S is an integral ring extension. Let D be a multiplicative


submonoid of R. Prove that
(a) D1 S is integral over D1 R;
(b) If P is a prime ideal of S, then S/P is integral over R/P R.
Solution.
(a) First of all, if D R is multiplicatively closed, then it is multiplicatively closed in
S as well, therefore, it makes sense to localize S at D.
Let /d, S, d D be an element of D1 S. Since is integral over R, there exists
a monic polynomial f (x) = xm + am1 xm1 + + a0 R[x] such that f () = 0.
Dividing both sides with dm we obtain
a0
f ()  m am1  m1
=
+
+ + m = 0.
m
d
d
d
d
d
Since the coefficients of the left hand side is from D1 R and this is a monic expression,
we see that /d is an integral element of D1 S over D1 R.
(b) If S and f (x) R[x] is a monoic polynomial such that f () = 0, then the
image of in S/P satisfies the polynomial f(x) (R/R P )[x] that is obtained from
f (x) by reducing its coefficients modulo P R. Since f (x) has a monic leading term,
f(x) is not identically zero. Therefore, the image of in S/P is integral over R/R P .

114

217. Let D be a multiplicative submonoid of R. Prove that there exists one-toone correspondence between prime ideals not intersecting D in R and prime
ideals of D1 R.
Solution.
Let denote the map sending a prime ideal I of R to the ideal generated by the image
of I in D1 R. It is clear that, if I D 6= , then (I) = D1 R. Therefore, we restrict
our attention to the prime ideals that do not intersect D.
Let r (J) for some other prime ideal J from R. Thus, r is of the form r1 s1 /d1 +
rk s1 /dk for some r1 , . . . , rk J, s1 , . . . , sk R and
Q d1 , . . . , dk D. Since s1 r1 /d1 +
0
0
sk rk /dk = d1 s1 r1 /d + + dk sk rk /d, where d = di , we see that r is of the form
r0 /d0 , where r0 J and d0 D. Thus, we identify the ideal generated by J in D1 R
with the set {r0 /d0 : r0 J, d0 D}.
Now, if (J) = (I), then r (I), hence there exists r00 I, d00 D such that
r0 /d0 = r00 /d00 , which implies that r0 I. In other words, J I. The same argument
shows that I J, hence I = J. Therefore, is injective.
Conversely, let U D1 R be a prime ideal and let I = U D1 R. We claim that I
is prime, I D = , and furthermore, (I) = U . If r, r0 R are two elements from R
such that rr0 I, then in particular rr0 D1 R. Thus, either r D1 R or r0 D1 R.
But r and r0 are from R, therefore, at least one of them is contained in I. In other
words, I is prime. It is clear that I D = , otherwise U contains a unit. Finally,
r/d U , then d r/d = r U . Since r is an element of R, we see that r I. It follows
that I generates U in D1 R.

218. Let P be a prime ideal in R and let D = R P denote the corresponding


multiplicative submonoid. Prove that there exists a unique maximal ideal
in D1 R (that is generated by P in D1 R).
Solution.
We know from Problem 217 that prime ideals of R not intersecting D are in one-to-one
correspondence with the prime ideals of D1 R. In particular the correspondence is
given by sending I to the ideal (I) generated by I in D1 R. It is obvious that
preserves the inclusion relation: I J implies (I) (J). Since the unique largest
prime ideal not intersecting D is P , there is a unique maximal ideal in D1 R.

219. Let R S be a pair of rings such that S is integral over R. If P is a prime


ideal in R, then prove that there exists a prime ideal Q in S such that
P = Q R.
Solution.
115

The idea is to analyze the situation locally. Let D denote the complement of P in R.
Then D is a multiplicative monoid. Notice that D is a multiplicative monoid in S as
well.
By Problem 216 we know that D1 S is integral over D1 R and also if M is a prime
ideal in D1 S then D1 S/M is integral over D1 R/D1 R M . Now, suppose M is
a maximal ideal in D1 S. (The existence of M is guaranteed by the Zorns lemma.)
Since D1 S/M is a field, by Problem 212 we see that D1 R/M D1 R is a field, also.
In other words, the ideal M0 := M D1 R is maximal in D1 R. Since D1 R has a
unique maximal ideal (generated by P ) we have M0 R = P . See Problem 218 .
Similarly, M S is a prime ideal in S and furthermore M does not intersect D.
Therefore, its restriction to R, namely M R does not intersect D neither. Thus it is
contained in P = R D. Combining this with the fact that M0 R = P , we see that
P = M.

220. Suppose P is a maximal ideal of R, and S is ring that contains R. If there


exists a finite number of elements s1 , . . . , sn S that generate S as a ring over
R (that is to say, the coefficients of the polynomial expressions in s1 , . . . , sn
are all from R), then prove that there are only finitely many maximal ideals
Q in S such that Q R = P .
Solution.
By Problem 219 we know that S has a prime ideal Q such that Q R = P . We claim
that there exists a maximal ideal M in S such that M R = P . If Q is maximal,
then there is nothing to do, so we assume that Q is contained in a maximal ideal M .
Clearly M R contains P . If this is a proper containment, since P is maximal, then
M R has to contain a unit, which is absurd. Therefore, M R = P .
Next, we claim that there only finitely many maximal ideals M of S such that M R =
P . Notice that, since M R = P , the canonical injection R , S induces an injection
R/P , S/M . Therefore, S/M is a field extension of R/P . We analyze the generators
of S/M as a field over R/P . It is clear that the images of si s generate S/M as field with
coefficients from R/P . Since si s are integral over R, we have polynomials fi (x) R[x]
such that fi (si ) = 0. Reducing the coefficients of fi s modulo P , we see that the
minimal polynomial over R/P of si s are divisors of fi (x) mod P . We conclude that
the field S/M is obtained from R/P by adjoining some roots of fi (x) mod P . Since
fi (x) has finitely many roots, there are finitely many different field extensions of the
form S/M over R/P . This proves that there are only finitely many different maximal
ideals of S that restricts to the same maximal ideal P in R.

221. Let S be an integral extension of R. Suppose P1 Pn is a sequence of


116

prime ideals of R and suppose that there exists a sequence of prime ideals
Q1 Qm of S such that Qi R = Pi for i = 1, . . . , m. Here, 1 m < n.
Prove that there exists prime ideals Qm+1 , . . . , Qn in S such that Qj Qj+1
and Qj R = Pj for all j = 1, . . . , n.
Solution.
This is a simple application of the Problem 219: We look at the integral extension
S/Qm of R/Pm . Since the image of Pm+1 in R/Pm is prime, there exists a prime ideal
Qm+1 in S, whose image in S/Qm lies over that of Pm+1 . It is clear that Qm+1 contains
Qm .

222. Let k be a positive integer. Prove that the following equalities are true for
two ideals I and J from a ring R:
(a) I k J = rad(I) rad(J);
(b) I k J I = rad(J) = rad(I);
(c) rad(IJ) = rad(I J) = rad(I) rad(J);
(d) rad(rad(I)) = rad(I);
(e) rad(I) + rad(J) rad(I + J) = rad(rad(I) + rad(J)).
Solution.
(a) We start with an easy observation: if I1 and I2 are two ideals such that I1 I2 P
for some prime ideal P , then either I1 P , or I2 P . Indeed, assuming otherwise
we find two elements a P I1 and b P I2 such that ab P . Since P is prime,
this produces a contradiction. This observation implies also that if P contains a power
of an ideal I k , then I P . Since rad(I) is the intersection of all primes containing
I (Problem 201), we see that rad(I) rad(I k ). On the other hand, as I k J, any
element x R such that xm I k satisfies xm J. Equivalently, rad(I k ) rad(J).
Therefore, rad(I) rad(J).
(b) By part (a) we know that rad(I) rad(J). Since J I = rad(J) rad(I)
the equality follows.
(c) Since IJ I J, we see that rad(IJ) rad(I J). Conversely, let x R be
an element of rad(I J) that is to say xm I J for some m 1. Then xm I
and xm J. Thus x2m IJ. It follows that x rad(IJ). (Almost) the same
argument shows that rad(I) rad(J) rad(I J). Since I J I, J we see that
rad(I J) rad(I), rad(J). Therefore, rad(I J) = rad(I) rad(J).
(d) The inclusion rad(rad(I)) rad(I) follows from the definition of radical ideals.
Conversely, I rad(I) implies that rad(I) rad(rad(I)).

117

(e) Let x rad(I), y rad(J) be two elements. Then xm I and


y n for some

positive integers m, n. Then terms of (x + y)mn are of the form k+l
xk y l with k +
k
 k l
l = mn. In this case, either k m, or l n. Therefore, either k+l
x y I
k
 k l
k+l
mn
or k x y J. It follows that (x + y)
I + J. Finally, we prove the last
equality. Since rad(I) + rad(J) rad(I + J), taking radicals of both sides implies that
rad(rad(I) + rad(J)) rad(rad(I + J)) = rad(I + J) (by the previous part of the
problem). Conversely, if x rad(I + J), then there exists m 1 such that xm I + J.
Since I + J rad(I) + rad(J), we have xm rad(I) + rad(J). In other words, x
belongs to the radical of rad(I) + rad(J). This finishes the proof.

223. Suppose R is a Noetherian local ring. Prove that if M is the unique maximal
ideal of R and dimR/M M/M 2 is a finite dimensional vector space over R/M .
If d = dimR/M M/M 2 , then prove that any generating set for M has at least
d elements.
Solution.
Let U denote M/M 2 . For m M and r R we denote by m and r the images of
m and r in U and R/M , respectively. By the action r m = rm of R/M , U becomes
a module over R/M . Since R is Noetherian, every ideal in R is finitely generated.
In particular M is finitely generated. The image in U of a generating set for M is a
generating set for U as an R-module. In particular, U is finitely generated R-module.
But any element from M acts as a zero on U , thus U is a finitely generated R/M module, hence a finite dimensional vector space. Let d = dimR/M M/M 2 be the vector
space dimension of M/M 2 and let x1 , . . . , xn be a list of generators of M as an ideal.
Since x1 , . . . , xn in U is a spanning set we see that n d.

224. Suppose R is a Noetherian integral domain. We assume further that R is


a local ring of Krull dimension 1. Prove that if M is the unique maximal
ideal of R and dimR/M M/M 2 = 1, then R is a DVR.
Solution.
Since R/M is a field and since dimR/M M/M 2 = 1, we have an element t M such
that t is the basis vector for M/M 2 .
We claim that M n /M n+1 is an R/M -vector space spanned by the image of tn in
M n /M n+1 . We prove this by induction on n. n = 1 case is given by the hypothesis and it is clear that M n /M n+1 is a vector space over R/M . Let y M n /M n+1 be
any element. Then y = x1 y1 + + xr yr for some xi M , yi M n1 . Write xi in
the form ai t + p where ai R is a unit and p M 2 . Thus, xi yi = ai tyi + pyi . By
induction hypothesis yi is of the form ci tn1 + q where ci R and q M n . Thus,
118

ai tyi + pyi = ai ci tn + ai tq + pyi . Since ai tq and pyi are from M n+1 , and since ai ci is a
unit, we see that xi yi modulo M n+1 is equal to ai ci tn , hence our claim follows.
Next, we show that R is a DVR by exhibiting a valuation for R. By the above discussion
we see that any element x of R is of the form atn modulo M n+1 for some unit a R.
Furthermore, the exponent n is unique. Thus, the function : R Z0 defined by
(x) = n is well-defined. We extend to the fraction field K of R by defining it to
be (x/y) = (x) (y). Well-definedness of the extension is clear. Checking that
satisfies the properties of a valuation on K is straightforward.

225. Suppose R is a Noetherian integral domain. We assume further that R is


a local ring of Krull dimension 1. Prove that if M is the unique maximal
ideal of R and if every ideal is a power of M , then R is a DVR.
Solution.
Let x1 , . . . , xn M be a minimal list of generators for M . Without loss of generality
we assume that x1
/ M 2 . The principal ideal I generated by x1 is a power of M
by hypothesis: I = M k for some k 1. Since x1
/ M 2 , in particular, x1
/ M k for
any k > 1. Therefore, I = M . In particular, M/M 2 is spanned by x1 . Therefore,
dimR/M M/M 2 = 1, hence by Problem 224 the problem is finished.

226. Prove that if R is a DVR, then R is a PID. Conversely, if R is a PID with


a unique maximal ideal, then R is a DVR.
Solution
() Let K denote the fraction field of R, and let : K Z denote the valuation of R.
Thus R = {x K : (x) 0}. First of all, we claim that the set {y R : (y) = 0}
is the set of invertible elements of R. Indeed, 0 = (1) = (y/y) = (y) + (1/y)
implies that if (y) = 0, then (1/y) = 0, hence 1/y R.
Next we show that M = {y R : (y) > 0} is the unique maximal ideal and it is
generated by a single element. The maximality of M is clear. To see that M is an
ideal let y M and z R. Then (zy) = (z) + (y) > 0, therefore, zy M .
Finally, we show that R is a PID. Let I R be an ideal. Let z I be an element with
(z) = m. Also, let t M be an element such that t M M 2 . In this case (t) = 1.
Since (ztm ) = (z) + (tm ) = m m = 0, we see that ztm is an invertible element
of R. Therefore, z = atm , where a R is a unit. In other words, I is generated by tn ,
where n is the smallest power such that tn I.
() Define a valuation on K as follows: if r R, then (r) = n; the unique nonnegative integer such that r M n M n+1 . It is straightforward to verify that
satisfies the axioms of being a valuation.
119

You might also like